Download as pdf or txt
Download as pdf or txt
You are on page 1of 68

I0 and rank-into-rank axioms

Vincenzo Dimonte∗
July 11, 2017
arXiv:1707.02613v1 [math.LO] 9 Jul 2017

Abstract
Just a survey on I0.
Keywords: Large cardinals; Axiom I0; rank-into-rank axioms; elemen-
tary embeddings; relative constructibility; embedding lifting; singular car-
dinals combinatorics.
2010 Mathematics Subject Classifications: 03E55 (03E05 03E35 03E45)

1 Informal Introduction to the Introduction


Ok, that’s it. People who know me also know that it is years that I am ranting
about a book about I0, how it is important to write it and publish it, etc. I
realized that this particular moment is still right for me to write it, for two
reasons: time is scarce, and probably there are still not enough results (or
anyway not enough different lines of research). This has the potential of being a
very nasty vicious circle: there are not enough results about I0 to write a book,
but if nobody writes a book the diffusion of I0 is too limited, and so the number
of results does not increase much.
To avoid this deadlock, I decided to divulge a first draft of such a hypothetical
book, hoping to start a “conversation” with that. It is literally a first draft, so it
is full of errors and in a liquid state. For example, I still haven’t decided whether
to call it I0 or I0 , both notations are used in literature. I feel like it still lacks a
vision of the future, a map on where the research could and should going about
I0. Many proofs are old but never published, and therefore reconstructed by
me, so maybe they are wrong. Maybe even well-known proofs are wrong: for
example the only published proof of 4.6 has something missing, and this was
discovered decades after, with a question on MathOverflow. So please, consider
this paper as very temporary, and if you spot some problem, do not hesitate to
write me!
Finally, if for some reason you need some result on this paper, do note cite
it! Unfortunately, given the direction bibliometric is going, citing this paper is
completely useless for my career, even harmful. So, as long as the world does
not wake up from this folly, please try to locate the original paper where the
result is located and cite that (and if you’re thankful, cite a random paper of
mine, since this is what mathematics is now1 ). I submitted a shorter version of
this survey to a journal, if it is published please cite that.
∗ Università degli Studi di Udine, via delle Scienze, 206 33100 Udine (UD) E-mail address:

vincenzo.dimonte@gmail.com
1 Of course, this is a joke on the current state of events, not to be taken as a serious

suggestion

1
It would be unfair to ignore the fact that there is already an excellent survey
on I0, but I feel it is done with a different scope. If you are interested in I0,
please read also [4], as it nicely complement the paper you are reading now.

2 Introduction
In 1984 Woodin showed [. . . ] that ADL(R) follows from a proposition
stronger than I1 and straining the limits of consistency [. . . ] More-
over, Woodin’s investigations of I0 was to produce a detailed and
coherent structural theory for L(Vδ+1 ).

With attention shifted upward to the strongest large cardinal hy-


potheses, Woodin boldly formulated I0 just at the edge of the Kunen
inconsistency and amplified Martin’s embeddings proof to establish
the following in early 1984:

Theorem 2.1 (Woodin). Assume I0. Then ADL(R) .


This sparse lines in Kanamori’s The Higher Infinite [21] were probably for
years the only published trace of the mysterious I0. This fact is even more
surprising as [21] was written in 1994, so ten years after the striking result and
even more from its first formulation. What happened? Why all this mystery?
I0 was just one of the many large cardinals that were created in a short
span of time: strong cardinals, supercompact cardinals, huge cardinals, Woodin
cardinals, rank-into-rank axioms (its smaller siblings), . . . were introduced in
a rapid fire, shifting the paradigms of set theory. But I0 could not keep up
the pace with all of them: its main victory was short-termed, as ADL(R) was
proven to be of much lower consistency strength, there were no other examples
of important propositions implied by I0, and it was definitely too large for inner
model theory (in fact, the larger cardinal axiom of all times). So, before even a
publication, it quietly disappeared, and with it the proof of Theorem 2.1.
Still, under the surface there was some activity: the bound between I0 and
ADL(R) ran deeper than just Theorem 2.1, it involved the whole structure of
a model of the form L(Vλ+1 ), that under I0 had properties very similar than
those in L(R) under AD. This peculiarity was explored by Woodin, and the first
output was Kafkoulis’ lecture notes An AD-like axiom [39] (at that time I0 was
called Axiom A), a transcription of Woodin’s lectures on the argument, that
started to be copied and distributed among mathematicians at the beginning
of the Nineties. This underground distribution was, of course, very limited
(Internet was at its infancy).
In the meantime the rank-into-rank axioms were enjoying a period of pop-
ularity, thanks to their connections with algebra, and Laver, in 2001, tried to
push some of his results on rank-into-rank (in particular reflection) towards I0.
In [28] we can find therefore the first officially published fragments of the theory
of I0, taken from Kafkoulis notes, and in 2004 Kafkoulis himself published a
paper on Coding Lemmata that built on such notes [20]. After that, still silence
for many years.
The rhythm of publications changed abruptly in 2011. The reason is Wood-
ins’ Suitable extender models magnum opus. The aim of such paper (published
in two parts) was to push inner model theory up to all large cardinals, so it was

2
necessary to know as much as possible as the top of the hierarchy. Therefore
in the sprawling 561 pages there are whole sections dedicated to I0, the struc-
ture of L(Vλ+1 ) under it (in a much more developed way respect to Kafkoulis
notes) and even stronger axioms. There was a new exciting realm to explore,
finally mature, and with connections to other fields of set theory. The people
that were Woodin’s students in that period (the author included) continued the
work, expanding the research on I0 in different directions.
Such a rapid expansion, after a long silence, carries the risk of excessive
fragmentation. It is then time to take stock of the situation and pose solid
bases on such a venture, not based on underground notes, folklore, and oral
tradition. The objective of this paper is therefore to collect all the results on
rank-into-rank axioms and I0, with a stress on the latest one, both from the I0
folklore and from the published papers, so that anybody can have all the tools
for reading the latest papers or doing research on her own.
Section 3 glances briefly at some important large cardinals, up until the
first inconsistency in the large cardinal hierarchy, i.e., the (non-)existence of
j :V ≺V.
Starting from this, Section 4 is about rank-into-rank axioms, i.e., axioms of
the form j : Vλ ≺ Vλ (I3) and j : Vλ+1 ≺ Vλ+1 (I1), with a particular attention
to the application operator.
In Section 5 I0 is introduced, and with it all the structural properties of
L(Vλ+1 ) that we can have for free. Much of this part is taken from [39].
Section 6 is a detailed analysis of many (all?) axioms from I3 to I0, with all
the proofs of strong implications among them. The techniques of inverse limits
and square roots are introduced, that are now a focal centre of the investigations
around I0 (see [3]).
Section 7 uses again [39] as main source, introducing all the results about
similarities between I0 and ADL(R) , including the newest developments: mea-
surability of λ+ , Coding Lemma, large cardinal properties of Θ, perfect set
property, an analogous of weakly homogenous Suslin-ness.
In Section 8 all these axioms are treated in relation to forcing, with many
results of indestructibility and independence results.
In Section 9 it is noticed how such independence result can actually destroy
the similarities between I0 and ADL(R) : the examples treated are the Ultrafilter
Axiom, Wadge Lemma, a partition property and Turing Determinacy.
Section 10 casts a quick glance on axioms stronger than I0, introduced in
[41]: the Icarus sets.
Section 11 describes highly speculatively some possible future lines of inquiry.

3 Kunen’s Theorem
To reach the top of the large cardinal hierarchy, it is worth to have a glance at
the lower levels before. In the bottom, large cardinal axioms are very combina-
toric in nature: an inaccessible cardinal is a cardinal that is regular and closed
under exponentiation, a Ramsey cardinal is a cardinal that satisfies a particular
partition property, a Rowbottom cardinal is a cardinal that satisfies another
partition property, and so on2 . The pivotal moment was when Scott proved
2 Admittedly, this is a very rough classification, as for example indescribable cardinals are

difficult to insert in this narrative

3
that a combinatoric property on a cardinal is equivalent to a global property of
the universe:
Definition 3.1 (Ulam, 1930). A cardinal κ is measurable iff there exists a
κ-complete ultrafilter on κ.
Definition 3.2. Let M, N be sets or classes. Then j : M → N is an elemen-
tary embedding and we write j : M ≺ N iff it is injective and for any formula
ϕ and a1 , . . . , an ∈ M , M  ϕ(a1 , . . . , an ) iff N  ϕ(j(a1 ), . . . , j(an )).
Fom now on, if we write j : M ≺ N we mean that j is not the identity.
So an elementary embedding is a homeomorphism of the logical structure.
In particular, M and N must satisfy the same sentences, so for example M  ZF
iff N  ZF. It is easy to see that for all ordinals α, j(α) ≥ α. If M  AC or
N ⊆ M , than it is possible to prove that there must exist an ordinal α such
that j(α) > α.
Definition 3.3. Let M, N be sets or classes such that M  AC or N ⊆ M .
Then the critical point of j, crt(j), is the least α such that j(α) > α.
Theorem 3.4 (Scott, 1961). A cardinal κ is measurable iff there exist an inner
model M and a j : V ≺ M such that crt(j) = κ.
This was the start of an intense research on new breed of large cardinal
hypotheses, defined more requirements for M . For example, one can ask how
much of the original universe is in M :
Definition 3.5 (Gaifman, 1974). A cardinal κ is γ-strong iff there exists j :
V ≺ M , crt(j) = κ, γ < j(κ) and Vκ+γ ⊆ M .
Or one can ask how close M is. The starting point is to notice that if
j : V ≺ M witnesses that κ is measurable, then M κ ⊆ M .
Definition 3.6 (Solovay, Reinhardt). A cardinal κ is γ-supercompact iff there
exists j : V ≺ M , crt(j) = κ, γ < j(κ) and M γ ⊆ M .
It is immediate to see that if κ is γ-supercompact then it is γ-strong. All
these definitions fix j(κ) as the limit of the closeness of M . Overcoming this
means to define stronger large cardinals:
Definition 3.7 (Gaifman, 1974). A cardinal κ is superstrong iff there exists
j : V ≺ M , crt(j) = κ and Vj(κ) ⊆ M .
Definition 3.8 (Kunen, 1972). A cardinal κ is huge iff there exists j : V ≺ M ,
crt(j) = κ and M j(κ) ⊆ M .
We can go on, defining stronger and stronger large cardinals:
Definition 3.9. Let M, N be sets or classes such that M  AC or N ⊆ M .
Then the critical sequence of j : M ≺ N , hκn : n ∈ ωi, is defined as:
• κ0 = crt(j);
• κn+1 = j(κn ).
Definition 3.10. A cardinal κ is n-superstrong iff there exists j : V ≺ M ,
crt(j) = κ and Vj(κn ) ⊆ M .

4
Definition 3.11. A cardinal κ is n-huge iff there exists j : V ≺ M , crt(j) = κ
and M j(κn ) ⊆ M .
This ascent eventually must stop. In 1969 Reinhardt, in his thesis, asked
whether it is possible to have directly j : V ≺ V . Kunen, few months later,
proved that it is inconsistent. Such result must be approached cautiously: if we
work in ZFC, it is not surprising:
Theorem 3.12 (Naive Kunen’s Theorem). (ZFC) There is no j : V ≺ V .
Proof by Suzuki, [38]. As we are working in ZFC, it means that j must be a
class that is definable with some first order formula. Let ϕ be a formula. Let a
be a parameter so that ϕ(a) defines an elementary embedding j from V to itself
with least critical point κ among all the possible choices of the parameter. Then
κ is definable in V , without parameters. So j(κ) satisfies the same definition,
and it is the least critical point of embeddings definable with ϕ. But j(κ) > κ,
contradiction.
Kunen’s Theorem is stronger than this, as it involves also classes that are
not definable: in the original paper, [24], the proof is in MK, Morse-Kelley
theory. It is possible to weaken the theory to NBG, or to ZFC(j), i.e., ZFC with
j as a predicate that can be inserted in replacement formulas. We leave the
interested reader to [23], where all such problems are solved in great detail, and
we state Kunen’s Theorem in vague terms, with the only provisos that j is not
necessarily definable in V , replacement for j holds and AC holds in V .
Theorem 3.13 (Kunen’s Theorem, [24]). (AC) There is no j : V ≺ V .
There are many proofs of this result. Instead of the original proof by Kunen,
we show the proof by Woodin ([21], Theorem 23.12, second proof), as it gives
information that will be useful in Section 5. It builds on the following classical
result by Solovay:
Theorem 3.14 (Solovay). Let κ > ω be a regular cardinal, and X ⊆ κ sta-
tionary. Then X can be partitioned
S in κ disjoint stationary sets, i.e., there is
hXη : η < κi such that X = η<κ Xη and all the Xη ’s are pairwise disjoint.
It will be useful this technical remark, that will be used constantly in the
paper without comment:
Remark 3.15. Let j : M ≺ N , M  AC or N ⊆ M . Then for any γ < crt(j)
and any sequence haα : α < γi ∈ M , j(haα : α < γi) = hj(aα ) : α < γi.
So if A ∈ M is well-orderable (for example M  AC) and |A| < crt(j), then
j(A) = {j(a) : a ∈ A}.
Proof. Consider the sequence as a function f ∈ M with domain γ. As γ < crt(j),
j(f )(α) = j(f )(j(α)) = j(f (α)).
Proof of 3.13 by Woodin, [39]. Let λ be the supremum of the critical sequence
of j. Then λ must be a fixed point for j, because

j(λ) = j(suphκn : n ∈ ωi) = sup j(hκn : n ∈ ωi) = suphj(κn ) : n ∈ ωi = λ.


+ +
Let Sωλ = {α < λ+ : cof(α) = ω}. Note that Sωλ is stationary: if C is a club
in λ+ , than any ω-limit of elements of C must be in C, as λ+ is regular, and

5
+
has cofinality ω 3 . We prove that Sωλ cannot be partitioned in crt(j) disjoint
stationary sets, in contradiction with Solovay’s result.
+
Suppose that hSα : α < crt(j)i is a partition in stationary subsets of Sωλ .
Then
j(hSα : α < crt(j)i) = hTβ : β < j(crt(j))i
+
must be a partition in stationary subsets of Sωλ (because j(λ+ ) = j(λ)+ = λ+ )
of length j(crt(j)). Let C = {α < λ+ : j(α) = α}. Then C is an ω-club in λ+ :
If hαn : n ∈ ωi is a sequence of ordinals in C, then

j(suphαn : n ∈ ωi) = sup j(hαn : n ∈ ωi) =


= suphj(αn ) : n ∈ ωi = suphαn : n ∈ ωi.

To see that it is unbounded, let α < λ+ , and consider the sequence α0 = α,


αn+1 = j(αn ). Then suphαn : n ∈ ωi ∈ C.
As Tcrt(j) is stationary, it must intersect the closure of C in some η. But as
η ∈ Tcrt(j) , it has cofinality ω, so it must be in C, therefore j(η) = η. Since
hSα : α < crt(j)i was a partition, there must be some α such that η ∈ Sα . But
then η = j(η) ∈ j(Sα ) = Tj(α) . But crt(j) cannot be in the image of j (it is not
a fixed point, and below it they are all fixed points), so Tcrt(j) 6= Tj(α) , so they
should have been disjoint, but they have η in common. Contradiction.
We can now analyze such proof, to understand what it really needs of the
hypotheses, with the objective of defining new axioms for which the proof does
not work, so that there is a hope that they are not inconsistent. The first remark
is that Solovay’s Theorem uses AC in an essential way, so the proof would not
stand without AC.

Definition 3.16. (ZF)[Reinhardt] A cardinal κ is Reinhardt iff there exists


j : V ≺ V , with crt(j) = κ.
It is not immediate to see, but a Reinhardt cardinal is consistently stronger
than all the hypotheses we will introduce in this paper. It is still open whether
a Reinhardt cardinal would imply a contradiction, and the study of Reinhardt
and even larger cardinals is still going on.
Another approach would be to weaken the ground theory, for example not
permitting replacement with j, so that the critical sequence is not in V and λ
is not definable. This is the “Wholeness Axiom”, and it was studied by Paul
Corazza ([1]).
A third way would be to consider something smaller than V (but large
enough so that the eventual large cardinal would sit at the top of the hierarchy).
This will be implemented in the next section.

4 Rank-into-rank embeddings
Analyzing the proof of 3.13, one realizes that the key object is hSα : α < crt(j)i.
Now, if β < λ+ , there is a well-order of λ that has length β, and such well-order
can be coded as a subset of λ. Therefore every ordinal less then λ+ can be
+
3 In fact, S λ is ω-stationary, that is, it intersects all the ω-clubs, i.e., the sets that are
ω
cofinal in λ+ and closed under supremum of ω-sequences

6
coded as an element of Vλ+1 , and a subset of λ+ can be coded as an element of
Vλ+2 . So the whole proof can be carried on for:
Corollary 4.1. For any η, there is no j : Vη+2 ≺ Vη+2 .
Proof. Suppose that there is j : Vη+2 ≺ Vη+2 . Let λ be the supremum of its
critical sequence. Then λ ≤ η, because λ is limit. Then Vλ+2 ⊆ Vη+2 , and the
proof of 3.13 gives the contradiction.
Therefore the proof of 3.13 does not exclude the following cases:

I3 There exists j : Vλ ≺ Vλ ;
I1 There exists j : Vλ+1 ≺ Vλ+1 .

There is a small ambiguity in the definition of I3. On one hand λ must be at


least the supremum of the critical sequence (because ran(j) ⊆ Vλ ), on the other
hand λ must be less then the supremum of the critical sequence +2, because
of the Corollary, but, as it is written, it can be the supremum of the critical
sequence +1. In general we will assume, without loss of generality, that λ is the
supremum of the critical sequence:

I3 There exists j : Vλ ≺ Vλ , where λ is the supremum of its critical sequence.

Admittedly, not being excluded by a proof is not the most solid of founda-
tions for new axioms. In fact, the “I” in their name stands for “Inconsistency”,
as this was the first thought of the community when they were introduced. Yet,
they survived all attempts of proving their inconsistency, and they are now in
the center of a rich and complex theory, that has many interesting results, some
of which will be shown in this paper.
Let j : Vλ ≺ Vλ . The mental picture is the following:

λ λ
.. ..
. .
κ3 .. κ3
.
κ2 κ2

κ1 κ1

κ0 κ0

so Vλ is divided in two: under a certain rank, all sets are fixed, above a certain
rank, all sets are moved. There are ω “stripes”, and all the sets on one stripe
are moved into the next. There is one stripe, then, that contains no image of j.
The members of the critical sequence are large cardinals:

7
Remark 4.2. Let hκn : n ∈ ωi be the critical sequence of j : Vλ ≺ Vλ . Then all
the κn ’s are measurable.
Proof. Consider the ultrafilter Uj = {X ∈ P(κ0 ) : κ1 ∈ j(X)}. As usual, it is a
κ0 -complete measure, so κ0 is measurable. Then by elementarity j(κ0 ) = κ1 is
measurable, and so on.
Therefore λ is strong limit: if α < λ, then there is a κn such that α < κn ,
and 2α < κn < λ. So |Vλ | = λ, and we can code every pair of sets in Vλ (in
fact every finite sequence of sets) with a single set of Vλ . Therefore we can code
every λ-sequence of elements in Vλ+1 as a single element in Vλ+1 (coding the
sequence haη : η < λi with {(η, x) : x ∈ aη }).
The “stripes” structure permits to extend j beyond its scope:
Definition 4.3. Let j : Vλ ≺ Vλ . Define j + : Vλ+1 → Vλ+1 as
[
∀A ⊂ Vλ j + (A) = j(A ∩ Vβ ).
β<λ

While it is not clear whether j + is an elementary embedding, every elemen-


tary embedding from Vλ+1 to itself is the ‘plus’ of its restriction to Vλ :
Lemma 4.4. If j : Vλ+1 ≺ Vλ+1 , then (j ↾ Vλ )+ = j. Thus every j : Vλ+1 ≺
Vλ+1 is defined by its behaviour on Vλ , i.e., for every j, k : Vλ+1 ≺ Vλ+1 ,

j=k iff j ↾ Vλ = k ↾ Vλ .

Proof. The critical sequence hκn : n ∈ ωi is a subset of Vλ , so it belongs to


Vλ+1 . But then for every A ⊆ Vλ , {A ∩ Vκn : n ∈ ω} ∈ Vλ+1 , so
[
(j ↾ Vλ )+ (A) = (j ↾ Vλ )(A ∩ Vκn )
n∈ω
[
= j(A ∩ Vκn )
n∈ω
[
= j( (A ∩ Vκn ))
n∈ω
= j(A).

It is worth noting that there is a strong connection between first-order for-


mulas in Vλ+1 and second-order formulas in Vλ . In fact, all the elements of Vλ+1
are subsets of Vλ , so they can be replaced with predicate symbols:
Lemma 4.5. Let A ∈ Vλ+1 \ Vλ and ϕ(v0 , v1 , . . . , vn ) be a formula. Fix Â
a predicate symbol, and define ϕ∗ (v1 , . . . , vn ) in the language of LST expanded
with  as following:
• for every occurrence of v0 , substitute Â;
• for every non-bounded quantified variable x, substitute every occurrence of
x with X, a second-order variable.

8
Then for every a1 , . . . , an ∈ Vλ

Vλ+1  ϕ(A, a1 , . . . , an ) iff (Vλ , A)  ϕ∗ (a1 , . . . , an ).

Proof. The proof is by induction on the complexity of ϕ.


The previous Lemma is key in clarifying the relationship between elementary
embeddings in Vλ and Vλ+1 , and to finally prove that j + is a Σ0 elementary
embedding from Vλ+1 to itself (or, alternatively that j is a Σ10 elementary em-
bedding from Vλ to itself).
Theorem 4.6. Let j : Vλ ≺ Vλ . Then j + : Vλ+1 → Vλ+1 is a ∆0 -elementary
embedding.
Proof. Let  be a symbol for an 1-ary relation, â1 , . . . , ân symbols for constants
and let ϕ be a formula in LST∗ , the language of LST expanded with  and
â1 , . . . , ân . We want to prove that for any A ⊆ Vλ :

(Vλ , A, a1 , . . . , an )  ϕ iff (Vλ , j + (A), j(a1 ), . . . , j(an ))  ϕ.

Actually we will prove only one direction for every ϕ, the other one following
by considering ¬ϕ.
First of all, we skolemize ϕ, so we find f1 , . . . , fm functions, fi : (Vλ )mi → Vλ ,
such that ϕ is equivalent to a formula ϕ∗ in LST∗ expanded with fˆ1 , . . . , fˆm ,
where ϕ∗ is ∀x1 ∀x2 . . . ∀xm ϕ′ , with ϕ′ a ∆0 formula.
Let t0 (x1 , . . . , xm ), . . . , tp (x1 , . . . , xm ) be all the terms that are in ϕ′ . We
can express as a logical formula the phrase “ti (x1 , . . . , xm ) exists“: we work by
induction on the tree of the term, writing a conjuction of formulas in this way
• every time that there is an occurrence of a function, i.e. fˆi (t′ ), we add to
the formula ∃y fi (t′ ) = y;
• every time that there is an occurrence of a constant, i.e. âi , we add to the
formula âi 6= ∅, if ai 6= ∅; otherwise we don’t write anything.
Then we have that

(Vλ , A, a1 , . . . , an , f1 , . . . , fm )  ϕ∗ iff
∀δ < λ (Vδ , A ∩ Vδ , a′1 , . . . , a′n , f1 ∩ Vδ , . . . , fm ∩ Vδ ) 
^
∀x1 , . . . , ∀xm ( ti (x1 , . . . , xm ) exists → ϕ′ ),
i<p

where a′i is ai if ai ∈ Vδ , otherwise is ∅. This is true because, with δ fixed, if


only one term does not exist the formula is satisfied, and if all the terms exist,
then they are a witness for the satisfaction (or not) of ϕ′ .

9
So

(Vλ , A, a1 , . . . , an )  ϕ
↔ (Vλ , A, a1 , . . . , an , f1 , . . . , fm )  ϕ∗
↔ ∀δ < λ (Vδ , A ∩ Vδ , a′1 , . . . , a′n , f1 ∩ Vδ , . . . , fm ∩ Vδ ) 
^
∀x1 , . . . , ∀xm ( ti (x1 , . . . , xm ) exists → ϕ′ )
i<p

↔ ∀δ < λ (Vδ , j(A ∩ Vδ ), j(a′1 ), . . . , j(a′n ), j(f1 ∩ Vδ ), . . . , j(fm ∩ Vδ )) 


^
∀x1 , . . . , ∀xm ( ti (x1 , . . . , xm ) exists → ϕ′ )
i<p

← (Vλ , j + (A), j(a1 ), . . . , j(an ), j + (f1 ), . . . , j + (fm ))  ϕ∗


↔ (Vλ , j + (A), j(a1 ), . . . , j(an ))  ϕ.

The only thing left to prove is the right direction of the fifth line. The bump
is in the fact that if a j + (fi ) is not total, then it would be possible to have
the equation in the fourth line satisfied because the relevant terms do not exist,
but the fifth line not satisfied on the elements not in the domain of j + (fi ). We
prove that actually the j + (fi ) are total, in fact even more:
Claim 4.7. If j : Vλ ≺ Vλ and f : Vλ → Vλ , even partial, then j + (dom(f )) =
dom(j + (f )).
Proof of claim. By definition,
[
j + (dom(f )) = j(dom(f ) ∩ Vκm ),
m∈ω

while
[
dom(j + (f )) = dom( j(f ∩ Vκm )) =
m∈ω
[ [
= dom(j(f ∩ Vκm )) = j(dom(f ∩ Vκm )).
m∈ω m∈ω

It is not immediate to see why the two are the same, because it is possible
that dom(f ) ∩ Vκm and dom(f ∩ Vκm ) are different, as on the left we are cutting
the dominion, on the right both the dominion and codominion. The solution is
to divide everything in even smaller pieces, that would be the same on the left
and on the right side.
[
dom(f ) ∩ Vκm = dom(f ∩ Vκn ) ∩ Vκm ,
n∈ω

and [
dom(f ∩ Vκm ) = dom(f ∩ Vκm ) ∩ Vκn .
n∈ω

Therefore:

10
[ [ [
j + (dom(f )) = j(dom(f ) ∩ Vκm ) = j( dom(f ∩ Vκn ) ∩ Vκm ) =
m∈ω m∈ω n∈ω
[ [ [ [
= j(dom(f ∩ Vκn ) ∩ Vκm ) = j(dom(f ∩ Vκn ) ∩ Vκm ) =
m∈ω n∈ω n∈ω m∈ω
[
= j(dom(f ∩ Vκn )) = dom(j + (f )).
n∈ω

The third and fifth equivalence are a delicate point. In the third equivalence,
the ω-sequence hdom(f ∩ Vκn ) ∩ Vκm : n ∈ ωi ∈ Vλ , even if it is defined with κn ,
therefore j behaves properly. In the fifth equivalence, what seems an infinite
union is in fact finite.

Therefore every j : Vλ ≺ Vλ can be extended to a unique j + : Vλ+1 → Vλ+1


that is at least a ∆0 -elementary embedding. Moreover it is possible to prove
• I3 holds iff for some λ there exists a j : Vλ+1 → Vλ+1 that is a ∆0 -
elementary embedding;
• I1 holds iff for some λ there exists a j : Vλ+1 ≺ Vλ+1 that is a Σn -
elementary embedding for every n
Theorem 4.6 implies that if j, k : Vλ ≺ Vλ , then j + (k) : Vλ ≺ Vλ . This
operation between elementary embeddings is called application, and we write
j + (k) = j · k. Coupled with composition (not to be mistaken to!) they create
an interesting algebra.
Theorem 4.8 (Laver, 1992 [25]). Fix λ and let Eλ = {j : Vλ ≺ Vλ }. For all
j ∈ Eλ the closure of {j} in (Eλ , ·) is the free algebra generated by the law

(Left Distributive Law) i · (j · k) = (i · j) · (i · k).

Moreover Laver proved that the free algebra generated by the laws above
satisfies the word problem. His proof used extensively I3, but later Dehornoy
([6]) managed to prove the same thing in ZFC. A proof of this can be found in
the Handbook of Set Theory [16], Chapter 11.
There are still open problems at the I3 level:
Question 4.9. Are there j, k : Vλ ≺ Vλ such that the algebra generated by them
is free?
Another interesting result in I3 regards a function on the integers. Consider
j : Vλ ≺ Vλ with critical point κ and let Aj be the closure of j in (Ej , ·). Then
define
f (n) = |{crt(k) : k ∈ Aj , κn < crt(k) < κn+1 }|.
Then f (0) = f (1) = 0 and f (2) = 1, because the simplest element of Aj that
has a critical point not in the critical sequence of j is ((j · j) · j) · (j · j). However
f (3) is very large. Laver ([26]) proved that for any n, f (n) is finite, but f
dominates the Ackermann function, so f cannot be primitive recursive ([13]).

11
A standard way of using application is to consider j, j(j), j(j(j)). . . , and
we write j 0 = j and j n+1 = j(j n ). If hκn : n ∈ ωi is the critical sequence of
j, then by elementarity, crt(j(j)) = j(crt(j)) = κ1 and hκn : n ≥ 1, n ∈ ωi
is the critical sequence of j 2 . By induction, crt(j n ) = κn . This is the reason
why, when talking about I3 or I1, the critical point is never mentioned, when
usually it is the main object of research (see strongness, supercompactness. . . ):
if there exists a λ that witnesses I3, then there are infinite critical points tied to
the same λ, so it is really λ the relevant cardinal, while the critical points are
interchangeable and unbounded in λ. Still, they carry a lot of strength. The
following result shows that I3 really sits on top of the large cardinal hierarchy:
Proposition 4.10. Let hκi : i < ωi be the critical sequence of j. Then for
every i, κi is γ-supercompact for any γ < λ and n-huge for every n.
We use the characterization via ultrafilter:
Proposition 4.11 (Solovay, Reinhardt). A cardinal κ is γ-supercompact iff
there exists a fine, normal ultrafilter on Pκ (λ). It is n-huge iff there is a κ-
complete normal ultrafilter U over some P(η) and κ = η0 < η1 < · · · < ηn = η
such that for any i < n {x ∈ P(η) : ot(x ∩ ηi+1 ) = ηi } ∈ U .
Proof of 4.10. Let κ0 ≤ γ < λ, and suppose γ < κn ; then it is routine to check
that γ-supercompactness of κ0 is witnessed by

U = {X ⊆ Pκ0 (γ) : (j n )′′ γ ∈ j n (X)}

and that n-hugeness of κ0 is witnessed by

U = {X ⊆ P(κn ) : j ′′ κn ∈ j(X)} and κ0 , . . . , κn .

Then κn is γ-supercompact and n-huge by elementarity.


On the other hand λ (the “true” rank-into-rank cardinal) does not enjoy
many large cardinal properties, as it is of cofinality ω, and usually large cardinals
are regular. The most one can say is that it is a Rowbottom cardinal.
We end the section with some interesting reflection properties of Vλ :
Lemma 4.12. Suppose j : Vλ ≺ Vλ and let hκn : n ∈ ωi be its critical sequence.
Then for every n ∈ ω, Vκn ≺ Vλ .

Proof. Since j is the identity on Vκ0 , it is easy to see that Vκ0 ≺ Vκ1 . Considering
j(j), since crt(j(j)) = κ1 and

j(j)(κ1 ) = j(j)(j(κ0 )) = j(j(κ0 )) = κ2 ,

we have also that Vκ1 ≺ Vκ2 . We can generalize this to prove that for every
n ∈ ω, Vκn ≺ Vκn+1 . But then h(Vκn , idVκn ), n ∈ ωi forms a direct system,
whose direct limit is Vλ .
In particular, as κ0 is inaccessible, Vλ  ZFC.

12
5 L(Vλ+1) and I0
It would seem that one could not go any further than I1, as already j : Vλ+2 ≺
Vλ+2 is inconsistent. But there is a way to enlarge the domain of j without
stumbling in Kunen’s inconsistency:
Definition 5.1. Let A be a set. Then the class of sets constructible relative to
A is:
• L0 (A) = the transitive closure of A;
• Lα+1 (A) = Def(Lα (A));
S
• Lγ (A) = α<γ Lα (A);
S
• L(A) = α∈Ord Lα (A).
This is the smallest ZF-model that contains A. We can now introduce I0:
Definition 5.2 (Woodin). I0 There exists j : L(Vλ+1 ) ≺ L(Vλ+1 ) with critical
point less then λ.
As L(Vλ+1 ))  V = L(Vλ+1 ), it must be that j(Vλ+1 ) = Vλ+1 . Thanks to the
added assumption that the critical point must be less than λ, then, I0 is stronger
than I1, as j ↾ Vλ+1 : Vλ+1 ≺ Vλ+1 (otherwise it could have been the identity).
This implies that λ must be the supremum of the critical sequence. If one wants
to avoid the menace of Kunen’s inconsistency, it must be that hSα : α < crt(j)i
is not in L(Vλ+1 ). This is constructed via the Axiom of Choice on Vλ+1 . So,
for I0 to hold, it must be that L(Vλ+1 ) 2 AC, and in particular Vλ+1 must not
be well-orderable. This is the first affinity with ADL(R) , as in that case we have
L(R) 2 AC. More affinities will be introduced in section 7.
At a superficial glance I0 seems a Reinhardt cardinal, as it is an embedding
in a “universe” of ZF. But the situation is completely different: our ground
model is V , we assume that V  ZFC, and we construct L(Vλ+1 ) inside V .
So j is a definable class of V , and we will see that L(Vλ+1 ) does not satisfy
replacement for j, so such a j is not a witness for Reinhardt, as in that case the
theory of the ground model must be richer.
Before delving in I0, we analyze the structure of L(Vλ+1 ) itself, without
further assumptions, that still carry similarities with L(R).
As L(Vλ+1 ) is L relativized to Vλ+1 , we can relativize in the same way OD
and HOD:
Definition 5.3. ODVλ+1 , the class of the sets that are ordinal-definable over
Vλ+1 , is the Gödel closure of {Vα : α ∈ Ord} ∪ Vλ+1 .
HODVλ+1 , the class of the sets that are hereditarily ordinal-definable over
Vλ+1 , is the class of sets whose transitive closure is contained in ODVλ+1 .
Lemma 5.4. There exists a definable surjection Φ : Ord × Vλ+1 ։ L(Vλ+1 ),
therefore L(Vλ+1 )  V = HODVλ+1
Proof. This is immediate from the theory of relative constructibility.

13
The first application of this Lemma comes in form of partial Skolem func-
tions. Since L(Vλ+1 ) 2 AC, we possibly could not have Skolem functions.
But with the previous Lemma we can define for every formula ϕ(x, x1 , . . . , xn ),
a ∈ Vλ+1 , a1 , . . . , an ∈ L(Vλ+1 ):

hϕ,a (a1 , . . . , an ) = y where y is the minimum in (ODa )L(Vλ+1 )


such that L(Vλ+1 )  ϕ(y, a1 , . . . , an ).
These are partial Skolem functions, and the Skolem Hull of a set is its closure
under such Skolem functions.
We indicate with ։ the surjectivity of a function.
Definition 5.5.
L(V )
ΘVλ+1λ+1 = sup{γ : ∃f : Vλ+1 ։ γ, f ∈ L(Vλ+1 )}.
L(V )
In the following, we will call ΘVλ+1λ+1 = Θ, because it is a lighter notation
and there is no possibility of misinterpretation.
The role of Θ in L(Vλ+1 ) is exactly the same of its correspondent in L(R). In
the usual setting, under AC, to measure the largeness of a set we fix a bijection
from this set to a cardinal or, equivalently, the ordertype of a well-ordering of
the set. Since there is no Axiom of Choice in L(Vλ+1 ), it is not always possible
to define cardinality for sets that are not in Vλ+1 in the usual way, so to quantify
the “largeness” of a subset of Vλ+1 we will not use bijections, but surjections,
or, equivalently, not well-orders, but prewellorderings (pwo for short).
An order is a pwo if it satisfies antireflexivity, transitivity, and every subset
has a least element; in other words, it is a well-order without the antisymmetric
property. It is easy to see that the counterimage of a surjective function is a
pwo. One can think of a pwo as an order whose equivalence classes are well-
ordered, or a well-ordered partition. This creates a strong connection between
subsets of Vλ+1 and ordinals in Θ:
Lemma 5.6. 1. For every α < Θ, there exists in L(Vλ+1 ) a pwo in Vλ+1
with ordertype α, that is codeable as a subset of Vλ+1 ;
2. for every Z ⊆ Vλ+1 , Z ∈ L(Vλ+1 ) there exists α < Θ such that Z ∈
Lα (Vλ+1 ).
Proof. 1. Let ρ : Vλ+1 ։ α. Then
Rα = {(a, b) ∈ Vλ+1 × Vλ+1 : ρ(a) ≤ ρ(b)}
is a pwo in Vλ+1 . Moreover, Vλ+1 × Vλ+1 can be codified as a subset of
Vλ+1 , so also Rα can.
2. Let γ be such that Z ∈ Lγ (Vλ+1 ) and consider H Lγ (Vλ+1 ) (Vλ+1 , Z) the
Skolem Hull in Lγ (Vλ+1 ) of Vλ+1 and Z. Then, since H Lγ (Vλ+1 ) (Vλ+1 , Z) ∼
=
Lγ (Vλ+1 ), by condensation its collapse X = Lα (Vλ+1 ) for some α. But
H Lγ (Vλ+1 ) (Vλ+1 , Z) is the closure under the Skolem functions, and since
there is a surjection from Vλ+1 to the Skolem functions, this surjection
transfers to H Lγ (Vλ+1 ) (Vλ+1 , Z) and to Lα (Vλ+1 ), so α < Θ. Since Z and
all its elements are in H Lγ (Vλ+1 ) (Vλ+1 , Z), Z is not collapsed and then
Z ∈ Lα (Vλ+1 ).

14
We can also have a “local” version of Θ:
Definition 5.7.
L (V
α λ+1 )
ΘVλ+1 = sup{γ : ∃f : Vλ+1 ։ γ, f ∈ L(Vλ+1 ),
{(a, b) : f (a) < f (b)} ∈ Lα (Vλ+1 )}.

The reason why we do not just relativize Θ to Lα (Vλ+1 ) is because there can
be pwos in L(Vλ+1 ) that are not in Lα (Vλ+1 ), but can be coded in Lα (Vλ+1 ),
so in this way ΘLα (Vλ+1 ) is more “truthful” to indicate the ordinals that are
“hidden” inside Vλ+1 .
The notion of “stable” also neatly generalizes. Let δ be the minimum or-
dinal such that Lδ (Vλ+1 ) ≺1 LΘ (Vλ+1 ). Then Lδ (Vλ+1 ) is the collapse of the
closure of Vλ + 1 under the Skolem functions hϕ,a such that ϕ is Σ1 . The map
that associates to any (ϕ, a, x) the collapse of hϕ,a (x) is therefore a Σ1 partial
surjection from Vλ+1 to Lδ (Vλ+1 ). A study of the fine structure of Lδ (Vλ+1 )
has the following results:
Lemma 5.8. • For any β < δ, there exists a total surjection from Vλ+1 to
Lβ (Vλ+1 ) that is in Lδ (Vλ+1 ). Therefore ΘLδ (Vλ+1 ) = δ.
• δ is the supremum of the ∆21 pwos of Vλ+1 and (∆21 )L(Vλ+1 ) = Lδ (Vλ+1 ) ∩
Vλ+2 .
• Let ρ : Vλ+1 → Lδ (Vλ+1 ) be a partial Σ1 map. Then for any Z ⊆ dom(ρ),
Z ∈ Lδ (Vλ+1 ), ρ ↾ Z is bounded.
The proofs are the same as in [37].
Definition 5.9.

DCλ : ∀X ∀F : (X)<λ → P(X) \ ∅ ∃g : λ → X ∀γ < λ g(γ) ∈ F (g ↾ γ).

Note that this is a generalization of DC, since DC = DCω : we use directly


a function on < λ-sequences instead of considering a binary relation because
binary relations cannot handle the limit stages.
Lemma 5.10. In L(Vλ+1 ) the following hold:
1. Θ is regular ([39]);
2. DCλ .
Proof. 1. We fix a definable surjection Φ : Ord × Vλ+1 ։ L(Vλ+1 ). For every
ξ < Θ there is a surjection h : Vλ+1 ։ ξ. First of all, we choose one
surjection for each ξ: we define t : Θ \ ∅ → Ord, where for every ξ < Θ,
t(ξ) is the least γ such that there exists x ∈ Vλ+1 such that Φ(γ, x) is a
surjection from Vλ+1 to ξ. Then we define
(
Φ(t(ξ), x)(y) if Φ(t(ξ), x) is a map in ξ;
hξ (hx, yi) =
∅ else.

We have that hξ : Vλ+1 → ξ is well defined, because hx, yi is codeable in


Vλ+1 , and it is indeed a surjection: by definition there exists x ∈ Vλ+1

15
such that Φ(t(ξ), x) is a surjection from Vλ+1 to ξ, so for every β < ξ there
exists y ∈ Vλ+1 such that Φ(t(ξ), x)(y) = hξ (hx, yi) = β. Moreover hξ is
definable in L(Vλ+1 ).
Now, suppose that Θ is not regular, i.e., there exists π : α → Θ cofinal in
Θ with α < Θ. Then we claim that H(hx, yi) = hπ◦hα (x) (y) is a surjection
from Vλ+1 to Θ: let β < Θ; then there exists γ < α such that π(γ) > β and
there must exist x ∈ Vλ+1 such that hα (x) = γ; so π(hα (x)) > β, and there
exists y ∈ Vλ+1 such that H(hx, yi) = hπ◦hα (x) (y) = β. Contradiction.
2. We have to prove that

∀X ∀F : (X)<λ → P(X) \ ∅ ∃g : λ → X ∀γ < λ g(γ) ∈ F (g ↾ γ).

The proof is through several steps. First of all, DCλ (Vλ+1 ), that is DCλ
only for X = Vλ+1 , is quite obvious, because for every F as above by AC
there exists a g as above in V , but since g is a λ-sequence of elements in
Vλ+1 we have that g is codeable in Vλ+1 , so g ∈ L(Vλ+1 ).
Then we prove DCλ (α × Vλ+1 ) for every ordinal α. The idea is roughly
to divide F in two parts, and to define g using the minimum operator
for the ordinal part, and DCλ (Vλ+1 ) for the other part. For every s ∈
(α × Vλ+1 )<λ , we define m(s) as the minimum γ such that there exists
x ∈ Vλ+1 such that (γ, x) ∈ F (s). We call π2 : (α × Vλ+1 )<λ → (Vλ+1 )<λ
the projection. For every t ∈ (Vλ+1 )<λ , say t = hxξ : ξ < νi, we define
c(t) by induction as a sequence in (α × Vλ+1 )<λ such that π2 (c(t)) ⊆ t:
c(t) = h(γξ , xξ ) : ξ < ν̄i, where

γξ = min{γ : (γ, xξ ) ∈ F (c(t ↾ ξ))},

so that ν̄ is the smallest one such that there is no γ such that (γ, xν̄ ) ∈
F (c(t) ↾ ν̄). Let G : (Vλ+1 )<λ → P(Vλ+1 ) \ ∅ defined as

G(t) = {x ∈ Vλ+1 : (m(c(t)), x) ∈ F (c(t))},

then by DCλ (Vλ+1 ) there exists g : λ → Vλ+1 such that for every β < λ
g(β) ∈ G(g ↾ β). Now let f : λ → (α × Vλ+1 ) be defined by induction,
f (β) = (m(f ↾ β), g(β)). We want to prove that f (β) ∈ F (f ↾ β) for every
β < λ.
We prove by induction that f ↾ β = c(g ↾ β). Suppose that for every
ξ < β, f ↾ ξ = c(g ↾ ξ). By definition c(g ↾ β) = h(γξ , g(ξ)) : ξ < β̄i, with

γξ = min{γ : (γ, g(ξ)) ∈ F (c(g ↾ ξ))},

so (γξ , g(ξ)) ∈ F (c(g ↾ ξ)). But g(ξ) ∈ G(g ↾ ξ), so by definition of G,

(m(c(g ↾ ξ)), g(ξ)) ∈ F (c(g ↾ ξ)),

therefore γξ = m(c(g ↾ ξ)) and

f (ξ) = (m(f ↾ ξ), g(ξ)) = (m(c(g ↾ ξ)), g(ξ)) = (γξ , g(ξ)) = c(g ↾ β)(ξ).

So f ↾ β = c(g ↾ β) and, since for every ξ, (γξ , g(ξ)) ∈ F (c(g ↾ ξ)),


f (β) ∈ F (f ↾ β).

16
Finally, let X ∈ L(Vλ+1 ). Let α be such that Φ′′ (α × Vλ+1 ) ⊇ X, and let
F : (X)<λ → P(X) \ ∅. For every t = h(γξ , xξ ) : ξ < νi ∈ (α × Vλ+1 )<λ ,
we call c(t) = hΦ(γξ , xξ ) : ξ < ν̄i ∈ X <λ , where ν̄ is the minimum such
/ X. Then we define G : (α × Vλ+1 )<λ → P(α × Vλ+1 ) \ ∅,
that Φ(γν̄ , xν̄ ) ∈

G(t) = {(γ, x) : Φ(γ, x) ∈ F (c(t))},

and by DCλ (α×Vλ+1 ) we find g : λ → (α×Vλ+1 ) such that g(β) ∈ G(g ↾ β)


for every β < λ. Then f = Φ ◦ g is as we wanted, because for every β < λ,
Φ(g(β)) ∈ F (c(g ↾ β)), and as above we can prove that c(g ↾ β) = f ↾ β.
So we have DCλ (X) for every X ∈ L(Vλ+1 ), that is exactly DCλ .

Suppose now for the rest of the section that j : L(Vλ+1 ) ≺ L(Vλ+1 ), with
crt(j) < λ. Define

Uj = {X ⊆ Vλ+1 : X ∈ L(Vλ+1 ), j ↾ Vλ ∈ j(X)}.

Then Uj is a normal non-principal L(Vλ+1 )-ultrafilter in Vλ+1 , and we can


construct the ultraproduct U lt(L(Vλ+1 ), Uj ). Note that for every f, g : Vλ+1 →
L(Vλ+1 )

[f ] = [g] iff {x ∈ Vλ+1 : f (x) = g(x)} ∈ Uj


iff j ↾ Vλ ∈ j({x ∈ Vλ+1 : f (x) = g(x)}) =
= {x ∈ Vλ+1 : j(f )(x) = j(g)(x)}
iff j(f )(j ↾ Vλ ) = j(g)(j ↾ Vλ ),

and in the same way [f ] ∈ [g] iff j(f )(j ↾ Vλ ) ∈ j(g)(j ↾ Vλ ), so

U lt(L(Vλ+1 ), Uj ) ∼
= {j(f )(j ↾ Vλ ) : dom f = Vλ+1 }.

Let i : L(Vλ+1 ) → U lt(L(Vλ+1 ), Uj ) be the natural embedding of the ultraprod-


uct, then for every a ∈ L(Vλ+1 ), i(a) = [ca ] corresponds in the equivalence to
j(ca )(j ↾ Vλ ) = j(a), so we can suppose i = j. Is j an elementary embedding
from L(Vλ+1 ) to U lt(L(Vλ+1 ), Uj )? Since we do not have AC, the answer is not
immediate because we possibly do not have Los’ Theorem.
We will prove Los’ Theorem for this case, and this will imply that j is an
elementary embedding. It is clear that the only real obstacle is to prove that
for every formula ϕ and f1 , . . . , fn ∈ L(Vλ+1 ) such that dom fi = Vλ+1

U lt(L(Vλ+1 ), Uj )  ∃x ϕ([f1 ], . . . , [fn ])


iff {x ∈ Vλ+1 : L(Vλ+1 )  ∃y ϕ(f1 (x), . . . , fn (x))} ∈ Uj .

The direction from left to right is immediate: if [g] witness the left side, then
g(x) witness the right side. For the opposite direction, we need a sort of Uj -
choice, i.e., we need to find a function g such that

{x ∈ Vλ+1 : L(Vλ+1 )  ϕ(g(x), f1 (x), . . . , fn (x))} ∈ Uj .

We re-formulate this considering f : Vλ+1 → L(Vλ+1 ) \ ∅,

f (x) = {y ∈ L(Vλ+1 ) : L(Vλ+1 )  ϕ(y, f1 (x), . . . , fn (x))}.

17
Lemma 5.11 (Los’ Theorem for I0, Woodin, [39]). Let j : L(Vλ+1 ) ≺ L(Vλ+1 )
and Uj as above. Then for every F : Vλ+1 → L(Vλ+1 ) \ {∅} there exists H :
Vλ+1 → L(Vλ+1 ) \ {∅} such that {x ∈ Vλ+1 : H(x) ∈ F (x)} ∈ Uj .
Proof. First we consider the case ∀a ∈ Vλ , F (a) ⊆ Vλ+1 . We have to define H
such that j(H)(j ↾ Vλ ) ∈ j(F )(j ↾ Vλ ). Fix a b ∈ j(F )(j ↾ Vλ ), and define
(
c if k : Vλ ≺ Vλ , k(k) = j and k(c) = b
H(k) =
∅ otherwise

Note that

Kb := {k ∈ Vλ+1 | k : Vλ ≺ Vλ , k(k) = j, ∃c k(c) = b} ∈ Uj ,

because

j(Kb ) = {k ∈ Vλ+1 | k : Vλ ≺ Vλ , k(k) = j(j), ∃c k(c) = j(b)}

and j ↾ Vλ ∈ j(Kb ) (with c = b), so {x ∈ Vλ+1 : H(x) 6= ∅} ∈ Uj . Then


(
c if k : Vλ ≺ Vλ , k(k) = j(j) and k(c) = j(b)
j(H)(k) =
0 otherwise

so j(H)(j ↾ Vλ ) = b ∈ j(F )(j ↾ Vλ ).


For the more general case ∀a ∈ Vλ F (a) ⊆ L(Vλ+1 ) fix Φ : Ord × Vλ+1 ։
L(Vλ+1 ) definable and define

F̂ (a) = {x ∈ Vλ+1 : ∃γ Φ(γ, x) ∈ F (a)}.

Then there exists Ĥ such that {a ∈ Vλ+1 : Ĥ(a) ∈ F̂ (a)} ∈ Uj . Let γa =


min{γ : Φ(γ, Ĥ(a)) ∈ F (a)}. Therefore H(a) = Φ(γa , Ĥ(a)) is as desired.
Therefore, calling

Z = {j(f )(j ↾ Vλ ) : f ∈ L(Vλ+1 ), dom(F ) = Vλ+1 },

j : L(Vλ+1 ) → Z is an elementary embedding, and Z ∼ = L(Vλ+1 ). Let kU be


the inverse of the collapse of Z. We’ve seen in the proof of the previous Lemma
that for every b ∈ Vλ+1 there exists h such that j(h)(j ↾ Vλ ), so Vλ+1 ⊆ Z and
kU : L(Vλ+1 ) ≺ Z. Moreover, if R is a pwo in Vλ+1 , then R = {a ∈ Vλ+1 :
j(a) ∈ j(R)}, and since j(a), j(R) ∈ Z and Vλ+1 ⊆ Z we have that R is not
collapsed, so Θ ⊆ Z and crt(kU ) > Θ.
Theorem 5.12 (Woodin, [39]). For every j : L(Vλ+1 ) ≺ L(Vλ+1 ) there exist
a L(Vλ+1 )-ultrafilter U in Vλ+1 and jU , kU : L(Vλ+1 ) ≺ L(Vλ+1 ) such that jU
is the elementary embedding from U , j = jU ◦ kU and j ↾ LΘ (Vλ+1 ) = jU ↾
LΘ (Vλ+1 ).
Definition 5.13. Let j : L(Vλ+1 ) ≺ L(Vλ+1 ). We say that j is weakly proper
if j = jU .

18
Note that U ∈ / L(Vλ+1 ), otherwise we could run the naive Kunen’s Theorem
argument, so L(Vλ+1 ) does not satisfy replacement for j, and this proves that
I0 is not a “L(Vλ+1 )-Reinhardt”. In fact, one cannot even have j ↾ LΘ (Vλ+1 ) ∈
L(Vλ+1 ), otherwise one could construct U from that, so j is not amenable.
In a certain sense weakly proper embeddings are the “right” embeddings
for I0, and the most natural extension of I1, if one considers that LΘ (Vλ+1 ) is,
loosely speaking, the Vλ+2 of L(Vλ+1 ), i.e., the set of subsets of Vλ+1 . Every em-
bedding has therefore a core part, that comes from an ultraproduct, it is unique
and depends solely on its behaviour on LΘ (Vλ+1 ), and a spurious part (note
that kU is not an I0-embedding, as it has critical point > λ) that moves things
up above without really changing the important part of the embedding. For
example, if there are indiscernibles, it could be a simple shift of indiscernibles,
so nothing that carries real information.
In fact, the behaviour of an embedding that is weakly proper depends only
on its Vλ part:
Lemma 5.14 (Woodin, [39]). For every j1 , j2 : L(Vλ+1 ) ≺ L(Vλ+1 ), if j1 ↾
Vλ = j2 ↾ Vλ then j1 ↾ LΘ (Vλ+1 ) = j2 ↾ LΘ (Vλ+1 ).
Proof. We can suppose that j1 and j2 are weakly proper. By the usual analysis
of the ultraproduct, we have that every strong limit cardinal with cofinality
bigger than Θ is a fixed point for j1 and j2 , so I = {η : j1 (η) = j2 (η) = η} is
a proper class. Let M = H L(Vλ+1 ) (I ∪ Vλ+1 ). Since Vλ+1 ⊆ M k ∗ , the inverse
of the transitive collapse of M , has domain L(Vλ+1 ). If k ∗ is not the identity
then, as Θ ⊆ M by Lemma 5.6, crt(k ∗ ) > Θ. But in that case crt(k ∗ ) is a
strong limit cardinal with cofinality bigger than Θ, so crt(k ∗ ) ∈ I, and this is a
contradiction, because I ⊆ ran(k ∗ ) and crt(k ∗ ) ∈
/ ran(k ∗ ). So k ∗ is the identity
L(Vλ+1 )
and L(Vλ+1 ) = H (I ∪ Vλ+1 ).
Therefore every element of LΘ (Vλ+1 ) is definable with parameters in I∪Vλ+1 .
Let A ∈ L(Vλ+1 ) ∩ Vλ+2 , A = {x ∈ Vλ+1 : L(Vλ+1 )  ϕ(η, a)} with η ∈ I and
a ∈ Vλ+1 . Since j1 ↾ Vλ = j2 ↾ Vλ , we have that j1 (a) = j2 (a). Then

j1 (A) = {x ∈ Vλ+1 : L(Vλ+1 )  ϕ(η, j1 (a))} =


= {x ∈ Vλ+1 : L(Vλ+1 )  ϕ(η, j2 (a))} = j2 (A).

But every element of LΘ (Vλ+1 ) is definable from an ordinal α < Θ and


an element of Vλ+1 , α is definable from some pwo in L(Vλ+1 ) ∩ Vλ+2 , so j1 ↾
LΘ (Vλ+1 ) = j2 ↾ LΘ (Vλ+1 ).
Corollary 5.15 (Woodin, [39]). Let j : L(Vλ+1 ) ≺ L(Vλ+1 ) with crt(j). Then
C = {α < Θ : j(α) = α} is unbounded in Θ.
Proof. We can suppose that j is weakly proper. Then every ordinal β < Θ is
definable from some elements of I ∪ Vλ+1 , say Iβ = {i0 , . . . , in } ∈ I.
S Let α0 < Θ and consider the transitive collapse of the Skolem closure Z of
β<α0 Iβ ∪ Vλ+1 . Then by condensation it must be some Lγ (Vλ+1 ). As α0 < Θ,
there exists π : Vλ+1 ։ Z, therefore γ < Θ. As α0 ⊆ Z, we have α0 < γ. As
j(Z) = Z, we have j(γ) = γ. Therefore for any α0 < Θ there exists a γ ∈ C
such that α0 < γ < Θ.

19
6 Strong implications and inverse limits
When introducing new axioms, it is good practice to prove that they really
are adding something, i.e., that their consistency strength is strictly larger (or
smaller) than the consistency of the already known ones.
Definition 6.1. Let A(x) and B(x) two large cardinal properties. We say that
A(x) strongly implies B(x) iff for any κ, A(κ) implies B(κ) and the smallest κ
such that A(κ) holds is bigger than the smallest η such that B(η) holds.
This is even stronger then just the strict implications, cfr the note after
Lemma 6.22.
Proposition 6.2. If j : Vλ ≺ Vλ with crt(j) = κ, then there is a normal
ultrafilter U on κ such that {α < κ : α is n-huge for every n} ∈ U . Therefore
I3 strongly implies being n-huge for every n.
Proof. By 4.10 κ is n-huge for every n ∈ ω. Let U = {X ⊆ κ : κ ∈ j(X)} and
let
X0 = {α < κ : α is n-huge for every n}.
Then
j(X0 ) = {α < j(κ) : α is n-huge for every n},
and κ ∈ j(X0 ), so X0 ∈ U .
Note that “ω-huge”, i.e, j : V ≺ M with M λ ⊆ M 4 , is inconsistent by
Kunen’s original proof of Kunen’s Theorem, so this is the best we can do.
Definition 6.3. Let j : M0 ≺ M1 . Then we say that (Mα , j α ) is the α-th
iterate if:
• j 0 = j;
• j α+1 = j α (j α );
• Mα+1 = j α (Mα ) is well-founded;
• jγ,α+1 = j α ◦ j α−1 ◦ · · · ◦ j γ . So jα,α+1 = j α ;
• if α is limit, then (Mα , j α , jn,α ) is the direct limit of the system (Mβ , jβ,γ )
with β, γ < α, if it is well-founded.
We say that j is α-iterable if (Mα , j α ) exists. We say that j is iterable if it
is α-iterable for any α ordinal.
4 This is not a standard definition. For different authors, “ω-huge” can mean different

things.

20
j0,α

jn,α
j2,n
j0 j1 jω
M0 M1 M2 ··· Mn ··· Mω Mω+1 ··· Mα

j0,2
j1,ω
jω+1,α
j0,ω

It is possible for the iterate to not exists, whether because j α (j α ) or j α (Mα )


are not defined, or because the model Mα is not well-founded. The direct limit
of the system (Mβ , jβ,γ ) is defined as an equivalence relation on the set of (β, a)
such that a ∈ Mβ : if β < γ then (β, a) is equivalent to (γ, b) iff jβ,γ (a) = b, and
[(β, a)] is in [(γ, b)] iff jβ,γ (a) ∈ b. If γ < β, viceversa. Then jβ,α (a) is defined
as [(β, a)] and j α = j0,α (j). Ambiguously, we call Mα both the domain of the
model defined via the equivalence class and the ∈-model equivalent to it. So if
M1 ⊆ M0 , then for all α such that Mα is defined we have Mα ⊆ M0 , but it is
possible that M0 ⊆ Mα if seen as ∈-models. It will be clear from context which
of the two will be used. Note that if a ∈ Mα , with α limit, then it must be in
the range of some jβ,α , with β < α.
So let j : Vλ ≺ Vλ . We have already defined j n , but

j n (j n ) = j(j n−1 )(j(j n−1 )) = j(j n−1 (j n−1 )) = j(j n ) = j n+1 ,

by left distributive law and induction, therefore the j n ’s are iterates of j. More-
over, j(Vλ ) = Vλ , so the first ω-iterates of j are (Vλ , j n ) and j is finitely iterable.
Could we push it any further? As all the Mα ⊆ Vλ , technically there is no prob-
lem in defining it, the only decisive factor being whether Mα is well-founded.
Proposition 6.4. “There exists j : Vλ ≺ Vλ ω-iterable” strongly implies I3(λ).
Proof. Suppose j : Vλ ≺ Vλ is ω-iterable, let hκn : n ∈ ωi be its critical sequence.
Then Mω is the set of equivalence classes of (n, a) such that a ∈ Vλ , where if
n < m then (n, a) is equivalent to (m, b) iff jn,m (a) = b. Also, jn,ω (a) is the
class of [(n, a)].
We prove by induction that jn,ω (α) = α for α ∈ κn . Note that if β < κn ,
then for every m > n, jn,m (β) = β, therefore [(n, β)] = [(m, β)]. So for any
m ∈ ω, β < λ, [(m, β)] < [(n, α)] iff [(n, β)] < [(n, α)]: If m ≤ n, then jm,n (β) <
α < κn = jm,n (κm ) therefore by elementarity β < κm and [(m, β)] = [(n, β)].
If m > n, then β < jn,m (α) = α, therefore also [(n, β)] < [(n, α)]. So jn,ω (α) =
[(n, α)] = {[(n, β)] : β < α}, that is by induction α.
We want to calculate now j0,ω (κ0 ). If α < j0,ω (κn ), then there exists n ∈
ω such that α = jn,ω (β) for some β < λ. But then jn,ω (β) < j0,ω (κ0 ) =

21
jn,ω (j0,n (κ0 )) = κn , therefore α = jn,ω (β) = β < λ. On the other hand, if α < λ
then there exists n ∈ ω such that α < κn , so jn,ω (α) = α < jn,ω (κn ) = j0,ω (κ0 ).
Therefore j0,ω (κ0 ) = λ.
If a ∈ Vλ , then there exists n ∈ ω such that a ∈ Vκn . So jn,ω (a) = a ∈ Mω .
Therefore Vλ ⊆ Mω . Let

T = {he0 , . . . , en i : ∃hαi : i ≤ n + 1i, ∀i ≤ n + 1 αi < λ,


∀i ≤ n∃ei : Vαi ≺ Vαi+1 , ∀i < n ei+1 ↾ Vαi = ei },

the tree of approximations of an I3 elementary embedding. Note that T = T Mω .


As j is a branch of T , T is ill-founded, and by absoluteness of well-foundedness
it is ill-founded also in Mω , with the difference that a branch cannot go up to
λ, as λ is regular in Mω by elementarity. So Mω  ∃γ < λ ∃k : Vγ ≺ Vγ . So by
elementarity ∃γ < κ0 ∃k : Vγ ≺ Vγ .

Corollary 6.5. Let α be a countable ordinal. Then “There exists j : Vλ ≺ Vλ


ω · (α + 1)-iterable” strongly implies “There exists j : Vλ ≺ Vλ ω · α-iterable”.
Proof. By induction, Vλ ⊆ Mω·α ⊆ Mω·(α+1) . We build again a tree of approxi-
mations,
S but this time we want to add rank functions, so that the ω ·α-th iterate
of n∈ω en will be well-founded. So a branch of T is (he0 , . . . , en i, hf0 , . . . , fn i),
where fi is a rank function on {(β, a) : β < ω · α, a ∈ Vαi } that is coherent
with the equivalence on Mω·α . A branch of T will give an embedding that is
ω · α-iterable, and the proof is as before.
Proposition 6.6. If j : Vλ ≺ Vλ is ω1 -iterable, then it is iterable.
Proof. This is a standard result in the theory of the iterations of elementary
embeddings. Suppose that j is not iterable, and let ν be the least such that Mν
is ill-founded. Let N be a countable substructure of Vλ , let h : N ≺ Vλ indicate
the embedding resulting form the collapse. Let h(M̄0 ) = Vλ , h(̄) = j, and build
the iterate sequence in N . Then h(M̄β ) = Mh(β) and let ν̄ = h−1 (ν). In partic-
ular ν̄ is countable. For any β ≤ ν define h′β so that the diagram commutes:

jα,β jβ,ν̄
Vλ ··· Mα Mβ Mν̄ ··· Mν

h h′0 h′α h′β h′ν̄


h
̄α,β ̄β,ν̄
M̄0 ··· M̄α M̄β M̄ν̄

Let h′0 = h ↾ M̄0 . If β is a limit ordinal, then every x ∈ M̄β is some j̄α,β (y),
with α < β and y ∈ M̄α . Then h′β (x) = jα,β (h′α (y)). If β = γ + 1, then by
elementarity, as j : Vλ ≺ Vλ , M̄β = M̄γ and Mβ = Mγ , so h′β = h′γ .
But then h′ν̄ : M̄ν̄ ≺ Mν̄ . As Mν is illfounded, then by elementarity also Mν̄
is illfounded, and since ν was the least and ν̄ is countable, ν = ν̄ is countable
and j is not ω1 -iterable.
While the “ω-hugeness” is inconsistent, it is not immediately clear what
happens with “ω-superstrongness”:

22
Definition 6.7. I2 There exists j : V ≺ M , with M ⊆ V , such that Vλ ⊆ M
for some λ = j(λ) > crt(j).
Proposition 6.8. Suppose j witnesses I2. Then λ is the supremum of its
critical sequence, and j ↾ Vλ : Vλ ≺ Vλ .
Proof. Let η be the supremum of the critical sequence of j. Then it must be
η ≤ λ, as η is the first fixed point of j above crt(j). If η + 2 ≤ λ then there is a
contradiction with Kunen’s Theorem, so we only have to check that η + 1 6= λ,
i.e., not all subsets of Vη are in M .
Suppose that j ′′ η ∈ M , then define U = {X ⊆ P(η) : j ′′ η ∈ j(X)} and let
i : V ≺ N be the ultrapower via U . Then i(η) = η, and
[
i′′ P(η) = { i(X ∩ α) : X ∈ P(η)} ∈ N.
α<η

Now, coding λ+ as sets in P(λ), i′′ λ+ ∈ N , and Woodin’s proof of Kunen’s


theorem goes smoothly (because C is definable from i′′ λ+ ). Contradiction.
So j ′′ η ∈
/ M , and η must be λ. In particular j ↾ Vλ : Vλ ≺ Vλ .
The previous Proposition says that I2 implies I3. We can do more:
Proposition 6.9. If j : V ≺ M witnesses I2(λ) then j ↾ Vλ is iterable.
Proof. Consider an iteration of j : V ≺ M with Vλ ⊆ M . Suppose that (Vλ )Mν
is ill-founded, with ν the least. Let α0 be the minimum such that j0,ν (α0 ) is
ill-founded. In other words, Vλ  “α0 is the least α such that ∃µ such that
j0,µ (α) is ill-founded”. Let β < j0,ν (α0 ) ill-founded. Let η < ν and α1 such
that jη,ν (α1 ) = β. Consider j0,η (α0 ). As jη,ν (α1 ) < j0,ν (α0 ), we must have
α1 < j0,η (α0 ). But by elementarity

(Vλ )Mη  “j0,η (α0 ) is the least α such that


∃µ such that [j0,η (j)]0,µ (α) is ill-founded′′ .

Note that [j0,η (j)]0,µ = [(jη,η+1 )]0,µ = jη,η+µ . But then α1 is such that,
contradiction because j0,η (α0 ) should have been the minimum.
Another way to extend I3 comes from the way we characterized I3 and I1,
respectively as Σ0 - and Σω -elementary embeddings from Vλ+1 to itself. We can
consider all the intermediate steps:
Definition 6.10 (Σ1n Elementary Embedding). Let j : Vλ ≺ Vλ . Then j is Σ1n
iff j + is a Σn -elementary embedding, i.e., iff for every Σ1n -formula ϕ(X), for
every A ⊆ Vλ ,
Vλ  ϕ(A) ↔ ϕ(j + (A)).
It is not immediate, however, to see if these hypothesis are really different,
and what is their relationship with I2. In fact, they are not always different:
the following theorems will prove that if n is odd, then j is Σ1n iff it is Σ1n+1 .
During the proof it will be clear that I2 is the same as j is Σ11 , therefore proving
that I1 implies I2.
To prove this we need what is often called the ‘descriptive set theory’ on
Vλ . The fact that λ has cofinality ω and is a strong limit, makes possible to

23
develop a description of the closed subsets of Vλ as projection of trees similar
to the classic one in descriptive set theory. The fact that both Σ11 and Σ12 sets
can be represented with trees will give the first equivalence, in a way similar to
Schoenfield Absoluteness Lemma. Fix λ and a critical sequence hκ0 , κ1 , . . . i.
Let ϕ(X, Y ) a Σ10 -formula, namely

∀x0 ∃y0 ∀x1 ∃y1 · · · ·n ψ(X, Y, x0 , y0 , . . . ).

We define the tree Tϕ(X,Y ) . The m-th level of Tϕ(X,Y ) is the set of (a, b, F, P )
such that:
• a, b ⊆ Vκm ;
• F is a partial function : (Vκm )≤n+1 → Vκm such that for all d0 , . . . , dn
where F is defined then

ψ(a, b, d0 , F (d0 ), d1 , F (d0 , d1 ), . . . );

• P : ((Vκm )≤n+1 \ dom(F )) → (ω \ (m + 1)).


We say that (a, b, F, P ) < (a′ , b′ , F ′ , P ′ ), with the first term in the m-th level
and the second in the m′ -th level, when a ⊆ a′ , b ⊆ b′ , a′ ∩Vκm = a, b′ ∩Vκm = b,
~ < m then d~ ∈ dom(F ), otherwise P ′ (d)
F ⊆ F ′ , and if P (d) ~ = P (d).
~
Informally, a and b are just the attempts to construct X and Y , F is an
approximation of the Skolem function that would witness the first order part of
ϕ, and P is booking the level in which the elements of Vκm that are not yet in
the dominion of F will be in its extension.
It is clear that if Tϕ(X,Y ) has an infinite branch, the union of the a’s and b’s
will give suitable X and Y , and the F ’s will construct a Skolem function, that
is total thanks to the P ’s.
Lemma 6.11. Let ϕ(X, Y ) a Σ10 -formula and B ⊆ Vλ . Then Vλ  ∃X ϕ(X, B)
iff Tϕ(X,B) has an infinite branch.
The Σ12 case is a bit more complex. Fix a B ⊆ Vλ . For every a ⊆ Vκm let

Gm (a) = {(c, F, P ) : (a, c, F, P ) ∈ m-th level of T¬ϕ(X,B,Y ) }.

Then the m-th level of Tϕ(X,B) is

{(a, H) : a ⊆ Vκm , H : Gm (a) → λ+ }.

We say that (a, H) < (a′ , H ′ ) when a′ ∩ Vκm and if (a, c, F, P ) < (a′ , c′ , F ′ , P ′ ),
then H(c, F, P ) > H ′ (c′ , F ′ , P ′ ).
Lemma 6.12. Let ϕ(X, Y ) a Σ10 -formula and B ⊆ Vλ . Then Vλ  ∃X∀Y ϕ(X, B)
iff Tϕ(X,B) has an infinite branch.
If Tϕ(X,B) has an infinite branch, with A the union of the a’s in the branch,
then the H’s assure that there are no possible infinite branches in T¬ϕ(A,B,Y )
for every Y ⊂ Vλ , because otherwise it would be possible to build a descending
chain in λ+ .
Lemma 6.13. Let j : Vλ ≺ Vλ . Then j is Σ11 iff j + preserves the well-founded
relations.

24
Proof. ‘Being well-founded’ is a ∆11 relation, so if j is Σ11 it preserves well-
founded relations. Vice versa, let ϕ(X, Y ) a Σ10 formula, B ⊆ Vλ and suppose
that Vλ  ∃Xϕ(X, B). Fix an X ⊆ Vλ such that Vλ  ϕ(X, B). Therefore
by Theorem 4.6 Vλ  ϕ(j + (X), j + (B)) and Vλ  ∃X ϕ(X, j + (B)). If Vλ 
∀Xϕ(X, B), then T¬ϕ(X,B) is without infinite branches. Fix a well-ordering
R of Vλ , and define the relative Kleene-Brouwer order. Therefore the Kleene-
Brouwer order of T¬ϕ(X,B) is well-founded. But then the Kleene-Brouwer order
relative to j + (R) on T¬ϕ(X,j + (B)) is well-founded. So Vλ  ∀Xϕ(X, j + (B)).
Corollary 6.14. If j : V ≺ M witnesses I2(λ), then j ↾ Vλ is Σ11 .
Proof. If R is well-founded, then (j ↾ Vλ )+ (R) = j(R) is well founded in M .
But by absoluteness of well-foundedness, j(R) is well-founded in V .
Definition 6.15 (λ-Ultrafilters Tower). Fix λ and hκn : n ∈ ωi a cofinal se-
quence of regular cardinals in λ. Then U~ = hUn : n ∈ ωi is a λ-ultrafilters tower
iff for every n ∈ ω, Un is a normal, fine, κ-complete ultrafilter on [κn+1 ]κn , and
if m < n then Un projects to Um ; i.e., for every A ∈ Um

{x ∈ [κn+1 ]κn : x ∩ κm+1 ∈ A} ∈ Un .

A λ-ultrafilters tower is complete if for every sequence hAi ∈ i ∈ ωi with


Ai ∈ Ui there exists X ⊆ λ such that for every i ∈ ω, X ∩ κi+1 ∈ Ai .

Lemma 6.16 (Gaifman ([15]), Powell ([34]), 1974). If U ~ is a complete λ-tower,


then the direct limit of the ultrapowers Ult(V, Un ) is well-founded.
~j = hUn : n ∈ ωi
When j : Vλ ≺ Vλ , it is natural to consider the sequence U
defined as
Un = {A ⊆ [κn+1 ]κn : j”κn+1 ∈ j(A)}.
~j is a complete λ-tower.
Lemma 6.17. If j is Σ11 , then U
Proof. Let hAi : i ∈ ωi such that Ai ∈ Ui . Let X = j”λ. Then for every i ∈ ω,
X ∩ κi+2 = j”κi+1 ∈ j(Ai ), so

∃X ⊂ λ∀i ∈ ω X ∩ κi+2 ∈ j(Ai ).

This is a Σ11 statement, so by Σ11 -elementarity ∃X ⊂ λ∀i ∈ ω X ∩ κi+1 ∈ Ai .


Definition 6.18. Let M be a transitive class, with Vλ ⊂ M . Then M is Σ1n -
correct in λ if for every Σ1n -formula ϕ(X), for every A ⊆ Vλ and A ∈ M ,
(Vλ  ϕ(A))M iff Vλ  ϕ(A).
Theorem 6.19 (Martin ([29])). Let j : Vλ ≺ Vλ . If j is Σ11 then
~j ), so we have I2;
• Vλ ⊆ Ult(V, U
~j ) is Σ1 -correct, therefore j is Σ1 .
• Ult(V, U 2 2

~j ) of the ultrapowers Ult(V, Un ). By


Proof. Consider the direct limit Ult(V, U
~
Lemma 6.16 and Lemma 6.17 Ult(V, Uj ) is well founded, and it is possible to
collapse it on a model M .

25
j0 i0
V M0 ∼
= Ult(V, U0 ) M∼ ~j )
= Ult(V, U

j1 j01
i1
M1 ∼
= Ult(V, U1 )

j12
jn

in
..
.

Mn ∼
= Ult(V, Un )

..
.

By the usual theory of normal ultrafilters, it is possible to prove that the


above diagram commute, that crt(jn ) = κ, crt(jn,n+1 ) = κn , so Vκn ⊂ Mn ,
crt(in ) = κn and therefore Vλ ⊂ M . Let i = in ◦ jn for every n. Then i ↾ Vλ = j,
i(λ) = λ and i((λ+ )V ) = (λ+ )V .
Observe that for every B ⊆ Vλ , B ∈ M we have that (Tϕ(B) )M = Tϕ(B) , so
M is Σ11 -correct in λ. Unfortunately, the same cannot be said to the Σ12 case,
since it is possible that (Tϕ(X,B,Y ) )M 6= Tϕ(X,B,Y ) . The reason for this is that
while it is true that (Gm (a))M = Gm (a) for every m ∈ ω and a ∈ Vκm , it is
possible that there exists H : Gm (a) → λ+ with H ∈ / M.
Claim 6.20. If c ∈ Vλ and F : c → Ord, then i ◦ F ∈ M .
Proof. Let c ∈ Vα , with α < λ, and pick n ∈ ω such that crt(in ) > α. Then
i ◦ F = in ◦ jn ◦ F = in (jn ◦ F ) ◦ in ↾ c = in (jn ◦ F ) ∈ M .
By Σ11 correctness in λ,

(Vλ  ∃X∀Y ϕ(X, B, Y ))M → Vλ  ∃X∀Y ϕ(X, B, Y ),

so it suffices to prove the other direction.


Suppose that (Vλ  ∀X∃Y ¬ϕ(X, B, Y ))M . Then (Tϕ(X,B,Y ) )M is well-
founded. Let L : (Tϕ(X,B,Y ) )M → Ord that witnesses it. Define L̃ : Tϕ(X,B,Y ) →
Ord as L̃((a, H)) = L((a, i ◦ H)). By the previous claim i ◦ H ∈ M , and more-
over i ◦ H is a function from Gm (a) to i(λ+ ) = λ+ , so (a, i ◦ H) ∈ (Tϕ(X,B,Y ) )M
and L̃ is well-defined. It remains to prove that L̃ witnesses that Tϕ(X,B,Y ) is
well-founded. Suppose that (a, H) < (a′ , H ′ ). If in Tϕ(X,B,Y ) we have that
(a, c, F, P ) < (a′ , c′ , F ′ , P ′ ) then H(c, F, P ) > H ′ (c′ , F ′ , P ′ ), so

i ◦ H(c, F, P ) > i ◦ H ′ (c′ , F ′ , P ′ ),

therefore (a, i ◦ H) < (a′ , i ◦ H ′ ). It follows that

L((a, i ◦ H)) > L((a′ , i ◦ H ′ )),

26
so L̃((a, H)) > L̃((a′ , H ′ ).
Then Tϕ(X,B,Y ) cannot have an infinite branch and Vλ  ∀X∃Y ¬ϕ(X, B, Y ),
i.e., M is Σ12 -correct in λ.
This proves that i ↾ Vλ is Σ12 : if ϕ is Σ12 in Vλ and V  ϕ(B), then by
elementarity Ult(V, U ~j )  ϕ(i(B)), therefore by Σ12 -correctness also V  ϕ(i(B)).
Since i ↾ Vλ = j we’re done.
This theorem proves that Σ11 and Σ12 elementary embeddings in Vλ are the
same, and they are equivalent to I2(λ). It is possible, after a pair of technical
lemmas, to generalize this for Σ1n and Σ1n+1 .
Lemma 6.21 (Laver, [27]). Let n be odd. Then “j is Σ1n ” is a Π1n+1 formula
in Vλ , with j as a parameter.
Proof. By definition j is Σ1n iff

∀B ⊆ Vλ ∀Σ10 formula ϕ(X1 , . . . , Xn , Y )


∃A1 ∀A2 . . . ∃An Vλ  ϕ(A,~ j + (B)) → ∃A1 ∀A2 . . . ∃An Vλ  ϕ(A,
~ B).

But for every D ⊂ Vλ , Vλ  ϕ(A, ~ D) iff there exists a branch in T ~ whose


ϕ(A)
projection is D. Using this is easy to check the Lemma.
Lemma 6.22 (Square root Lemma for Σ1n , Laver, [27]). Let n be odd, n > 1.
Let j be Σ1n , crt(j) = κ. Let β < κ, A, B ⊆ Vλ . Then there exists a k : Vλ ≺ Vλ
that is Σ1n−1 such that k(k) = j, k(B) = j(B), k(A′ ) = A for some A′ ⊆ Vλ
and β < crt(k) < κ.
Proof. The formula “∃k, Y, k is Σ1n−2 , k(k) = j, k(B) = j(B), k(Y ) = A, β <
crt(k) < κ” is Σ1n by Lemma 6.21. The following formula is clearly true:

j is Σ1n−2 , j(j) = j(j), j(j(B)) = j(j(B)),


j(A) = j(A), j(β) = β < crt(j) < j(κ)
but then, with a smart quantification of some of the parameters of the formula
above, we have

∃k, Y k is Σ1n−2 , k(k) = j(j), k(j(B)) = j(j(B)),


k(Y ) = j(A), j(β) < crt(k) < j(κ)
By elementarity the lemma is proved.
The lemma therefore provides a “square root” for j, with certain desirable
characteristics, but with the catch that for a square root of a Σ1n -elementary
embedding we can only assure that it is Σ1n−2 . It is not possible to do better:
suppose that κ is the minimum critical point of j : Vλ ≺ Vλ that is Σ1n . Then
the critical point of the square root k of j is less then κ, therefore k cannot be
Σ1n . Also note that the square root of an iterable embedding is iterable, as a
square root of j is, in a certain sense, j −1 .
Note that this would be enough to prove strong implication if we considered
the existence of the embeddings as a property of the critical point, and we would
have also that I2(κ) strongly implies iterable I3(κ). But we already argued that
this seems not the right way to look at this properties. Moreover, it is possible
to prove the strong implications of such axioms respect to λ:

27
Corollary 6.23. I2 strongly implies iterable I3.
Proof. Note that “j is ω1 -iterable”, and therefore “j is iterable”, is Σ11 (there
exist α < ω1 , M0 , . . . , Mα ), j0 , . . . , jα such that. . . ). Therefore one can put “j
is iterable” in 6.22 and the proof works.
Theorem 6.24 (Laver, [27]). Let n be odd. Then if j is Σ1n , it is also Σ1n+1 .
Proof. Theorem 6.19 is the case n = 1. We proceed by induction on n.
Let B ⊆ Vλ . We have to prove that for every ψ Σn+1 formula and B ⊆ Vλ+1 ,

Vλ  ψ(B) iff Vλ  ψ(j + (B)).

By induction, the direction from left to right is immediate, so it suffices to show


the other direction. Suppose

Vλ  ∃X1 ∀X2 ∃X3 . . . ∀Xn+1 ϕ(X1 , . . . , Xn+1 , j + (B)).

Let X1 = A a witness. By Lemma 6.22, we can pick a k that is Σ1n−2 such that
k(B) = j(B) and there exists A′ ⊆ Vλ+1 such that k(A′ ) = A. By induction, k
is also Σ1n−1 , and since

Vλ  ∀X2 ∃X3 . . . ∀Xn+1 ϕ(k(A′ ), X2 , . . . , Xn+1 , k(B)),

then
Vλ  ∀X2 ∃X3 . . . ∀Xn+1 ϕ(A′ , X2 , . . . , Xn+1 , B).
Therefore A′ is a witness for ψ(B).
Theorem 6.24 shows a peculiar asimmetry. What problems arise when n is
even? The key is in the proof of Lemma 6.21. When the number of the quanti-
fiers of a Σn formula is odd, then the last one is an existential quantifier. Since
by Lemma 6.12 the satisfaction relation for Σ11 formulae is Σ11 , this quantifier is
absorbed by the satisfaction formula. When n is even, however, the last quan-
tifier is a universal one, and “being Σ1n “ is still Π1n+2 . This seems an ineffectual
detail, but in fact Laver proved that it is an essential one, since he showed that
being a Σ1n+1 -embedding, for n even, is strictly stronger than being a Σ1n one.
Theorem 6.25 (Laver, [27]). Let h and k be Σ1n embeddings. Then h · k is a
Σ1n -embedding.
Proof. By Theorem 6.24 we can suppose n odd. Let ϕ be a Σ10 formula with
n + 1 variables. Then

Vλ  ∀Y ∃X1 ∀X2 . . . ∃Xn ϕ(X1 , . . . , Xn , k(Y ))



→ ∃X1 ∀X2 . . . ∃Xn ϕ(X1 , . . . , Xn , Y ) .

This formula is Π1n+1 . Since by Theorem 6.24 h is Σ1n+1 , we have

Vλ  ∀Y ∃X1 ∀X2 . . . ∃Xn ϕ(X1 , . . . , Xn , h(k)(Y ))



→ ∃X1 ∀X2 . . . ∃Xn ϕ(X1 , . . . , Xn , Y ) .

so h(k) is Σ1n .

28
Note that the converse is not true: minimizing the critical point, we can find
a k that is Σ1n such that any h so that h(h) = k cannot be Σ1n (see comment
after Lemma 6.22).
However, if we switch application with composition, then also the converse
is true.
Lemma 6.26. If h and k are Σ1m and h ◦ k is Σ1m+1 , then k is Σ1m+1 .
Proof. Suppose that

Vλ  ∀X1 ∃X2 . . . ∀Xm+1 ϕ(X1 , . . . , Xm+1 , B)

with B ⊆ Vλ . Then

Vλ  ∀X1 ∃X2 . . . ∀Xm+1 ϕ(X1 , . . . , Xm+1 , h ◦ k(B)).

Let A ⊆ Vλ . Then in particular

Vλ  ∃X2 . . . ∀Xm+1 ϕ(h(A), X2 , . . . , Xm+1 , h ◦ k(B)).

By elementarity

Vλ  ∃X2 . . . ∀Xm+1 ϕ(A, X2 , . . . , Xm+1 , k(B)).

Since this is true for every A ⊆ Vλ , we have

Vλ  ∀X1 ∃X2 . . . ∀Xm+1 ϕ(X1 , . . . , Xm+1 , k(B)).

Theorem 6.27 (Laver, [27]). Let h, k ∈ Eλ . Then h, k are Σ1n iff h ◦ k is Σ1n .

Proof. If h and k are Σ1n , then obviously h ◦ k is Σ1n . We prove by induction on


m ≤ n that h and k are Σ1m .
The case m = 0 is by hypothesis. Suppose it is true for m. Then by Theorem
6.25 h(k) is Σ1m . It is easy to calculate that h ◦ k = h(k) ◦ h, and this is Σ1m+1
by hypothesis. By using Lemma 6.26 in the left side, we have that k is Σ1m+1 ,
and using it on the right side we have that h is Σ1m+1 .
Theorem 6.27 is promising for our objective, that is proving that being Σ1n+2
is strictly stronger than being Σ1n for an elementary embedding. The most
natural idea for doing this is using some sort of reflection, to prove that if there
is a j ∈ Eλ Σ1n+2 , then there is a k ∈ Eλ′ that is Σ1n for some λ′ < λ. It
would be tempting to use, as in the case of iterability, direct limit of elementary
embeddings, but unfortunately in this setting there are counterexamples:
Theorem 6.28 (Laver, [26]). There exists a j that is Σ1n that has a stabilizing
direct limit of members of Aj that is not Σ11 ,

So we will consider inverse limits instead.


Let hj0 , j1 , . . . i be a sequence of elements of Eλ , and let J = j0 ◦ j1 ◦ . . . be
the inverse limit of the sequence. By definition the dominion of J is {x ∈ Vλ :
∃nx ∀i > nx ji (x) = x}. But we know that ji (x) = x iff x ∈ Vcrt(ji ) , so this is
{x ∈ Vλ : ∃nx ∀i > nx x ∈ Vcrt(ji ) }. That is, x ∈ dom J depends only on the

29
rank of x, and this implies that dom J = Vα for some α. It is also possible to
calculate α, since β < α iff ∃n ∀m ≥ n β < crt(jm ):

α = sup inf crt(jm ) = lim inf crt(jn ).


n≥0 m≥n n∈ω

With some cosmetic change, we can also suppose that α as the supremum
of the critical points, not only the limit inferior. This will also simplify the
following proofs and notations.
So let λn = inf m≥n crt(jm ). Then α = supn∈ω λn , and λn is increasing in
n. If the supremum is also a maximum, we incur in the trivial case, where J is
in fact just a finite composition of elementary embeddings: if n is the first one
such that λn = α, then all crt(jm ) with m > n are bigger than α, so they are
constant in the domain of J and they don’t change anything.
Suppose then that α is a proper supremum of the λn sequence. We can
suppose crt(jn ) = λn by aggregating multiple elementary embeddings in just
one: consider the largest n such that crt(jn ) = λ0 (there will be a largest one
because α is a proper supremum of the λn sequence), define the new k0 as the
old j0 ◦· · ·◦jn , and repeat this for every λn . The following is a graphical example:

k0 k1 k2 k3

The columns represent the behaviours of each jn on λ, where the column on the
left represent j0 , and the horizontal lines indicate the critical point.
Definition 6.29. Let J = j0 ◦ j1 ◦ . . . . Then we define Jn = jn ◦ jn+1 ◦ . . . and
J0(n−1) = j0 ◦ j1 ◦ · · · ◦ jn−1 .
Lemma 6.30. Let jm ∈ Eλ for every m ∈ ω, define αm = crt(jm ) and suppose
that for every m ∈ ω, αm < αm+1 . Let α = supm∈ω αm and J = j0 ◦ j1 ◦ . . . .
Then
• J ′′ α is unbounded in λ;
• J : Vα ≺ Vλ is elementary.
Proof. • Let δm = sup Jm “α. We prove that when δm < λ, then δm 6= δm+1 .
Obviously δm ≥ α, because otherwise Jm (δm ) ∈ Jm “α and Jm (δm ) < δm .
In particular δm is above αm , i.e., the critical point of jm , so δm is moved
by jm and there exists a µ < δm such that jm (µ) > δm . By contradiction,
suppose that δm = δm+1 . Then µ is also less than δm+1 , so by definition
there exists an i ∈ ω such that Jm+1 (αi ) ≥ µ. Therefore

Jm (αi ) = jm ◦ Jm+1 (αi ) ≥ jm (µ) ≥ δm ,

contradiction.

30
Suppose then that δ0 < λ. Therefore for any m ∈ ω δm+1 < δm , but this
creates a strictly descending sequence of ordinals, contradiction.
• Fix n ∈ ω and let k = j0 (j1 (. . . jn (jn ) . . . )). Then

crt(k) = crt(j0 (j1 (. . . jn (jn ) . . . ))) =


= j0 (j1 (. . . jn (crt(jn )) . . . )) = j0 ◦ j1 ◦ · · · ◦ jn (αn ) = J(αn ).

By Lemma 4.12, then, VJ(αn ) ≺ Vλ . But J ↾ Vαn : Vαn → VJ(αn ) is an


elementary embedding, because J ↾ Vαn = J0,n ↾ Vαn , so for every n ∈ ω,
J ↾ Vαn ≺ Vλ . With methods similar to those in the proof of Theorem
4.6, it is possible to prove that this implies J : Vα ≺ Vλ .

Like in the Vλ case,


S we can extend J to Vα+1 in the expected way: when
A ⊆ Vα , J(A) = β<α J(A ∩ Vβ ). Now we want to prove an equivalent of
Lemma 6.22, but for inverse limits.
Lemma 6.31 (Square root for inverse limits of Σ1n ). Let J : Vα ≺ Vλ an
inverse limit of Σ1n+1 elementary embeddings. Then for all A, B ⊆ Vα there
exist K : Vα ≺ Vλ inverse limit of Σ1n elementary embeddings and A′ ⊆ Vα such
that k(A′ ) = A and k(B) = J(B).
Proof. We define km and Am by induction, with repeated uses of Lemma 6.22.
At the end, K will be the inverse limit of the km ’s, and the Am ’s will be the
images of A′ through the inverse limit.
Let A0 = A, k0 and A1 such that k0 is a Σ1n elementary embedding, A1 ⊆ Vλ ,
k0 (A1 ) = A = A0 ,
k0 (J1 (B)) = j0 (J1 (B)) = J(B)
and crt(k0 ) < crt(j0 ).
More generally, km+1 and Am+2 are such that km+1 is a Σ1n elementary
embedding, Am+2 ⊆ Vλ , km+1 (Am+2 ) = Am+1 ,

km+1 (Jm+2 (B)) = jm+1 (Jm+2 (B))

and crt(jm ) < crt(km+1 ) < crt(jm+1 ).


So km and Am satisfy:
• km is Σ1n ;
• km (Am+1 ) = Am ;
• km (Jm+1 (B)) = Jm (B);
• crt(k0 ) < crt(j0 ) < crt(k1 ) < · · · < crt(jm ) < crt(km ) < crt(jm+1 ) < . . .
Let K be the inverse limit of the k’s. Then supm∈ω crt(km ) = α, and by
Lemma 6.30 K : Vα ≺ Vλ is an elementary embedding. Note that

K(B ∩ Vcrt(km ) ) = k0 ◦ k1 ◦ · · · ◦ km (Km+1 (B ∩ Vcrt(km ) ) =


= k0 ◦ · · · ◦ km (B ∩ Vcrt(km ) ) = k0 ◦ · · · ◦ km (Jm+1 (B ∩ Vcrt(km ) ) =
= J(B ∩ Vcrt(km ) ).

31
So K(B) = J(B).
Finally, consider Am+1 ∩ Vcrt(km ) :
Am+2 ∩ Vcrt(km ) = km+1 (Am+2 ∩ Vcrt(km ) ) = Am+1 ∩ Vcrt(km ) ,
so
K(Am+1 ∩ Vcrt(km ) ) = k0 ◦ · · · ◦ km (Am+1 ∩ Vcrt(km ) ) = A ∩ VK(crt(km )) .
Define A′ = m∈ω (Am+1 ∩ Vcrt(km ) ). Then K(A′ ) = A.
S

We use Lemma 6.31 to calculate the strength of an inverse limit:


Theorem 6.32 (Laver, [27]). If J : Vα ≺ Vλ is an inverse limit of Σ1n elemen-
tary embeddings, then J is Σ1n .
Proof. The case n = 0 is Lemma 6.30 combined with an obvious generalization
of Theorem 4.6, so we proceed by induction on n.
Suppose that J is Σ1n−1 , we need to prove that for every ϕ Π1n−1 -formula,
and any B ⊆ Vλ ,
Vλ  ∃Xϕ(X, J(B)) → Vα  ∃Xϕ(X, B).
Suppose Vλ  ∃Xϕ(X, J(B)), and fix A a witness. Using Lemma 6.31, we
find K inverse limit of Σ1n−2 elementary embeddings such that K(A′ ) = A and
K(B) = J(B) for some A′ ⊆ Vα . So Vλ  ϕ(K(A′ ), K(B)), and by elementarity
Vα  ϕ(A′ , B), that is Vα  ∃X ϕ(X, B).
Finally, we can prove that the existence of a Σ1n+2 elementary embedding
strongly implies the existence of a Σ1n elementary embedding.
Theorem 6.33 (Laver, [27]). Let j : Vλ ≺ Vλ be Σ1n+2 . Then
• for every B ⊆ Vλ , there exist an α < λ and a kα : Vα ≺ Vλ such that
kα (Bα ) = B for some Bα ⊆ Vα . In fact, we can find an ω-club C ⊆ λ of
such α’s.
• there exist an α < λ and a jα : Vα ≺ Vα that is Σ1n . Moreover, we can find
an ω-club C ⊆ λ of such α’s. Therefore “there exists j : Vλ ≺ Vλ Σ1n+2 ”
strongly implies “j : Vλ ≺ Vλ Σ1n+2 ”. In particular I2 strongly implies I3.
Proof. • Let

G = {hl0 , . . . , lm i : li : Vλ ≺ Vλ is Σ1n , crt l0 < crt l1 < · · · < crt lm < κ0 ,


∃B0 , . . . , Bm l0 (B0 ) = B, ∀i li+1 (Bi+1 ) = Bi }.
By Lemma 6.22 the set
{θ < κ0 : ∃l hl0 , . . . , lm , li ∈ G, crt(l) = θ}
is unbounded in κ0 . Pick an infinite branch hl0 , l1 , . . . i of G, and let
α = supi∈ω crt(li ). Let kα be the Sinverse limit of the li ’s. Then by
Theorem 6.32 kα is Σ1n . Define B ′ = m∈ω (Bm+1 ∩ Vcrt(lm ) ) as in Lemma
6.31 to have kα (B ′ ) = B. To prove the existence of the ω-club C, note that
we could have used any infinite branch of T , and the set of the ordinals
that are the supremum of the critical points of the elementary embeddings
appearing in an infinite branch of T (like α) contains an ω-club.

32
• Let the B above be j. Then there exists α (again, any α ∈ C works),
kα : Vα ≺ Vλ , and jα ⊆ Vα such that kα (jα ) = j. Suppose, by Theorem
6.24, that n is odd. Then by Lemma 6.21 ”j is Σ1n ” is Π1n+1 . Again by
Theorem 6.24 and Lemma 6.32 kα is Σ1n+1 , so by elementarity jα : Vα ≺ Vα
is Σ1n .

The situation for I1 has some peculiarities. First of all, if j : Vλ+1 ≺ Vλ+1 ,
then j is definable from j ↾ Vλ , so it is actually a definable class of Vλ+1 This
is not possible in the I3 case, as Vλ  ZFC, so it would contradict the naive
Kunen’s Theorem. We can reformulate Lemma 6.21 in this way:
Remark 6.34. Suppose that j : Vλ+1 → Vλ+1 . Then there are Πn+1 formulas
ϕn (x) such that for any n ∈ ω, Vλ+1  ϕn (j ↾ Vλ ) iff j is an elementary
embedding.
So let j : Vλ+1 ≺ Vλ+1 . Consider j − = j ↾ Vλ . We can define j − (j − ) : Vλ ≺
Vλ . By the remark above [j − (j − )]+ is an elementary embedding, and we write
it simply as j(j). So we can iterate also the I1 embeddings.
As j : Vλ+1 ≺ Vλ+1 is Σ1n for every n, by Lemma 6.22 for any n it is possible
to find a square root of j that is Σ1n . Therefore for any n is it possible to find
α < λ and K : Vα ≺ Vλ inverse limit that is Σ1n , so I1 strongly implies any
“There exists j : Vλ ≺ Vλ that is Σ1n ”. We can do more:
Corollary 6.35. I1(λ) strongly implies ∀n ∃j : Vλ ≺ Vλ that is Σ1n
Proof. Run the proof of Theorem 6.33 n times, each time considering that j is
Σ1n . So there are Cn ω-clubs in λ such that for any α ∈ Cn there exists k : Vα ≺
Vα that is Σ1n . The intersection of ω ω-clubs is an ω-club, so α ∈ n∈ω Cn is as
T
desired.
It is possible to push this kind of proof even more:
Lemma 6.36 (Square root Lemma for L( Vλ+1 ), Laver, [27]). Let j : L1 (Vλ+1 ) ≺
L1 (Vλ+1 ). For every A, B ⊆ Vλ , β < crt(j) there exists k : Vλ+1 ≺ Vλ+1 , with
β < crt(k) < crt(j), such that k(B) = j(B) and there exists A′ ⊆ Vλ such that
k(A′ ) = A.
Proof. The only detail we should care of is the fact that “j : Vλ+1 ≺ Vλ+1 ” must
be definable in L1 (Vλ+1 ). But this is true, because the satisfaction relation in
Vλ+1 is definable in L1 (Vλ+1 ), so the proof is, mutatis mutandis, the same as
Lemma 6.22.
Theorem 6.37 (Laver, [27]). “There exists j : L1 (Vλ+1 ) ≺ L1 (Vλ+1 )” strongly
implies “there exists j : Vλ+1 ≺ Vλ+1 ”.
Proof. The proof follows the same method of Theorem 6.33.
Note that in this case there is a fundamental difference between being Σ1n
for every n and being an elementary embedding from Vλ+1 to itself. Suppose
that we have ji : Vλ+1 ≺ Vλ+1 . Then the inverse limit J of hji : i ∈ ωi is not an
elementary embedding from Vλ′ +1 to Vλ+1 , because the domain of J is just not
Vλ′ +1 . Consider again the definition of the domain of J:

H = {x ∈ Vλ+1 : ∃n ∈ ω ∀m ≥ n jm (x) = x}.

33
Following this definition, for example, λ′ ∈ / H, because it is moved by all ji ,
while on the other hand λ ∈ H, because it is never moved. What we can prove
with the methods provided, is that the unique extension of the inverse limits of
the ji ↾ Vλ to Vλ′ +1 is an elementary embedding.
As I0 is equivalent to j : LΘ (Vλ+1 ) ≺ LΘ (Vλ+1 ), we consider now hypotheses
like “there exists j : Lα (Vλ+1 ) ≺ Lα (Vλ+1 )”, α < Θ, with the aim of finding
strong implications among them.
Note that with such aim not any α will be appropriate. We defined strong
implications only between properties of λ, while “there exists j : Lα (Vλ+1 ) ≺
Lα (Vλ+1 )” is a property of α and λ. We should consider therefore only α’s that
depend on λ, for example α’s that are definable from λ (e.g., λ+ , λ+ + ω, . . . )
or just absolutely definable (e.g., ω, ω ω , . . . ). This kind of ordinals have also
the characteristic of being a fixed point for any large enough embedding, so we
already have that I0 implies “there exists j : Lα (Vλ+1 ) ≺ Lα (Vλ+1 )”.
Our objective is now, given j : Lβ (Vλ+1 ) ≺ Lβ (Vλ+1 ) to find a α < β and
ᾱ, λ̄ < λ such that Lᾱ (Vλ̄+1 ) ≡ Lα (Vλ+1 ) and there exists k : Lᾱ (Vλ̄+1 ) ≺
Lᾱ (Vλ̄+1 ). With β and α as above, this will imply that “exists j : Lβ (Vλ+1 ) ≺
Lβ (Vλ+1 )” strongly implies “exists j : Lα (Vλ+1 ) ≺ Lα (Vλ+1 )”.
The strategy is to follow Laver’s proof for strong implications as much as pos-
sible. The first step is therefore to prove some square root principle like Lemma
6.22. The best possible outcome would be to find for any j : Lα+1 (Vλ+1 ) ≺
Lα+1 (Vλ+1 ) a square root k that is k : Lα (Vλ+1 ) ≺ Lα (Vλ+1 ). For this, it is
needed that
Lα+1 (Vλ+1 )  ∃k : Lα (Vλ+1 ) ≺ Lα (Vλ+1 ),
i.e., that j ↾ Lα (Vλ+1 ) ∈ Lα+1 (Vλ+1 ). Can we assume that?
The next step would be to define an inverse limit of embeddings at the
Lα (Vλ+1 ) level. Here another problem arises following the remarks after The-
orem 6.37: the domain inverse limit of the ki : Lα (Vλ+1 ) ≺ Lα (Vλ+1 ) is not
some Lᾱ (Vλ̄+1 ), in fact it does not even contain Vλ̄+1 , therefore we should do
the inverse limit of ki ↾ Vλ , and then extend it to some Lᾱ (Vλ̄+1 ). How to do
it?
Both problems were solved with the study of the structure of the sets
Lα (Vλ+1 ) for α < Θ made by Laver in [28].
Definition 6.38 (Laver, [28]). Let λ be a cardinal and let α < Θ. Then α is
good iff every element of Lα (Vλ+1 ) is definable in Lα (Vλ+1 ) from an element
in Vλ+1 .

The successor of a good ordinal is a good ordinal: let α be good; the largest
ordinal in Lα+1 (Vλ+1 ) is λ + 1 + α, therefore α is definable in Lα+1 (Vλ+1 )
with λ as a parameter, so Lα (Vλ+1 ) is definable in Lα+1 (Vλ+1 ) with λ as a
parameter. But every element in Lα+1 (Vλ+1 ) is definable using Lα (Vλ+1 ) and
elements of Lα (Vλ+1 ) (which in turn are definable with parameters from Vλ+1 ),
and therefore α + 1 is good. This proves that the natural numbers are good.
But also ω is good: every element of Lω (Vλ+1 ) is in some Ln (Vλ+1 ), n ∈ Vλ+1
and n is good for every n. Following the same line of reasoning, every ordinal
up to λ is good, and considering that all ordinals less than λ+ are coded as
subsets of λ, and therefore in Vλ+1 , every ordinal up to λ+ is good.
On the other side, non-good ordinals exist. The definition of good ordinal
is restricted to ordinals strictly less than Θ because larger ones are trivially not

34
good: if x is definable with a parameter, then it is uniquely determined by its
definition, therefore for any α there exists in L(Vλ+1 ),

π : Vλ+1 ։ Gα = {x ∈ Lα (Vλ+1 ) :
Lα (Vλ+1 )  x is definable from an element in Vλ+1 };

then if Θ ⊆ Lα (Vλ+1 ), Gα must be strictly contained in Lα (Vλ+1 ), by def-


inition of Θ. But non-good ordinals exist also below Θ: define Lγ (Vλ+1 )
as the collapse of the Skolem closure of Vλ+1 in LΘ (Vλ+1 ); as LΘ (Vλ+1 ) 
∃x x not definable from an element in Vλ+1 , by elementarity the same must be
true in Lγ (Vλ+1 ), as the collapse does not collapse Vλ+1 , therefore γ is not good.
One can ask how many good ordinals there are.
Lemma 6.39 (Laver, [28]). Let λ be a strong limit cardinal. Then the good
ordinals are unbounded in Θ.
Therefore assuming α good is in most cases a reasonable choice.
By the usual condensation argument, if α < Θ and i : Lα (Vλ+1 ) → Lα (Vλ+1 )
then i ∈ LΘ (Vλ+1 ). If α is good, however, it is possible to be much more precise:
Lemma 6.40 (Laver, Woodin, [39],[28]). Let λ and α be such that α is good
and there exists i : Lα (Vλ+1 ) ≺ Lα (Vλ+1 ) with crt(i) < λ. Then i is induced by
i ↾ Vλ , and therefore i ∈ Lα+1 (Vλ+1 ).
Finally:
Lemma 6.41 (Square root Lemma for Lα (Vλ+1 )). Let α be good, and let j :
Lα+1 (Vλ+1 ) ≺ Lα+1 (Vλ+1 ). Then for any A, B ∈ Lα (Vλ+1 ), for any β < crt(j)
there exists k : Lα (Vλ+1 ) ≺ Lα (Vλ+1 ) such that β < crt(k) < crt(j), k(k ↾ Vλ ) =
j ↾ Vλ , k(B) = j(B) and there exists A ∈ ran(k).
Proof. Exactly as Lemma 6.22. The key point is that as α is good, “∃k k :
Lα (Vλ+1 ) ≺ Lα (Vλ+1 )” is expressible in Lα+1 (Vλ+1 ) and it is satisfied by j ↾
Lα (Vλ+1 ), so it can be put inside the formula instead of “j is Σ1n−2 ”.
Note that if α is not good, then we can have a similar result: for j :
Lβ (Vλ+1 ) ≺ Lβ (Vλ+1 ), let α be maximum such that j ↾ Lα (Vλ+1 ) ∈ Lβ (Vλ+1 ).
Then we can find a square root that extend to Lα (Vλ+1 ). The difference is that
in this case the gap can be large, and it is somewhat arbitrary, while for the
good case is just one.
Moreover, I0 implies “∃j : Lα (Vλ+1 ) ≺ Lα (Vλ+1 )”:
Lemma 6.42 (Woodin, [39]). Let λ be a cardinal. If there exists j : LΘ (Vλ+1 ) ≺
LΘ (Vλ+1 ), then for any α < Θ there exists an i : Lα (Vλ+1 ) ≺ Lα (Vλ+1 ).
Proof. Suppose it is false. Then there is a counterexample α such that ev-
ery i : Lα (Vλ+1 ) → Lα (Vλ+1 ) is not an elementary embedding. All such
i’s are in LΘ (Vλ+1 ) (see remark before Lemma 6.40), therefore LΘ (Vλ+1 ) 
∃α (α is a counterexample). Let α0 be the least counterexample. Then α0 is
definable in LΘ (Vλ+1 ) and j(α0 ) = α0 . Then j ↾ Lα0 (Vλ+1 ) is as in the lemma,
contradiction.

35
Compare this lemma with the fact that Uj ∈
/ L(Vλ+1 ), and therefore j ↾
LΘ (Vλ+1 ) ∈
/ L(Vλ+1 ). Then if j(α) = α and

j ↾ Lα (Vλ+1 ) : Lα (Vλ+1 ) ≺ Lα (Vλ+1 ),

for any β > α there must be a j1 elementary embedding with domain Lβ (Vλ+1 )
and such that j1 6= j ↾ Lβ (Vλ+1 ), otherwise one could build j ↾ LΘ (Vλ+1 )
inside L(Vλ+1 ). In other words, the tree of embeddings with domain of the type
Lα (Vλ+1 ) has levels of “cardinality” < Θ (by 7.7), has height Θ and there is
no cofinal branch in L(Vλ+1 ). In a certain sense, it is a Θ-Aronszajn tree in
L(Vλ+1 ).
Now we can do the inverse limit like in Theorem 6.31, and so we have
J : Vλ̄ ≺ Vλ , inverse limit of embeddings that can be extended to Lα (Vλ+1 ).
With ᾱ fixed, we can extend J to Lᾱ (Vλ̄+1 ). But how to find a ᾱ so that J is
an elementary embedding?
This point needs more work, and has been solved by Scott Cramer in his
PhD Thesis. The idea is always to use 6.41 repeatedly, to build inverse limits
that have in the range witnesses for existential formulas, so that in the end the
inverse limit is elementary. The difference is that we need λ different witnesses
to be considered every time, so the plan is to build a forcing that at the same
time constructs a limit of inverse limits and collapses λ to ω. We follow with
a sketch of the ideas behind such a construction, for the details we refer the
reader to [3].
The structure of the proof of Theorem 6.31 is like this:

j ↾ Vλ j ↾ Vλ j ↾ Vλ j ↾ Vλ ...

K: k0 k1 k2 k3 ...

where each embedding below is a square root of the embedding above. We


have some freedom for the critical points in the embedding belows, in that for
each one we can say that it must be bigger than a certain fixed ordinal, and
(optionally) less than crt(j). In this way, we can push the limit of the critical
points of the ki ’s to be λ, so that the inverse limit will be from Vλ to itself,
or less than crt(j), as in Lemma 6.31, so to have J : Vλ̄ ≺ Vλ . So, by Lemma
6.41, the existence of j : Lα+1 (Vλ+1 ) ≺ Lα+1 (Vλ+1 ) implies the existence of an
inverse limit J of square roots of j ↾ Vλ such that J : Vλ̄ ≺ Vλ (possibly with
λ̄ = λ), and all the square roots can be extended to Lα (Vλ+1 ).
Instead of j, we can start already with an inverse limit:

J: j0 j1 j2 j3 ...

K: k0 k1 k2 k3 ...
We say that K (the inverse limit of the ki ’s) is a limit root of J (the inverse
limit of the ji ’s) iff there exists an n ∈ ω such that ki = ji for i < n and ki
is a square root of ji for i ≥ n. So for any inverse limit J of embeddings that
extend to Lα+1 (Vλ+1 ) there exist a limit root of J of embeddings that extend
to Lα (Vλ+1 ). Moreover, by 6.41, if J : Vλ̄ ≺ Vλ , for any B ∈ Vλ̄+1 and any
A ∈ Vλ+1 we can have K as before such that K : Vλ̄ ≺ Vλ , K(B) = J(B) and
A ∈ ran(K).

36
The next step is to build a limit of inverse limits. Let j be an embedding
that extends to Lα+ω (Vλ+1 ). Then consider something like this:

j j j j ...

J: j0 j1 j2 j3 ...

K1 j0 k11 k21 k31 ...

K2 j0 k11 k21 k32 ...

.. .. .. .. ..
. . . . . ...
where J is an inverse limit of square roots of j, and every Ki+1 is a limit
n
root of Ki . We can choose the km ’s so that they all extend to Lα (Vλ+1 ): Let
ni be an increasing sequence of integers. Then choose ji to be a square root
of j extendible to Lα+ni (Vλ+1 ). Once defined kim , if it is extendible to some
Lα+n (Vλ+1 ) with n > 0, then choose kim+1 to be a square root of kim that is
extendible to Lα+n−1 (Vλ+1 ), otherwise kim+1 = kim . As ni was increasing, the
last case can happen only a finite amount of times, so Km+1 will always be a
limit square root of Km . Also, at each stage, for any Am ⊆ Vλ̄ (where Vλ̄ )
is the domain of J) and any Bm ⊆ Vλ , we can have K m+1 (A) = K m (A) and
B ∈ ran(kim+1 ◦ ki+1m+1
◦ . . . ), where i is the minimum such that kim 6= kim+1 .
Note that this is less than a Square root Lemma for inverse limits for Lα (Vλ+1 ),
as we ask only B to be in the range of a tail of the limit, and not of K. But
this will be enough.
Theorem 6.43 (Cramer, 2015 [3]). Let α be good, and let j : Lα+ω (Vλ+1 ) ≺
Lα+ω (Vλ+1 ). Let J be as the first step of the construction above, with Vλ̄ its
domain. Then there exist ᾱ, λ̄ < λ such that Lᾱ (Vλ̄+1 ) ≡ Lα (Vλ+1 ).
Proof. First of all, via a definable surjection ρ : Vλ+1 ։ Lα (Vλ+1 ), we can think
of any element of Lα (Vλ+1 ) as a subset of Vλ . We work in V [G], where G is the
generic of the collapse of λ on ω. Therefore Vλ̄+1 is countable: let han : n ∈ ωi
be an enumeration of it, and let hϕi : i ∈ ωi be an enumeration of all formulas.
We continue the construction of the limit of limit roots as above. The idea
is that we gradually add to Am all the elements of Vλ̄ , each time adding just a
finite amount, while Bm will be the set of witnesses of existential formulas with
parameters in ρ′′ Km ′′
Am .
So suppose that we have defined Km and that there exists an nm such that
Am = hai : i < nm i. If ϕm is not a formula with an existential quantifier, then
Km+1 = Km and Am+1 = Am . Otherwise suppose that

Lα (Vλ+1 )  ∃x ψ(x, ρ(Km (ai0 )), . . . ρ(Km (ail ))),

with i0 , . . . , il < nm . We are following the construction of the limit of limit


roots, so at this step there exists a p such that K m+1 should be a limit square
m+1
root of Km such that k0m+1 = k0m , . . . , kpm+1 = kpm , kp+1 m
6= kp+1 , etc... Call
m m
(Km )p = kp ◦ kp+1 ◦ . . . . Then by elementarity (and the definability of ρ)

Lα (Vλ+1 )  ∃x ψ(x, ρ((Km )p (ai0 )), . . . ρ((Km )p (ail ))).

37
Let b that witnesses this, and let Bm be such that ρ(Bm ) = b. Finally let
Km+1 be the limit root of Km such that Km+1 (Am ) = Km (Am ) and Bm ∈
ran((Km+1 )p ). By the remarks above Km+1 exists, and there exists a ∈ Vλ̄+1
such that ρ((Km+1 )p (a)) = b, so
Lα (Vλ+1 )  ψ(ρ((Km+1 )p (a)), ρ((Km+1 )p (ai0 )), . . . ρ((Km+1 )p (ail )))
(note that (Km+1 )p (ai ) = (Km )p (ai )). By elementarity
Lα (Vλ+1 )  ψ(ρ((Km+1 )(a)), ρ((Km+1 )(ai0 )), . . . ρ((Km+1 )(ail ))).
Let a = at , and fix Am+1 = han : n ≤ ti.
If a = an ∈ Vλ̄ , then for any m such that nm > n Km (a) = Km+1 (a). Define
J∗ : Vλ̄+1 ≺ Vλ+1 as J∗ (an ) = K m (an ) with nm > n. It is called the common
part of the sequence hKi : i < ωi. Let M = ρ′′ J∗′′ Vλ+1
¯ . Then M ≺ Lα (Vλ+1 ):
suppose that
Lα (Vλ+1 )  ∃x ψ(x, ρ(J∗ (ai0 )), . . . ρ(J∗ (ail ))),
then there exists m such that nm > ni,0 , . . . , nil and
Lα (Vλ+1 )  ∃x ψ(x, ρ(Km (ai0 )), . . . ρ(Km (ail ))).
But we built Km+1 so that
Lα (Vλ+1 )  ψ(ρ((Km+1 )(a)), ρ((Km+1 )(ai0 )), . . . ρ((Km+1 )(ail ))),
and Km+1 (a) = J∗ (a). By condensation, the collapse of M is a Lᾱ (Vλ̄+1 ) for
some ᾱ. By absoluteness, this is true also in V .
Theorem 6.43 gives us finally the right ᾱ where to extend J, the inverse
limit of embeddings that can be extended to Lα (Vλ+1 ) in the most natural
way: as ᾱ is (λ̄−) good, then every element b ∈ Lᾱ (Vλ̄+1 ) is defined from some
parameter a ∈ Vλ̄+1 , and J(b) is the element of Lα (Vλ+1 ) that is defined with
the same formula and parameter J(a). It remains to see that J is actually an
elementary embedding. Theorem 6.43 gives us an elementary embedding, J∗ ,
but it is in V [G]. But if we consider the partial order of all possible sequences of
Ki ’s, that start with J, coupled with the collapse, then J∗ will be the common
part of the generic of such forcing. Considering J as a sequence of length 1,
for any b ∈ Lᾱ (Vλ̄+1 ) we can always extend J so that J ∗ (b) = J(b), and then
elementarity comes from absoluteness. All the details are in [3].
Theorem 6.44 (Cramer, 2015, [3]). If j : Lα+ω (Vλ+1 ) ≺ Lα+ω (Vλ+1 ), then
there exist ᾱ, λ̄ < λ and k : Lᾱ (Vλ̄ ) ≺ Lᾱ (Vλ̄ ). In particular, if α depends only
on λ, then “∃j : Lα+ω (Vλ+1 ) ≺ Lα+ω (Vλ+1 )” strongly implies “∃j : Lα (Vλ+1 ) ≺
Lα (Vλ+1 )”.
So all the following are strong implications:

I0(λ) ⇒ · · · ⇒ ∃j : Lα+ω (Vλ+1 ) ≺ Lα+ω (Vλ+1 ) ⇒ ∃j : Lα (Vλ+1 ) ≺ Lα (Vλ+1 ) ⇒


⇒ · · · ⇒ ∃j : Lω (Vλ+1 ) ≺ Lω (Vλ+1 ) ⇒ ∃j : L1 (Vλ+1 ) ≺ L1 (Vλ+1 ⇒
⇒ I1(λ) ≡ ∃j : Vλ ≺ Vλ Σ1ω ⇒ ∀n ∃j : Vλ ≺ Vλ Σ1n ⇒ . . .
· · · ⇒ ∃j : Vλ ≺ Vλ Σ1n ≡ ∃j : Vλ ≺ Vλ Σ1n+1 (n is odd) ⇒ . . .
· · · ⇒ ∃j : Vλ ≺ Vλ Σ12 ≡ j : Vλ ≺ Vλ Σ11 ≡ I2(λ) ⇒ iterable I3(λ) ⇒ . . .
· · · ⇒ ω · α-iterable I3(λ) ⇒ α-iterable I3(λ) ⇒ I3(λ).

38
7 Similarities with ADL(R)
We already noticed that for I0(λ) to hold it is necessary that L(Vλ+1 ) 2 AC,
just like for AD to hold in L(R) it is necessary that L(R) 2 AC. This is the
first sign of something bigger. The research is still ongoing on this respect, but
there are many results that indicate a strong similarity between L(Vλ+1 ) under
I0 and L(R) under AD. On the other hand, there are also striking differences,
so there need to be further work on how much the two models are similar and
why (and why they are not too similar).
The first similarity will be the following:
Theorem 7.1 (Solovay). Suppose L(R)  AD. Then L(R)  ω1 is measurable.
Lemma 7.2 (Woodin, [39]). Suppose that there exists j : L(Vλ+1 ) ≺ L(Vλ+1 )
with crt(j) < λ. Then for every δ < λ regular, define
+
Sδλ = {η < λ+ : cof(η) = δ},

the stationary set of ordinals with the same cofinality and let F be the club filter
+
on λ+ . Then there exists η < λ and hSα : α < ηi ∈ L(Vλ+1 ) a partition of Sδλ
such that for every α < η F ↾ Sα is a L(Vλ+1 )-ultrafilter in λ+ . In particular
λ+ is measurable.
Proof. Define κ0 = crt(j). Suppose that there is a δ such that this is false, and
pick the minimum one. Then δ is definable (using only λ as a parameter, that
is a fixed point), so j(δ) = δ, and this means that δ < κ0 . Moreover, note that
j ↾ λ+ is in L(Vλ+1 ), because the elements of λ+ are well-orders of λ, so j ↾ λ+
depends on j ↾ Vλ+1 , that in turn is defined by j ↾ Vλ , that is in Vλ+1 .
+
As in the proof of 3.13, there is no partition of Sδλ in κ0 stationary sets.
+
The difference is that here we need that C = {α < λ : j(α) = α} is a δ-club.
But it is: let T~ be a δ-sequence of elements of D. Then T~ can be coded in
Vλ+1 , and since j(η) = η for every η ∈ ~t and j(δ) = δ we have j(~t) = ~t, so
j(sup ~t) = sup j(~t) = sup ~t.
Let I be the non-stationary ideal, i.e. the ideal of the non-stationary sets,
+
and consider B = P(Sδλ )/I. We prove that B is atomic, and its atoms are
dense. Looking for a contradiction suppose that there exists a set S ∈ I + that
contains no atom, so that

∀T ⊂ S (T ∈ I + → (∃T0 , T1 ∈ I + (T0 ∪ T1 = T ∧ T0 ∩ T1 = ∅))).

We construct by induction a tree T of subsets of S that refines reverse inclusion.


Indicating Levγ (T ) as the γ-th level of T we define Lev0 (T ) = {S} and every
T ∈ T has exactly two successors T0 , T1 ∈ I + such that T0 ∪T1 = TTand T0 ∩T1 =
∅, chosen with DCλ . At a limit α, we put the intersections T = β<α Tβ , with
Tβ ∈ Levβ (T ), such that T ∈ I + . We call Res(α) = S \ Levα (T ) the set of
the points in S that don’t belong to any T in Levα (T ). Now, let α ≤ κ0 be a
we want to prove by induction that Res(α) ∈ I and Levα (T ) 6= ∅.
limit ordinal, S
Let x ∈ S \ β<α Res(β), then for every β < αSthere exists T ∈ Levβ (T )
such that x ∈ T , so we can defineTbx = {Z ∈ β<α Levβ (T )S: x ∈ Z} a
branch of T of length α and x ∈ bx . So S is the union T of β<α Res(β),
the points already skimmed by the process, and all of the bx . By induction

39
every Res(β) S ∈ I, and since I is the dual of T
the club filter, SI is λ+ -complete
by DCλ , so β<α Res(β) ∈ I. We consider { bx : x ∈ S \ β<α Res(β)}: it
is a partition of S \ β<α Res(β) ∈ I + and since λ is a strong limit we have
S

that there cannot be more than 2|α| < λ branches,Tso by λ+ -completeness there
must be at least one set T ∈ I + such that T = bx for someTbranch bx , i.e.
there must exist T ∈ Levα (T ). As for the branches such that bx ∈ / I + , their
union, again by completeness, must be in I, so
[ [ \ \
Res(α) = Res(β) ∪ { bx : / I + } ∈ I,
bx ∈
β<α

and we can carry on the induction. If we consider a branch in T of length κ0 ,


say hTα : α < κ0 i, then

{Res(κ0 ) ∪ (T0 \ T1 )} ∪ {Tβ \ Tβ+1 : β < κ0 }

is a partition of S in κ0 stationary sets, and we’ve just seen that this cannot be,
so we’ve reached a contradiction.
+
We’ve proved that the atoms of B = P(Sδλ )/I are dense, so for every
+
S ⊆ Sδλ there exists T ⊆ S atom for B, in other words there exists T ⊆ S in I +
such that F ↾ T is an ultrafilter. Since by DCλ we have that F is λ+ -complete,
F ↾ T is a measure. By induction (using DCλ ) we define hSα : α < ηi:
+
• Let S = Sδλ . Then there exists a T ∈ I + such that F ↾ T is a measure,
and we choose S0 = T .
+
Let α ordinal, and suppose that for every β < α, Sβ is defined. If Sδλ \
• S
β<α Sβ ∈ I, then we stop the sequence and η = α. Otherwise there
+
exists Sα ⊆ Sδλ \ β<α Sβ such that Sα ∈ I + is stationary and F ↾ Sα is
S
a measure.
In particular, F ↾ S0 is a measure for λ+ .
It is remarkable that the proof is completely different, yet in both theorems
the measure is not just any measure, but it comes from the club filter. Woodin
calls the cardinals that are measurable for this reason “ω-strongly measurable
cardinals”, and they have a key role in the development of the inner model
descriptive set theory.
The fact that λ+ is measurable has a lot of ADR -like consequences:
Theorem 7.3 (Shi, Trang, 2017 [36]). Suppose I0(λ) holds. Then in L(Vλ+1 ):
• there is no λ+ -Aronszajn tree;
• λ fails;
• there is no scale at λ in Vλ+1 ;
• ♦λ+ fails;
• there is no λ+ -sequence of distinct members of P(λ).
In fact Theorem 7.1 was just the first ω-strongly measurable cardinal found
in L(R), when there is a rich structure of them under AD. The same is true in
L(Vλ+1 ) under I0. Moschovakis’ Coding Lemma is the way to find them:

40
Theorem 7.4 (Coding Lemma, Moschovakis). Suppose L(R)  AD. Let η < Θ
and ρ : R ։ η, ρ ∈ L(R). Then there exists γρ < Θ such that for every
A ⊆ R × R, A ∈ L(R), there exists B ⊆ R × R such that:
• B ⊆ A;
• B ∈ Lγρ (R);
• for every α < η if ∃(a, b) ∈ A ρ(a) = α then ∃(a, b) ∈ B ρ(a) = α.
It is a sort of very weak choice principle for subsets of R: Consider Aξ =
{b : (a, b) ∈ A, ρ(a) = ξ}. Thus A can be seen as an η-sequence of subsets of
R. If we had the Axiom of Choice, we could have a selector for such a sequence.
Instead the Coding Lemma gives us a sequence Bξ ⊆ Aξ that is in some Lγρ (R).
But however “complex” (i.e., high in the constructive hierarchy) we choose A,
γρ is the same, so the “selector” is actually simpler than A.
We want to prove the Coding Lemma in L(Vλ+1 ). First we prove a weakening
of the Coding Lemma, that will be used in the proof of the Coding Lemma itself.
Lemma 7.5 (Weak Coding Lemma, Woodin, [39]). Suppose that there exists
j : L(Vλ+1 ) ≺ L(Vλ+1 ) with crt(j) < λ. Let η < Θ and ρ : Vλ+1 ։ η,
ρ ∈ L(Vλ+1 ). Then there exists γρ < Θ such that for every A ⊆ Vλ+1 × Vλ+1 ,
A ∈ L(Vλ+1 ), there exists B ⊆ Vλ+1 × Vλ+1 such that:
• B ⊆ A;
• B ∈ Lγρ (Vλ+1 );
• for cofinally α < η if ∃(a, b) ∈ A ρ(a) = α then ∃(a, b) ∈ B ρ(a) = α.
Proof. Since this lemma is quite rich in terms of quantifiers, we will use some
abbreviation, calling W CL the sentence of the Lemma, W CLη the same, but
with fixed η, W CLη,ρ the same, but with fixed η and ρ, and so on. . .
Looking for a contradiction, suppose that W CL is false and let η be the
least such that ¬W CLη . Then η is definable, so j(η) = η, and η is a limit: in
fact, W CLη → W CLη+1 , since for cofinally α < η + 1 means for α = η, and in
that case we can just choose an element in {(a, b) ∈ A : ρ(a) = η}.
Let ρ be such that ¬W CLη,ρ . We define by induction, using DCλ , h(γξ , Zξ ) :
ξ < λi:
• Suppose that we have defined (γξ , Zξ ). Since ¬W CLη,ρ , for every γ < Θ
¬W CLη,ρ,γ ; so by ¬W CLη,ρ,γξ there exists Zξ+1 ⊆ Vλ+1 × Vλ+1 such that
– {ρ(a) : ∃b (a, b) ∈ Zξ+1 } is unbounded in η;
– for every B ⊆ Zξ+1 , B ∈ Lγξ (Vλ+1 ), {ρ(a) : ∃b (a, b) ∈ B} is bounded
in B.
Let γξ+1 be such that Zξ+1 ∈ Lγξ+1 (Vλ+1 ) and γξ + ω < γξ+1 . By Lemma
5.6 we can suppose that γξ+1 < Θ.
• In the limit case, we choose γξ such that γξ > γζ + ω for every ζ < ξ and
a Zξ ⊆ Vλ+1 × Vλ+1 such that Zξ ∈ Lγξ (Vλ+1 ).
So, for every ξ1 < ξ2 < λ we have:

41
• γξ1 + ω < γξ2 ;
• Zξ1 ∈ Lγξ1 (Vλ+1 );
• {ρ(a) : ∃b (a, b) ∈ Zξ+1 } is unbounded in η and for every B ⊆ Zξ+1 ,
B ∈ Lγξ (Vλ+1 ), {ρ(a) : ∃b (a, b) ∈ B} is bounded in B.
Let ρ0 = ρ, ρn+1 = j(ρn ),

h(γξ0 , Zξ0 ) : ξ < λi = h(γξ , Zξ ) : ξ < λi

and
h(γξn+1 , Zξn+1 ) : ξ < λi = j(h(γξn , Zξn ) : ξ < λi).

Let n be the minimum such that γκn0 +1 ≤ γκn+1


0 +1
(there must exist, otherwise
hγκn0 : n ∈ ωi would be a descendent chain of ordinals). By elementarity we
have that:
• Zκn0 +1 ∈ Lγκn (Vλ+1 );
0 +1

• {ρn+1 (a) : ∃b (a, b) ∈ Zκn+1


1 +1
} is unbounded in η and for every B ⊆ Zκn+1
1 +1
,
B ∈ Lγκn+1 (Vλ+1 ) we have that {ρn+1 (a) : ∃b (a, b) ∈ B} is bounded in η;
1

• j(Zκn0 +1 ) = Zκn+1
1 +1
.
Let B = {(j(a), j(b)) : (a, b) ∈ Zκn0 +1 }. The parameters used in the definition
of B are j ↾ Vλ+1 (that in turn is defined by j ↾ Vλ ) and Zκn0 +1 , so B ∈
Lγκn +1 +1 (Vλ+1 ) ⊆ Lγκn1 (Vλ+1 ). If (a, b) ∈ Zκn0 +1 , then
0

j((a, b)) = (j(a), j(b)) ∈ j(Zκn0 +1 ) = Zκn+1


1 +1
,

so B ⊆ Zκn+1
1 +1
. Finally, for every α < η there exists (a, b) ∈ Zκn0 +1 such that α <
ρn (a), so α ≤ j(α) < ρn+1 (j(a)), and {ρn+1 (a) : ∃b (a, b) ∈ B} is unbounded in
η. Contradiction.
Theorem 7.6 (Coding Lemma, Woodin, [39]). Suppose that there exists j :
L(Vλ+1 ) ≺ L(Vλ+1 ) with crt(j) < λ. Let η < Θ and ρ : Vλ+1 ։ η, ρ ∈ L(Vλ+1 ).
Then there exists γρ < Θ such that for every A ⊆ Vλ+1 × Vλ+1 , A ∈ L(Vλ+1 ),
there exists B ⊆ Vλ+1 × Vλ+1 such that:
• B ⊆ A;
• B ∈ Lγρ (Vλ+1 );
• for every α < η if ∃(a, b) ∈ A ρ(a) = α then ∃(a, b) ∈ B ρ(a) = α.
Proof. The proof is by induction on η. Let ρ : Vλ+1 ։ η. For every α < η we
define ρα : Vλ+1 ։ α:
(
ρ(a) if ρ(a) < α;
ρα (a) =
0 otherwise.

By induction, for every α < η there exists γρα < Θ that satisfies the Coding
Lemma for η, ρα . Let β0 = sup{γρα : α < η}. Since Θ is regular, we have that

42
β0 < Θ. Let γρ be the ordinal that witnesses the Weak Coding Lemma for η, ρ,
and let β1 = sup{β0 , γρ }. Since β1 < Θ, there exists π : Vλ+1 ։ Lβ1 (Vλ+1 ),
and this π can be codified as a subset of Vλ+1 . We call β the ordinal < Θ such
that the code of π is in Lβ (Vλ+1 ). We prove that β + 1 witnesses the Coding
Lemma for η, ρ.
Fix A and define A<α = {(a, b) ∈ A : ρ(a) < α}. We can suppose that for
every α < η there exists (a, b) ∈ A such that ρ(a) = α. We want to code the set
{(A<α , B) : α < η, B witnesses the Coding Lemma for η, ρα , A<α } as a subset
of Vλ+1 × Vλ+1 :

A∗ = {(a, b) ∈ Vλ+1 × Vλ+1 : ρ(a) > 0, π(b) ⊆ A<ρ(a) ,


∀ξ < ρ(a) ∃(x, y) ∈ π(b) ρ(x) = ξ}.

Since β witnesses the Coding Lemma for all the ρα , we have that {ρ(a) :
∃b (a, b) ∈ A∗ } = η, and since the Weak Coding Lemma holds for η, ρ, β,
there exists B ∗ ⊆ A∗ such that B ∗ ∈ Lβ (Vλ+1 ) and {ρ(a) : ∃b (a, b) ∈ B ∗ } is
cofinal in η. S
Let B = {π(b) : (a, b) ∈ B ∗ }. Then B ⊆ A and B ∈ Lβ+1 (Vλ+1 ). More-
over, for every α < η, there exists (a, b) ∈ B ∗ such that ρ(a) > α, π(b) ∈ A<ρ(a)
and there exists (x, y) ∈ π(b) such that ρ(x) = α. But then (x, y) ∈ B, and B
witnesses the Coding Lemma for η, ρ, β + 1, A.

The Coding Lemma is used to prove a sort of inaccessibility of Θ in L(Vλ+1 ):


as L(Vλ+1 ) 2 AC, we cannot say 2γ < Θ for all γ < Θ; but we have the following:
Lemma 7.7 (Woodin, [39]). Suppose that there exists j : L(Vλ+1 ) ≺ L(Vλ+1 )
with crt(j) < λ. Then in L(Vλ+1 ) for every α < Θ there exists a surjection
π : Vλ+1 ։ P(α).
Proof. Fix α < Θ, ρ : Vλ+1 ։ α, and let γ be the maximum between the least
β < Θ such that ρ ∈ Lβ (Vλ+1 ) and the witness for the Coding Lemma for α, ρ.
For every A ⊆ α define A∗ = {(a, 0) ∈ Vλ+1 : ρ(a) ∈ A}. Then A∗ ∈ L(Vλ+1 ).
So there exists B ∗ ⊆ A∗ , B ∗ ∈ Lγ (Vλ+1 ) such that {ρ(a) : ∃b (a, b) ∈ B ∗ } = A.
But then A ∈ Lγ+1 (Vλ+1 ). This means that P(α) ⊆ Lγ+1 (Vλ+1 ), and using
π : Vλ+1 ։ γ + 1 we’re done.
The following is the strongest theorem related to the Coding Lemma that
has been proved until the writing of this paper, and it is akin to a similar one
in ADL(R) (cfr. with [32]):
Theorem 7.8 (Woodin, [39]). Suppose that there exists j : L(Vλ+1 ) ≺ L(Vλ+1 )
with crt(j) = κ0 < λ. Then Θ is a limit of γ such that:
1. γ is weakly inaccessible in L(Vλ+1 );
2. γ = ΘLγ (Vλ+1 ) and j(γ) = γ;
3. for all β < γ, P(β) ∩ L(Vλ+1 ) ∈ Lγ (Vλ+1 );
4. for cofinally κ < γ, κ is an ω-strongly measurable cardinal in L(Vλ+1 );
5. Lγ (Vλ+1 ) ≺ LΘ (Vλ+1 ).

43
Proof. Fix α < Θ, j(α) = α and let γ < Θ be the minimum such that
Lγ (Vλ+1 ) ≺1 LΘ (Vλ+1 ). As also j(Θ) = Θ, we have j(γ) = γ. This will be
the γ as in the Theorem. By Corollary 5.15 we can have γ as large as we want
below Θ.
It is tempting to stop here, and say that γ has the same properties of Θ by
elementarity, but we do not have this directly, as Θ ∈ / LΘ (Vλ+1 ).
Note that, by 5.8 relativized to Σ1 (α) formulas, ∆21 (α) sets, etc. . . , ΘLγ (Vλ+1 ) =
γ, and therefore (2). Also, cof(δ) > λ: Otherwise let hβξ : ξ < µi be a sequence
cofinal in δ with µ < λ. Then any βξ is the length of a ∆21 (α) pwo. With DCλ
choose one pwo for any ξ, and then gluing this pwos together it is possible to
find a ∆21 (α) pwo of γ, and this contradicts 5.8. Now the Weak Coding Lemma
can be re-proved in Lγ (Vλ+1 ): the key points are that since ΘLγ (Vλ+1 ) = γ,
γξ+1 < γ and since cof(γ) > λ the sequence of the Zξ is longer than λ, and we
can find the contradiction.
With the Weak Coding Lemma we prove that γ is regular. It is similar to
the proof that its cofinality is larger than λ, using the Weak Coding Lemma
instead of DCλ : Let β < γ and f : β → γ cofinal in γ. Let π be a Σ1 (α) partial
bounded surjection from Vλ+1 to Lγ (Vλ+1 ), as in Lemma 5.8. As ΘLγ (Vλ+1 ) = γ,
there exists ρ : Vλ+1 ։ β with ρ ∈ Lγ (Vλ+1 ). Let δ witness the Weak Coding
Lemma for ρ in Lγ (Vλ+1 ), and consider

A = {(a, b) : b ∈ dom(π) ∧ π(b) = f (ρ(a))}.

Then there exists B ⊆ A, B ∈ Lδ (Vλ+1 ) such that for cofinally ξ < β if


there exists (a, b) ∈ A ρ(a) = ξ then there exists (a, b) ∈ B such that ρ(a) = ξ.
As for every ξ, Aξ 6= ∅, this means that for cofinally ξ, Bξ 6= ∅. Let B 0 =
{a : ∃b (a, b) ∈ B}. Then π ′′ B 0 is cofinal in β, so (f ◦ π)′′ B 0 is cofinal in γ.
Let Z = {b : ∃a ∈ Vλ+1 (a, b) ∈ B}. As B ⊆ A, ρ′′ Z = (f ◦ π)′′ B 0 , but that
contradicts the boundedness of ρ.
Since γ is regular, then the Coding Lemma can be re-proved relativized to
Lγ (Vλ+1 ), so also Lemma 7.7 holds in Lγ (Vλ+1 ), therefore (3) is proved. So γ
is weakly inaccessible because of regularity and (3).
Let Sωγ = {α < γ : cof(α) = ω}. Let F be the club filter on Sωγ . Instead
of proving that F is an ultrafilter on a stationary set, we define another filter
F0 ⊆ F , that is an ultrafilter on a stationary set, and that coincides with F
there.
Let E = {k : Lγ (Vλ+1 ) ≺ Lγ (Vλ+1 )}. For each k ∈ E, let Ck = {η ∈ Sωγ :
k(η) = η}. As γ is regular, Ck is an ω-club. For each σ ⊆ E such that |σ| ≤ λ, let
Cσ = ∩{Ck : k ∈ σ}. Let F0 be the filter on Sωγ generated by the sets Cσ ∩ Sωγ .
It is λ+ -complete by DCλ . All its elements are clubs on Sωγ , so F0 ⊆ F . Also,
j(F0 ) = F0 .
As in 7.2 S cannot be partitioned in κ0 many F0 -positive sets, since j ↾
Lγ (Vλ+1 ) ∈ E and therefore Cj ∈ F0 . So there must exists a T ⊆ S stationary
such that F0 ↾ T is an ultrafilter. As it is a maximal filter on T , it must be that
F ↾ T = F0 ↾ T . It remains to show that F on T is γ-complete.
Let β < γ and hAξ : ξ < βi a sequence of subsets of T of F -measure one.
Let ρ : Vλ+1 ։ β, with ρ ∈ Lγ (Vλ+1 ). Let η to witness the Coding Lemma in
Lγ (Vλ+1 ) for ρ. Ideally, we would like to use the Coding Lemma on

A = {(Aξ , Cσ ) : σ ⊆ E, |σ| ≤ λ, ξ < β, Cσ ⊆ Aξ },

44
to “choose” some Cσ inside every Aξ , so that we can intersect them in the
following way: Suppose we can find B ⊆ A “small” enough, so that, if

EB = {k ∈ E : ∃ξ ∃σ ξ < β, σ ⊆ E, |σ| ≤ λ, (Aξ , Cσ ) ∈ B k ∈ σ},

for any ξ < γ, sup{k(ξ) : k ∈ EB } < γ. Then pick ξ0 > α, ξn+1 = sup{k(ξn ) :
k ∈ EB }. Then ξω = supn∈ω ξn ∈ T and for any k ∈ EB k(ξω ) = ξω . This means
that for any Aξ , ξω ∈ Aξ , so we found an intersection.
Therefore we have to code A as a subset of Vλ+1 × Vλ+1 : Note that γ is
good, therefore by 6.40 for any k1 , k2 ∈ E, k1 = k2 iff k1 ↾ Vλ = k2 ↾ Vλ , so there
is a bijection between E and E0 = {k ↾ Vλ : k ∈ E} ⊆ Vλ+1 . Moreover let F be
any bijection between Vλ+1 and λ Vλ+1 : for a ∈ Vλ+1 we can define Ca = Cσ(a)
with σ(a) = {k : k ↾ Vλ ∈ F (a)}.
Let
A = {(a, b) ∈ Vλ+1 × Vλ+1 : F (b) ⊆ E0 , Cb ⊆ Aρ(a) }.
Let B ⊆ A, B ∈ Lη (Vλ+1 ) given by the Coding Lemma. Let

EB = ∪{F (b) : ∃a (a, b) ∈ B} ∈ Lη+1 (Vλ+1 ),

a set of codes for embeddings k such that Ck ⊆ Aξ for some ξ. Let a ∈ dom(π).
Note that, as π is Σ1 (α)-definable, if a ∈ dom(π) then k(a) ∈ dom(π) for any k
embedding, so

Wa = {k(a) : k ∈ E, k ↾ Vλ ∈ EB } ⊆ dom(π),

and Wa ∈ Lη+2 (Vλ+1 ). So

π ′′ Wa = {π(k(a)) : k ∈ E, k ↾ Vλ ∈ EB } = {k(π(a)) : k ∈ E, k ↾ Vλ ∈ EB }.

Let ξ < γ, and a ∈ Vλ+1 such that π(a) = ξ. So π ′′ Wa = {k(ξ) : k ∈ E, k ↾ Vλ ∈


EB }. As Wa is bounded, then π ′′ Wa is bounded, so sup{k(ξ) : k ∈ E, k ↾ Vλ ∈
EB } < γ. Now we can find a fixed point for all the extension of embeddings in
EB , and we are done.
For the point (5) we just sketch the proof. The idea is to consider P , the
theory of LΘ (Vλ+1 ), and relativize the previous proof for Lγ (Vλ+1 )[P ]. Then

(Lγ (Vλ+1 )[P ], P ∩ Lγ (Vλ+1 )) ≺1 (LΘ (Vλ+1 ), P )

implies full elementarity.


There are still open questions about the combinatorial nature of cardinals
between λ and Θ, following the similarities with ADL(R) :
Question 7.9. Suppose I0(λ), are λ++ , λ+++ regular or singular in L(Vλ+1 )?
Question 7.10. Suppose I0(λ). Is it true that in L(Vλ+1 ) every regular cardinal
below Θ is measurable?
Another point in favour of AD is the regularity properties that it entails, and
this is partially true also for I0.
We consider on Vλ+2 the topology with basic open sets O(α,a) = {x ⊆ Vλ :
x ∩ Vα = a}, with α < λ and a ⊆ Vα . Then a set X ⊆ Vλ+1 is perfect iff Πn∈ω κn
can be continuously embedded into X. A set has the perfect set property iff it
has cardinality ≤ λ or it contains a perfect subset.

45
Theorem 7.11 (Cramer, 2015 [3]). Suppose there exists j : L(Vλ+1 ) ≺ L(Vλ+1 ),
with crt(j) < λ. Then every set in L(Vλ+1 ) ∩ Vλ+2 has the perfect set property
in L(Vλ+1 ).
The proof uses heavily all the inverse limit tools developed in Section 6. It
exploits the fact that there is a multiplicity of possible square roots: Consider
J : Lᾱ+1 (Vλ̄+1 ) ≺ Lα+1 (Vλ+1 ), inverse limit, and let X ∈ Lα+1 (Vλ+1 ) ∩ Vλ+2
and X̄ ∈ Lᾱ+1 (Vλ̄+1 ) such that J(X̄) = X. Then if |X| > λ we can find for any
a ∈ X̄ many (> λ) limit roots of J, all with different images on a. Repeated
uses of this kind of pigeonhole principle will be used to construct a perfect set
inside X.
Question 7.12. Is there an equivalent in L(Vλ+1 ) for Baire property or Lebesgue
measurability?
Some key properties of subsets of R are proven thanks to the fact that
under AD all such sets have a nice tree-structure, namely they are weakly ho-
mogeneously Suslin. Woodin introduced a definition that wants to develop a
similarity with this concept: U(j)-representability.
If x ∈ Vλ+1 and hλi : i ∈ ωi is cofinal in λ, then we can see x as the ω-
sequence hx ∩ Vλi : i ∈ ωi, just like any real can be seen as an ω-sequence of
natural numbers. So any set X ⊆ Vλ+1 can be seen as a set of ω-sequences. In
R, homogeneous Suslin-ness consists in labeling any finite sequence of natural
numbers with an ultrafilter, so that if a sequence labeled with U2 extends an-
other labeled with U1 , then U2 projects to U1 . If U2 projects to U1 , then there
is a related elementary embedding from Ult(V, U1 ), to Ult(V, U2 ), therefore for
any infinite sequence one can define the direct limit of all the ultrapowers of the
ultrafilters that label its finite initial segments. Then a set Z is homogeneously
Suslin iff there is such labeling so that a branch is in Z iff such direct limit is
well-founded.

.. ..
. .

U5 Ult(V, U5 )

U4 Ult(V, U4 )

U3 Ult(V, U3 )
U3

U2 Ult(V, U2 )
U2

U1 Ult(V, U1 )
U1

46
Weakly homogeneous Suslin-ness adds another dimension, considering instead
the couples of finite sequences.
So the first step is to decide which ultrafilters are going to label our se-
quences:
Definition 7.13. Let U(j) be the set of U ∈ L(Vλ+1 ) such that in L(Vλ+1 ) the
following hold:
• U is a λ+ -complete ultrafilter;
• for some γ < Θ, U ⊆ P(Lγ (Vλ+1 ));
• for some A ∈ U and all sufficiently large n ∈ ω, j n (U ) = U and {a ∈ A :
j n (a) = a} ∈ U .
Let κ < Θ, κ ≤ ΘLκ (Vλ+1 ) . Let hai : i < ωi be such that for all i ∈ ω there
exists n ∈ ω such that j n (ai ) = ai . Then U(j, κ, hai : i ∈ ωi) is the set of U ∈
U(j) such that there exists an n ∈ ω such that for all k : Lκ (Vλ+1 ) ≺ Lκ (Vλ+1 )
with the property that there are m, l ∈ ω, k m = j l ↾ Lκ (Vλ+1 ), if for all i ≤ n
k(ai ) = ai , then {a ∈ Lκ (Vλ+1 ) : k(a) = a} ∈ U .
Now we can introduce U(j) representability:
Definition 7.14 (Woodin, 2011 [41]). Let Z ∈ L(Vλ+1 )∩Vλ+2 . Then Z is U(j)-
S are κ, hai : i ∈ ωi, an increasing sequence hλi : i ∈ ωi,
representable if there
and a function π : {Vλi +1 × Vλi +1 × {i} : i ∈ ω} → U(j, κ, hai : i ∈ ωi) such
that the following hold:
• for all i ∈ ω and (a, b, i) ∈ dom(π) there exists A ⊆ (L(Vλ+1 ))i such that
A ∈ π(a, b, i);
• for all i ∈ ω and (a, b, i) ∈ dom(π), if m < i then (a ∩ Vλm , b ∩ Vλm , m) ∈
dom(π) and π(a, b, i) projects to π(a ∩ Vλm , b ∩ Vλm , m) ∈ dom(π), i.e., for
every A ∈ π(a ∩ Vλm , b ∩ Vλm , m)

{x ∈ (L(Vλ+1 ))i : x ↾ m ∈ A} ∈ π(a, b, i);

• for all x ⊆ Vλ , x ∈ Z iff there exists y ⊆ Vλ such that the tower hπ(x ∩
Vλm , y ∩ Vλm , m : m ∈ ωi is well-founded.
The question is now whether there are sets in L(Vλ+1 ) ∩ Vλ+2 that are U(j)-
representable, and particularly which ones are. The desired result would be, in
similarity with AD, that they are all U(j)-representable. There has been some
twists and turns on the still ongoing quest towards this result, and it is worth
to see them step by step, with some advice for the reader that wants to know
all the details.
The main source for U(j)-representability is [41]. There the first closure
properties are introduced, reminiscent of the closure properties for weakly ho-
mogeneously Suslin sets:
Theorem 7.15 (Woodin, Lemma 114, Lemma 115 and Theorem 134 in [41]).
Suppose j : L(Vλ+1 ) ≺ L(Vλ+1 ) with crt(j) < λ. Then
• the λ-union of U(j)-representable sets is U(j)-representable

47
• the projection of a U(j)-representable set is U(j)-representable
• if Z ∈ Lλ (Vλ+1 ) is U(j)-representable, then its complement is U(j)-representable.
Therefore all sets in Lλ (Vλ+1 ) ∩ Vλ+2 are U(j)-representable.
This was the best result for U(j)-representability in [41]. Further research
was blocked by the Tower Condition. Remember by 6.16 that a tower of ultrafil-
ters hUn : n ∈ ωi is well-founded iff there exists an X such that X ∩ dom(Un ) ∈
Un for all n ∈ ω. The Tower Condition holds for U(j) iff for any U(j)-
representation, it is possible to find such an X for any well-founded tower
uniformly:
Definition 7.16 (Woodin, 2011 [41]). The Tower Condition holds for U(j) iff
for any A ⊆ U(j), A ∈ L(Vλ+1 ), |A| ≤ λ, there is a function F : A → L(Vλ+1 )
such that, for all hUi : i ∈ ωi ∈ L(Vλ+1 ) that satisfies:
• for all i ∈ ω there exists Z ∈ Ui such that Z ⊆ L(Vλ+1 )i ;
• for all i ∈ ω Ui ∈ A and Ui+1 projects to Ui .
Then the tower is wellfounded iff there exists a function f : ω → L(Vλ+1 ) such
that for all i ∈ ω, f ↾ i ∈ F (Ui ).
Note that if π is a U(j) representation and A = ran(π), then all towers in A
satisfy the conditions. Woodin immediately noticed that the Tower Condition
was central for the study of U(j)-representable sets:
Lemma 7.17 (Woodin, Lemma 127 in [41]). If the Tower Condition holds for
U(j), then the complement of a U(j)-representable set is U(j)-representable,
therefore all the sets in Lλ+ (Vλ+1 ) are U(j)-representable.
For much of Section 7 of [41], the Tower Condition is a constant and rather
cumbersome presence, as at the time of the publication it was still not proven. In
[3] Cramer actually managed to prove the Tower Condition, therefore unlocking
much of the content of Section 7 of [41]. The suggestion for the completist
reader is therefore first to read [3], and reading Section 7 of [41] after, so that
the Tower Condition provides the most results.
This approach would also make some proofs in [41] more direct and easier
to read. For example:
Lemma 7.18 (Woodin, Lemma 121 of [41]). Suppose Z ∈ L(Vλ+1 ) is U(j)-
representable and |Z| > λ. Then Z contains a R-perfect set5 .
The proof of such Lemma starts by fixing an elementary embedding j0 :
+
V → M0 such that (M0 )λ ⊆ M0 , A0 = {j0 (U ) : U ∈ ran(π)}. All of this
is to prove the Tower Condition for j0′′ A0 . Now that we know that the Tower
Condition holds, we can ignore this trick and just consider M0 = V , j0 = id
and A0 = ran(π).
It is also possible to improve 7.17:
5 With R-perfect set we intend that 2ω can be continuously embedded in Z, a weaker

property than to be a perfect set. Therefore this Lemma is now incorporated in 7.11

48
Lemma 7.19 (Woodin, Corollary 148 of [41]). Suppose there is j : L(Vλ+1 ) ≺
L(Vλ+1 ) with crt(j) < λ. Let κ = λ+ and η = sup{(κ+ )L[A] : A ⊆ λ}. Then
every set in Lη (Vλ+1 ) is U(j)-representable.
So it is still open:
Question 7.20. Are all sets in L(Vλ+1 ) ∩ Vλ+2 U(j)-representable?

8 Rank-into-rank and forcing


A recurrent image for describing the difference of the extension of the universe
of sets via large cardinals or via forcing is that large cardinals extend the uni-
verse vertically, while forcing extends the universe horizontally. Such image has
definitively its shortcomings, first because stronger large cardinals do not always
mean larger large cardinals (we will see an example of this), second because it
implies that large cardinals and forcing are orthogonal, while there are cases of
interdependence. It is therefore natural, facing new axioms, to ask what is their
relationship with forcing. The ideal results are two:

• A forcing always destructs the large cardinal axiom. This would be a


strong indication that the propositions forced could be inconsistent with
the large cardinal.
• A forcing never destructs the large cardinal axiom. This means that the
proposition forced is independent from the large cardinal, and we are
actually in a situation of orthogonality.
The test case has always been Cohen forcing: any large cardinal cannot be
destructed by Cohen forcing, therefore the Continuum Hypotheses is indepen-
dent from any large cardinal.
The main lemma for proving the indestructibility of large cardinals via forc-
ing is the following:
Lemma 8.1 (Lifting Lemma). Let j : M ≺ N . If P ∈ M is a forcing notion,
and G is P-generic, then for any H j(P)-generic such that j ′′ G ⊆ H there exists
j ∗ : M [G] ≺ N [H] such that j ∗ ↾ M = j.
Proof. The key point is that the forcing relation is definable. For any P-name
τ , define j ∗ (τG ) = (j(τ ))H . Then M [G]  ϕ(τG ) iff there exists p ∈ G such
that p  ϕ(τ ). So by elementarity j(p)  ϕ(j(τ )). But j(p) ∈ H, i.e., N [H] 
ϕ((j(τ ))H ), i.e., N [H]  ϕ(j ∗ (τG )). The opposite direction holds considering
¬ϕ.
This gives immediately a result of lifting: Let j : Vλ ≺ Vλ , P ∈ Vcrt(j) . Then
j(P) = P, so G = H in the Lifting Lemma works, as j ′′ G = G. Therefore j
extends to j ∗ : Vλ [G] ≺ Vλ [G]. But this is still not V [G]  I3(λ), because that
is ∃k : V [G]λ ≺ V [G]λ , where V [G]λ = (Vλ )V [G] . The difference is subtle, but
fundamental: Vλ [G] is the set of τG such that τ ∈ Vλ , V [G]λ is the set of τG
with rank of τG less than λ in V [G]. As the rank of the interpretation of a name
is always less or equal than the rank of the name, clearly Vλ [G] ⊆ V [G]λ , but
it is not immediate to see whether the opposite direction is true. This is solved
by the nice names lemma:

49
Lemma 8.2 (Name Rank Lemma). If τ is a P-name, P ∈ Vγ+1 and τG ∈ V [G]β
for any G P-generic, then there is a name σ ∈ Vγ+3·β such that σG = τG for
any G P-generic.
In particular, if P ∈ Vcrt(j) , any element of V [G]λ will be in some V [G]κn ,
and so it will have a name in V3·κn ⊆ Vλ , therefore Vλ [G] = V [G]λ .
This is peculiar to rank-into-rank axioms: the indestructibility proof must
pass two phases: first we have to extend the embedding, then we have to make
sure that the domain of the embedding is exactly what we want.
By the discussion before Proposition 4.10, crt(j) can be as large as we want
below λ, therefore we have the following:
Lemma 8.3. Suppose I3(λ), let P ∈ Vλ be a forcing notion and G be P-generic.
Then V [G]  I3(λ), So I3 is not destructed by “small” forcings.
The Lifting Lemma holds easily also for j : Vλ+1 ≺ Vλ+1 and P ∈ Vcrt(j) , so
there exists j ∗ : Vλ+1 [G] ≺ Vλ+1 [G]. We have to
S prove that Vλ+1 [G] = V [G]λ+1 .
If τG ∈ V [G]λ+1 , then τG ⊆ V [G]λ , so τG = n∈ω (τG ∩ V [G]κn ). As Vλ [G] =
V [G]λ , any τG ∩ V [G]κn has a name in Vλ . The name of a union of sets is the
union of the names of the sets, therefore there exists a name for τG that is the
ω-union of sets in Vλ , so it must be in Vλ+1 .
Again, crt(j) can be unbounded below λ, so:
Lemma 8.4. Suppose I1(λ) (or anything slightly weaker, like I2 or Σ1n ), let
P ∈ Vλ be a forcing notion and G be P-generic. Then V [G]  I1(λ) (or the
corresponding weaker axiom). So I1 is not destructed by “small” forcings.
Finally we consider I0 and P ∈ Vcrt(j) . Again, the Lifting Lemma holds,
so there exists j ∗ : L(Vλ+1 )[G] ≺ L(Vλ+1 )[G] with crt(j ∗ ) < λ. This time
we cannot always prove that L(Vλ+1 )[G] = L(V [G]λ+1 ), but it actually does
not matter: as a constructible set will have a constructible name, we have
L(V [G]λ+1 ) ⊆ L(Vλ+1 )[G]. Also, L(V [G]λ+1 ) is a definable (with parameter λ)
subclass of L(Vλ+1 )[G], therefore
j ∗ ↾ L(V [G]λ+1 ) : L(V [G]λ+1 ) ≺ L(V [G]λ+1 )
and we have I0(λ) in the extension.
Can we say that crt(j) can be unbounded below λ? For this, we have to
prove that a j that witnesses I0 is at least finitely iterable. If j is not weakly
proper, then it is not clear whether this is possible. But if j is weakly proper,
then it is:
Lemma 8.5 (Woodin, Lemma 16 of [41]). Let j : L(Vλ+1 ) ≺ L(Vλ+1 ) with
crt(j) < λ and weakly proper. Then j is finitely iterable.
Proof. Let I be the class of fixed points of ξ (it is a class by the proof of
5.14). For each s ∈ [I]<ω , let Zs = HullL(Vλ+1) (Vλ+1 ∪ {Vλ+1 } ∪ s). Then
hZs : s ∈ [I]<ω i is a directed system. Let Z be its limit. Then Z ≺ L(Vλ+1 ).
For each s ∈ [I]<ω , j ↾ Zs is definable from {j ↾ Vλ , s} ∈ L(Vλ+1 ), therefore
ks = j(j ↾ Zs ) is well-defined. As j(Zs ) = Zs , we have that ks : Zs ≺ Zs . Then
we can define k ∗ : Z ≺ Z, its direct limit. Composing with the collapse of Z, we
have k : L(Vλ+1 ) ≺ L(Vλ+1 ), and k ↾ Vλ = j(j ↾ Vλ ). If k is not weakly proper,
then we consider kU its weakly proper part, and we call it still k.
This proves the existence of j 2 . As j 2 is weakly proper, we can prove by
induction that j n exists and is weakly proper for every n ∈ ω.

50
In particular j n ↾ Vλ = (j ↾ Vλ )n , therefore crt(j n ) = κn , and so:
Lemma 8.6. Suppose I0(λ), let P ∈ Vλ be a forcing notion and G be P-generic.
Then V [G]  I0(λ), So I0 is not destructed by “small” forcings.
The reason why we do not know whether L(Vλ+1 )[G] = L(V [G]λ+1 ) is quite
deep:
Theorem 8.7 (Woodin, Theorem 175 in [41]). Suppose j : L(Vλ+1 ) ≺ L(Vλ+1 )
with crt(j) < λ. Let P ∈ Vλ , G be P-generic and λω 6= (λω )V [G] . Then Vλ+1 ∈
/
L(V [G]λ+1 ), so L(Vλ+1 )[G] 6= L(V [G]λ+1 )6 .
Proof. Let hκi : i ∈ ωi be the critical sequence of j. As P ∈ Vλ , we can suppose
that P ∈ Vκ0 , and then there exists j ∗ : L(V [G]λ+1 ) ≺ L(V [G]λ+1 ) that extends
j. Suppose that Vλ+1 ∈ L(V [G]λ+1 ). Then by Theorem 7.11 in L(V [G]λ+1 ) it
is possible to embed continuously (λω )V [G] in Vλ+1 .
We do some cosmetic changes so that the proof is easier to read: we fix ρn
bijections between Vκn +1 and |Vκn +1 |, rename |Vκn +1 | as κn , and modify the
continuous embedding so that its domain is (Πn∈ω κi )V [G] . So there exists π that
continuously embed (Πn∈ω κi )V [G] to (Πn∈ω κi )V . We prove that (Πn∈ω κi )V [G] ⊆
V , and therefore (λω )V [G] = λω , contradiction. S
Let π be such embedding. We define for every s ∈ n∈ω Πi<n κn a set
Ss ⊆ κlh(s) , Ss ∈ V in such a way:
• 0 ∈ Ss ;
• Fix n = lh(s). Let α ∈ Ss . Then let Pα = {β < κn : ∃p ∈ p p  β =
min{π̇(a)(n) : a ∈ (Πn∈ω κn )v , π(a) ↾ n = s, π̇(a)(n) >Sα}}. Then
|Pα | < κ0 , therefore Pα is bounded in κn . As for any s ∈ n∈ω Πi<n κn
{π(a)(n) : a ∈ (Πn∈ω κn )V [G] , π(a) ↾ n = s} is unbounded in κn , Pα 6= ∅,
so let α′ = max Pα + 1 ∈ Ss .
S
Note that not only Ss ∈ V , but {Ss : s ∈ n∈ω Πi<n κn } ∈ V .
By the construction, (*) for every a ∈ (Πn∈ω κn )V [G] and every α ∈ Sπ(a)↾n ,
there exists b ∈ (Πn∈ω κn )V [G] such that π(a) ↾ n = π(b) ↾ n = s, and α <
π(b)(n) < min Ss \ α.
We define now another continuous embedding, π ′ . For any Ss , let Ss (α) be
the α-th element of Ss .
Let a ∈ (Πn∈ω κn )V [G] . Then we define by induction bn :
• Applying (*) with n = 0, a anything and α = S∅ (a(0)), there exists b0
such that S∅ (a(0)) < π(b0 )(0) < S∅ (a(0) + 1);
• Suppose that bn is already defined. Applying (*) with a = bn and α =
Sπ(bn )↾n (a(n + 1)), there exists bn+1 such that π(bn+1 ) ↾ n = π(bn ) ↾ n
and Sπ(bn )↾n (a(n + 1)) < π(bn+1 )(n + 1) < Sπ(bn )↾n (a(n + 1) + 1).
As π is continuous, the bn ’s are more and more closer. Let b be their limit.
Then we define π ′ (a) = π(b).
6 The proof in [41] is more convoluted, as it proves that V
λ+1 , if in L(V [G]λ+1 ) and U(j)-
representable, is a perfect set, and this brings a contradiction. By 7.11 we immediately have
that Vλ+1 is perfect.

51
Note that, if α and α′ are two consecutive members of Ss , then |{π(b)(n) :
∃a ∈ (Πn∈ω κn )V [G] , π ′ (a) = π(b), π(b) ↾ n = s} ∩ [α, α′ ]| = 1. Therefore π ′ is
well-defined and injective. But then the inverse of π ′ is definable in V , as for
any a ∈ (Πn∈ω κn )V [G] , π ′ (a)(n) = |Sπ′ (a)↾n ∩ a(n)|.
On the other hand, if the forcing is ω-closed in Vλ then we have equality:
Theorem 8.8. Suppose j : L(Vλ+1 ) ≺ L(Vλ+1 ) with crt(j) < λ. Let P ∈ Vλ be
ω-closed in Vλ , i.e., (Vλ )ω = ((Vλ )ω )V [G] , G be P-generic. Then L(Vλ+1 )[G] =
L(V [G]λ+1 ).
Proof. Fix a bijection in V between Vλ and λ, this extends naturally to a bijec-
tion from Vλ+1 and P(λ), so it suffices to define P(λ) inside L(V [G]λ+1 ):

P(λ) = {a ∈ P V [G] (λ) : ∀n ∈ ω a ∩ κn ∈ V }.


S
This is because if a ∩ κn = an ∈ V , then han : n ∈ ωi ∈ V and a = n∈ω an ∈
V.
Corollary 8.9. Suppose j : L(Vλ+1 ) ≺ L(Vλ+1 ) with crt(j) < λ. Let P ∈ Vλ be
a forcing notion, G be P-generic. Then L(Vλ+1 )[G] = L(V [G]λ+1 ) iff (Vλ )ω =
((Vλ )ω )V [G] .
So I3, I2, I1 and I0 are not changed by forcing notions bounded in λ. Also,
trivially, if a forcing notion is λ-closed, then V [G]λ+1 = Vλ+1 , therefore I3, I2,
I1 and I0 are witnessed by the exact same embedding, and are not destructed.
In other words, they are not changed by forcings that are above λ.
This case can give some very nice results:
Theorem 8.10 (Shi, Trang, 2017 [36]). Suppose I0(λ) holds. Then all the
following statements are true in a generic extension where I0(λ) still holds:
• special λ+ -Aronszajn trees exist;
• λ+ -Suslin tree exist;
• there is a very good scale at λ+ ;
• Stationary Reflections fails at λ+ ;
• ♦λ+ :
This is particularly striking if compared with Theorem 7.3.
It remains to see what happens when the forcing has size λ. The first case
is when the forcing is unbounded in λ (the second case will be when the forcing
is exactly at λ). The typical case is when the forcing is a λ-iteration of small
forcings. In such case, the reverse Easton iteration has been proven in the past
to behave very well with large cardinals:
Definition 8.11. Let Pα be a forcing iteration of length α, where α is either
a strong limit cardinal or is equal to ∞, the class of all ordinals. Then Pα is
a reverse Easton iteration if nontrivial forcing is done only at infinite cardinal
stages, direct limits are taken at all inaccessible cardinal limit stages, and inverse
limits are taken at all other limit stages; moreover, Pα is the direct limit of
hPδ : δ < αi if α is regular or ∞, the inverse limit otherwise.

52
Such iteration was introduced by Easton to force different behaviours of the
function κ 7→ 2κ , the power function. There are two known rules for the power
function: if κ < η then 2κ ≤ 2η , and cof(2κ ) > κ for κ regular. Easton proved
that any function that satisfies this two rules can be the power function on the
regular cardinals:
Definition 8.12. Let E : Reg → Card be a class function. Then E is an
Easton function iff:
• α < β → E(α) ≤ E(β);
• cof(E(α)) > α for all α ∈ Reg.
Then Easton proved:
Theorem 8.13 (Easton, 1970 [14]). Let E be an Easton function. Then there
exists a generic extension V [G] of V such that V [G]  ∀κ(κ ∈ Reg → 2κ =
E(κ)).
The forcing used was a reverse Easton iteration. The easiest application is
to force GCH: in that case for any γ < α Qγ forces that 2γ = γ + if γ is a
cardinal, it is trivial otherwise, Pγ+1 = Pγ ∗ Q̇γ and G will the generic filter of
the reverse Easton iteration P∞ . The problem is when one, for example, starts
with a model of GCH and wants to increase everywhere on the regulars the
power function: If we force with Qω 2ω = ω2 and after that we force 2ω1 = ω3
via Cohen forcing, then in the final model 2ω will collapse to ω1 again. The
idea is then that Qδ is defined only on δ closed under E, and Qδ is the Easton
product7 of the forcings that make 2γ = E(γ) for all γ ∈ [δ, δ ∗ ), where δ ∗ is the
smallest cardinal closed under E bigger than δ (this is still in [14]).
So let E be an Easton function, and Pλ that forces 2κ = E(κ) for any regular
κ, with the stages indicated with Qδ for δ < λ. We want to know for which E’s
the forcing Pλ does not destroy I3, . . . , I0.
If for some δ < λ |Qδ | ≥ λ, then there is a α < δ ∗ such that in V [G] 2α ≥ λ,
therefore λ is not strong limit anymore in V [G] and the embedding is destroyed.
Therefore we must have |Qδ | < λ for any δ < λ.
If for some δ < λ we have that j(Pδ ) 6= Pj(δ) , then possibly j(2α ) 6= 2j(α) for
some α < δ, therefore we should ask that j(Pδ ) = Pj(δ) . If E is definable then
this is trivial.
Also note that Qδ is δ-directed closed. This is enough to lift I3, . . . , I0 in
the generic extension:
Theorem 8.14 ([9]). Suppose j witnesses I3(λ) (or I1(λ) . . . ). Let Pλ be a
forcing iteration of length λ that is reverse Easton, directed closed (i.e., for all
δ < λ, Qδ is δ-directed closed), λ-bounded (i.e., for all δ < λ, |Qδ | < λ) and
j-coherent (i.e., for all δ < λ, j(Pδ ) = Pj(δ) ). Then j lifts to any generic
extension via Pλ .
Sketch of proof. The proof is in two stages: lifting and right domain.
First notice that if γ < κn , then |Pγ | < κn , by elementarity, λ-boundedness
and n applications of j. Fixing an n < ω, consider the image of Qκn under j: it
7 It is the product with support the Easton ideal, i.e., the sets that are bounded below

every inaccessible cardinal.

53
is a subset of Qκn+1 of cardinality less than κn+1 , therefore by directed closeness
there is an element Qκn+1 that extends all the j(p) with p ∈ Qκn . Doing the
same for all n and also for Pκn ,κn+1 , one gets a condition of Pλ that, if in a
generic G, implies that j ′′ G ⊆ G. Therefore we can apply the Lifting Lemma.
As for the right domain, since the forcing is directed closed every element in
the generic extension is actually in a generic extension for some Pκn , therefore
the previous proofs still hold.
Theorem 8.14 is just the final step of a series of theorems that goes in that
direction: in [22] Hamkins has proved such theorem for I1 and reverse Easton
directed closed j-coherent iterations such that |Qδ | < 2δ ; in [2] Corazza has
proved it for I3 and reverse Easton directed closed j-coherent iterations such
that |Qδ | < i(δ), where i(δ) is the smallest inaccessible larger than δ; in [17]
Friedman has proved it for I2 (called there ω-superstrong) and GCH.
The hypotheses in Theorem 8.14 are very reasonable, and most of the reverse
Easton forcing usually employed satisfy them. So, for example:
Corollary 8.15. Suppose I3 (or I1, . . . ). Then in a generic extension can hold:
• I3 + GCH;
• I3 + 2κ = κ+++ for any κ regular;
• I3 + V = HOD;
• I3 + ♦ everywhere;
• I3 + every κ supercompact is Laver indestructible (see [35]);
• ...
Finally the last case is when the forcing is changing λ+ . This is the hardest
case, and at the moment the indestructibility results are very scarce. For ex-
ample, Gitik short extenders Prikry forcing construction in [18] can be carried
on under I3 (it needs o(κn ) ≥ (κn )+m for any n, m ∈ ω): such construction
push 2λ to λ+δ+1 for any desired δ < ℵ1 , but does not add bounded subsets of
λ, therefore V [G]λ = Vλ and I3 is not destructed. Yet, this construction for I1
does not work, as Vλ+1 6= V [G]λ+1 .
In [5] Cummings and Foreman, using something more than I2, manage to
prove that I2(λ) is consistent with 2λ = λ++ .
The most successful strategy has been instead to start with I0(λ), and using
some sophisticated tool to prove that, adding a Prikry sequence to the critical
point κ0 , in the extension I1(κ0 ) holds.
Definition 8.16. Let κ be a measurable cardinal, and U a normal measure on
κ. Then P, the Prikry forcing on κ via U , is the set of (s, A) such that s ∈ [κ]<ω ,
A ∈ U and min A > max s.
We say that (s, A) < (t, B) if s ⊒ t, A ⊆ B and for any n ∈ lh(s) \ lh(t),
s(n) ∈ B.
If (s, A) and (t, B) S
are in the generic set G, then, s and t must be compatible.
Therefore by density {s : ∃A (s, A) ∈ G} is an ω-sequence cofinal in κ. So
(cof(κ) = ω)V [G] , but it is a very delicate forcing, as it does not add any bounded
subset of κ.
There is a convenient condition for an ω-sequence cofinal in κ to be generic:

54
Theorem 8.17 (Mathias Condition, or geometric condition). Let κ be a mea-
surable cardinal, P the Prikry forcing on κ via the normal ultrafilter U and let
hαn : n ∈ ωi be a cofinal sequence in κ. Then hαn : n ∈ ωi is generic for P iff
for any A ∈ U the set hαn : n ∈ ωi \ A is finite.
In Theorem 8.5 we noticed that if j witnesses I0(λ) is weakly proper, then
it is finitely iterable, and its iterates are still weakly proper embeddings from
L(Vλ+1 ) to itself. Woodin proved that iterability can go further:
Theorem 8.18 (Woodin, Lemma 21 in [41]). Suppose that j : L(Vλ+1 ) ≺
L(Vλ+1 ) with crt(j) < λ is weakly proper. Then j is iterable.
From the proof of Lemma 6.4 j0,ω (κ0 ) = λ, therefore Mω is different than
L(Vλ+1 ): in it, by elementarity, λ is regular. Let U be the ultrafilter from j.
Then j0,ω (U ) is a normal ultrafilter on λ, therefore λ is measurable in Mω . If
P is the Prikry forcing on κ0 via U , then j0,ω (P) is the Prikry forcing on λ via
j0,ω (U ).
Remark 8.19. If hκn : n ∈ ωi is the critical sequence of j, then hκn : n ∈ ωi
is j0,ω (P)-generic in Mω .
Proof. Note that Mathias’ characterization needs the Axiom of Choice, but by
elementarity (Vλ )Mω  AC, so we can use it. If A ∈ j0,n (U ), then by the
definition of U κn ∈ jn,ω (A), as A ∈ j0,n (U ) iff κn ∈ j n (A) and

κn = jn+1,ω (κn ) ∈ jn+1,ω (j n (A)) = jn,ω (A).

So if A ∈ j0,ω (U ), there exist n ∈ ω and Ā ∈ L(Vλ+1 ) such that A = jn,ω (Ā).


By elementarity Ā ∈ j0,n (U ) and jn,n+i (Ā) ∈ j0,n+i (U ). So for any i ∈ ω,
κn+i ∈ jn+i,ω (jn,n+i (Ā)) = A.
So it make sense to consider M [hκi : i ∈ ωi].
Theorem 8.20 (Generic Absoluteness, Woodin, [41]). Let j : L(Vλ+1 ) ≺
L(Vλ+1 ) with crt(j) be a weakly proper elementary embedding. Let hκn : n ∈ ωi
be its critical sequence, and Mω its ω-iterated model.
Let δ be such that all the sets in Lδ (Vλ+1 ) ∩ Vλ+2 are U(j)-representable.
Then there exists an elementary embedding

π : Lδ (Mω [hκn : n ∈ ωi] ∩ Vλ+1 ) ≺ Lδ (Vλ+1 )

such that π ↾ Vλ+1 is the identity.


Two examples to understand better the theorem:
• As hκn : n ∈ ωi ∈ Vλ+1 , clearly

Mω [hκn : n ∈ ωi] ∩ Vλ+1 = (Vλ+1 )Mω [hκn :n∈ωi] ⊆ Vλ+1 .

Since π ↾ Vλ+1 = id, the Theorem says that Mω [hκn : n ∈ ωi] ∩ Vλ+1 ≺
Vλ+1 .
• If A is definable on Mω [hκn : n ∈ ωi], then π(A) will be the set defined in
the same way on Vλ+1 , and π(A) ∩ Mω [hκn : n ∈ ωi] = A.

55
The proof exploits the fact that in the definition of U(j)-representability
we had j n (U ) = U for the ultrafilters used in a representation, therefore given
a set in Lδ (Vλ+1 ), its representation is also in Mω , in a certain sense is the
set encrypted. When we add the critical sequence to λ, then we can rebuild
something similar to the set (if not the set itself) using such representation.
The Theorem uses the theory of Lδ (Vλ+1 ), as a set, to build an embedding.
This essential tool has an immediate corollary:
Corollary 8.21. Suppose that there exists j : L(Vλ+1 ) ≺ L(Vλ+1 ) with crt(j) =
κ0 < λ. Let P be the Prikry forcing on κ0 . Then there exists G P-generic such
that there exists k : V [G]κ0 +1 ≺ V [G]κ0 +1 .
Proof. We have seen that j ↾ Vλ+1 is definable from j ↾ Vλ , therefore it is in
L1 (Vλ+1 ). Then

π −1 (j) = j ∩ Mω [hκn : n ∈ ωi] : (Vλ+1 )Mω [hκn :n∈ωi] ≺ (Vλ+1 )Mω [hκn :n∈ωi] .

So in Mω there is a condition in j0,ω (P) that forces the existence of a I1(λ)


embedding. By elementarity, in V there is a condition on P that forces the
existence of a I1(κ0 ) embedding.
This corollary, coupled with Theorem 8.14, can expand the possibilities of
consistency results: for example one can force 2κ0 > κ+0 with the reverse Easton
forcing, and then force with Prikry to have I1(κ0 ). Prikry will not change
2 κ0 > κ+ λ
0 , and therefore we have a model for I1(λ) + 2 > λ .
+

Theorem 8.22. Suppose there exists j : L(Vλ+1 ) ≺ L(Vλ+1 ) with crt(j) < λ.
Then for every Easton function E such that E ↾ λ is definable over Vλ , there is a
generic extension V [G] of V that satisfies ∃γ < λ ∃k : Vγ+1 ≺ Vγ+1 ∧2γ = E(γ).
In particular this proves that it is consistent with I1(λ) that there are no
strongly compact or supercompact cardinals below λ, since λ is singular and
Solovay’s Theorem says that above a strongly compact or a supercompact car-
dinal we have 2λ = λ+ on singular cardinals. Therefore this is one of the cases
where “stronger” does not mean “larger”.
The first way to improve such results is to note that Theorem 8.20 can express
more than just I1, therefore the same proof works for stronger hypotheses (for
example ∃j : Lλ (Vλ+1 ) ≺ Lλ (Vλ+1 )), with the only bound given by which sets
are U(j)-representable, making Question 7.20 even more relevant.
Another way is to notice that in the proof of Theorem 8.20 it is not necessary
to limit ourselves to Prikry forcing and to hκn : n ∈ ωi: the important thing is
that there is a G ∈ V j0,ω (P)-generic and there is an ω-sequence cofinal in λ in
Mω [G]. If G ∈/ V then there are easy counterexamples: if CH holds in V , then
it holds also in Mω , and a forcing that adds a Cohen real to Mω would make
M [G]
Vλ+1ω not elementary in Vλ+1 (not even contained, in fact).
There are numerous variations on Prikry forcing that add ω-sequences to a
large cardinal, and as λ is a very large cardinal in Mω (see 4.10) they are usually
applicable in this context. For example, it is clear in Theorem 8.22 that the
reverse Easton forcing forces 2η = E(η) for all the regular cardinals η < γ, not
only for γ, therefore with the construction of Theorem 8.22, based on classical
Prikry forcing, γ is not the first cardinal on which GCH fails. Being the first one
is a stronger property, that is forced usually with Gitik-Magidor long extender

56
Prikry forcing (see [?]). If P is therefore the Gitik-Magidor long extender Prikry
forcing on κ0 and it could be possible to find a G ∈ V j0,ω (P)-generic in Mω ,
then by Generic Absoluteness we would have a generic extension in which I1(κ0 )
+ 2 κ0 > κ+0 and GCH below κ0 .
To check if this and other forcing notions work with Generic Absoluteness
there is the need of a criterion that, if satisfied, implies that there is a G ∈ V ,
j0,ω (P)-generic in Mω .
Definition 8.23. Let λ be an infinite cardinal. A partially ordered set P is
λ-good if it adds no bounded subsets of λ and for every P-generic G and every
A ∈ V [G], A ⊂ Ord, |A|S < λ, there is a non-⊂-decreasing ω-sequence hAi : i <
ωi ⊆ V such that A = i∈ω Ai .
Proposition 8.24 (Shi, Proposition 3.20 in [35]). Let j : L(Vλ+1 ) ≺ L(Vλ+1 )
with crt(j) = κ0 < λ and let P ∈ Vλ be a κ0 -good forcing. Then if (Mω , j0,ω ) is
the ω-iterate of j, there is a G ∈ V j0,ω (P)-generic on Mω .
The original definition of λ-goodness involved families of dense sets, and this
is a more convenient way to prove that various Prikry forcings are λ-good. The
following is a sufficient condition:
Definition 8.25 ([10]). Let λ be an infinite cardinal. Let P be a forcing notion
equipped with a length measure of the conditions l : P → ω such that l(1P ) = 0
and for p, q ∈ P, if p ≤ q then l(p) ≥ l(q) (for example if P is the Prikry forcing,
then l(s, A) = lh(s)).
We say that P is λ-geometric if it does not add bounded subsets of λ and for
any α < λ, any hDβ : β < αi collection of open dense sets and any p ∈ P there
exists a condition q ≤ p such that whenever a filter contains q and meets all the
dense open sets En = {p : l(p) > n}, it also meets all the Dβ ’s.
Remark 8.26. Let λ be an infinite cardinal. Let P be a λ-geometric forcing
notion. Then P is λ-good.
Proof. Let G be P-generic. Let A ⊆ Ord, |A| = µ < λ, A ∈ V [G]. Let
f : µ → Ord such that ran(f ) = A, f ∈ V [G]. For any α < µ, let Dα = {p ∈
P : ∃β p  f˙(α) = β}, the set of the conditions that decide the α-th element
of A. By λ-geometricity, the set of q’s such that every filter that contains q
and intersects En intersects also all Dα ’s, is dense in P, therefore there exists
q0 ∈ G with such property. Define by induction qn+1 that extends qn such that
qn+1 ∈ G ∩ En . Let Bn = {α <Sµ : qn ∈ Dα } ∈ V be the set of α’s such that qn
decides f (α). Then since F = n∈ω Fqn , by λ-geometricity, intersects all Dα ’s
S by qn ), for any α < µ there exists n ∈ ω such that
(Fqn is the filter generated
qn ∈ Dα , therefore µ = n∈ω Bn . Define

An = {β : ∃α ∈ Bn qn  f˙(α) = β} ∈ V.

If β ∈ A, then there exists α < µ such that f (α) = µ, and there exists n ∈ ω
such that α ∈ Bn , i.e., qn decides f (α).SBut since qn ∈ G, it must be that
qn  f˙(α) = β, so β ∈ An , therefore A = n∈ω An .
The key point is that many variations of Prikry forcing satisfy a variation of
the Prikry condition: Let p ≤∗ q be p ≤ q and l(p) = l(q). Then such variation
says that for any dense set D and every p ∈ P there is a q ≤∗ p, such that

57
for any r ≤ q sith l(r) = l(q) + n, r ∈ D. In other words every condition can
be “changed” so that any extension above a certain length is in the dense set.
This condition appears frequently in literature, for example in [30] for Prikry
forcing, or [31] for Gitik-Magidor long extender Prikry forcing. In [11] there
are other examples, and in both [36] and [11] it is exploited to prove the λ-
goodness of Gitik-Magidor long extender Prikry forcing, in different ways. If
≤∗ is sufficiently closed, it is easy to see that with repeated uses of this variation
of Prikry condition one can find a q that witnesses λ-geometricity, and therefore
λ-goodness.
It is possible also to prove it directly:
Remark 8.27 ([10]). Let j : L(Vλ+1 ) ≺ L(Vλ+1 ) with crt(j) = κ0 < λ and let
P ∈ Vλ be a κ0 -geometric forcing. Then if (Mω , j0,ω ) is the ω-iterate of j, there
is a G ∈ V j0,ω (P)-generic on Mω .
Proof. Note that in V there are only λ open dense sets of j0,ω (P): as P ∈ Vλ there
exists n ∈ ω such that P ∈ Vκn , so j0,ω (P) ∈ Mω ∩ Vj0,ω (κn ) . But Mω ∩ Vj0,ω (λ)
is the range of all jn,ω ↾ Vλ , therefore |j0,ω (P)| < |Mω ∩ Vj0,ω (λ) | = |Vλ | = λ.
Let hDα : α < λi be an enumeration of the dense sets of j0,ω (P) in V . For
every n ∈ ω, fix qn that witnesses λ-geometricity for hDα : α < κn i. Then for
′ ′
every m there exists a qn,m
S < qn such that qn,m ∈ Em . Let H be the filter
F
n,m∈ω qn,m
′ , with F q the filter generated by q. Then H ∈ V is generic.
Therefore, thanks to the work that in literature has already been done with
the structure of Prikry-like forcing, for many variations of Prikry forcing it is
possible to prove Generic Absoluteness. Interestingly, for many applications
such variations need a forcing preparation that is actually a reverse Easton
iteration, therefore Theorem 8.22 is necessary. In [10] are collected some results:
Theorem 8.28. Suppose I0(λ). Then the following are consistent:
• I1(γ) + 2γ > γ + + ∀κ < γ 2κ = κ+ (see [19], preparation forcing to force
GCH, then Gitik-Magidor long extender Prikry forcing);
• I1(γ) + Tree property at γ + (see [33], preparation forcings that make
supercompact cardinals Laver indestructible and add many subsets to all
regular cardinals, then diagonal supercompact Prikry forcing);
• I1(γ) + Tree property at γ ++ (see [12], preparation forcing is reverse
Easton iteration of Sacks forcings, then Prikry forcing);
• ...

9 Dissimilarities with ADL(R)


While there are many similarities between I0 and ADL(R) , not everything re-
ally matches. The first thing that stands out is that while L(R)  ADL(R) ,
L(Vλ+1 ) 2 I0(λ). But the big problem is Theorem 8.6: the fact that many
structural properties below λ are independent from I0 in fact implies that I0
cannot possibly prove them, and these properties can be the similarities we are
interested in. In the ADL(R) case this situation is avoided as there are no forc-
ings below ω. For example, we have proven that λ+ is measurable in L(Vλ+1 )

58
under I0, just like ω1 was measurable in L(R) under AD. But in the L(R) case
we have that the club filter itself is an ultrafilter, while Theorem 7.2 stops short
and proves only that the club filter on a stationary set is measurable. So the
question is: how close can we get to the original case?
+ + +
As λ > ω, for any α < λ regular Sαλ is stationary, and for α 6= β Sαλ ∩Sβλ =
∅, therefore it is not possible to have the club filter to be an ultrafilter. The
best outcome would be then to have the club filter to be an ultrafilter on every
+
Sαλ ; this is called Ultrafilter Axiom at λ. But this is consistently false because
of Theorem 8.6:
Proposition 9.1. Let j : L(Vλ+1 ) ≺ L(Vλ+1 ) with crt(j) < λ, let ω < γ < λ
regular. Then in a generic extension the following is true: I0(λ) still holds and
+
in L(Vλ+1 ) there are S1 and S2 disjoint stationary sets such that S1 ∪ S2 = Sγλ .
Proof. First of all, notice that λ+ is coded by subsets of λ, so it can be considered
a subset of Vλ+1 . As Vλ+1 is closed by λ-sequences, the cofinality of a cardinal
less than λ+ is the same in V and in L(Vλ+1 ) (because is witnessed by an element
+ +
of Vλ+1 ), therefore (Sηλ )V = (Sηλ )L(Vλ+1 ) for any η regular. Let P ∈ L(Vλ+1 )
be Coll(γ + , γ). Then by 8.6 I0(λ) holds in V [G]. As γ > ω, P is ω-closed. Let
+ +
G be P generic. Consider (Sγλ+ )V and (Sγλ )V , both in L(Vλ+1 ) and therefore in
+ +
L(Vλ+1 )[G]. Since P does not add unbounded sets in λ+ , (Sγλ+ )V and (Sγλ )V are
+ +
still stationary in L(Vλ+1 )[G], are both subsets of (Sγλ )V [G] = (Sγλ )L(Vλ+1 )[G]
and of course are disjoint. Since P is ω-closed, by 8.8 L(Vλ+1 )[G] = L(V [G]λ+1 ),
+ +
so (Sγλ+ )V , (Sγλ )V ∈ L(V [G]λ+1 ) are S1 and S2 .

Note that if γ = ω then the proof breaks down: then L(Vλ+1 )[G] 6= L(V [G]λ+1 )
and it is not clear why S1 and S2 would be in L(V [G]λ+1 ). The following prob-
lems are still open:
Question 9.2. Is the Ultrafilter Axiom consistent with I0?
Question 9.3. Is it consistent with I0 to have the club filter an ultrafilter on
cofinality ω? Could it be a consequence of I0?
Towards a solution of the second problem, there are the following results:
Theorem 9.4 (Woodin, Theorem 176 in [41]). Suppose I0(λ). Then there is
+
no partition of Sωλ into two stationary sets such that all the sets definable from
them are U(j)-representable8 .

Theorem 9.5 (Cramer, Corollary 4.6 in [3]). Suppose there exists j : Lω (Vλ+1 , Vλ+1 ) ≺
♯ +
λ
Lω (Vλ+1 , Vλ+1 ). Then there is no partition of Sω into two stationary (in V )
sets such that S1 , S2 ∈ L(Vλ+1 ).
Note that if S is stationary in V , then it is also stationary in L(Vλ+1 ), but
not necessarily viceversa, therefore it is still open whether there can be two
8 This rather clumsy statement comes from the fact that the proof uses the fact that some

ZS defined from the stationary set S is U(j)-representable. If all the sets in some Lδ (Vλ+1 ),
with δ limit, are U(j)-representable, this implies that we cannot find two disjoint stationary
sets in Lδ (Vλ+1 ). If all the sets are U(j)-representable, this means that the club filter is an
ultrafilter on cofinality ω.

59
+
disjoint subsets of Sωλ that are in L(Vλ+1 ), stationary there, but not stationary
in V .
Hypotheses like in Theorem 9.5 are investigated further in Section 10. It is
proven using inverse limits.
Towards a solution for the first problem, we have a weakening of the Ultra-
+
filter Axiom. Theorem 7.2 says that we can split Sγλ in hSα : α < ηγ i disjoint
+
stationary sets for any γ < λ regular. The Ultrafilter Axiom says that ηγ = 1
for any γ < λ+ regular. The Weak Ultrafilter Axiom states that ηγ ≤ γ + for
any γ < λ+ regular.
Theorem 9.6 (Woodin, Theorem 16 in [42]). Suppose I0(λ). Then in a generic
extension I0(λ) + the Weak Ultrafilter Axiom hold.
Another problematic point is the Wadge hierarchy. Wadge’s Lemma states
that under AD for any two subsets A, B of R either there is a continuous function
f with A = f −1 [B] or there is one with B = f −1 [R \ A]. Therefore any two
subsets of R are reducible one to another. Under I0 this is consistently false:
Theorem 9.7 (Woodin, Theorem 174 in [41]). Suppose I0(λ). Suppose c is
a generic Cohen real of V . Then there exist X, Y ∈ V [c]λ+2 such that X ∈
/
Lω (Y, V [c]λ+1 ) and Y ∈
/ Lω (X, V [c]λ+1 ).
As by 8.6 in L(V [c]λ+1 ) I0(λ) holds, this proves the consistency of I0 with a
negation of Wadge Lemma.
Another aspect one can consider is partition combinatorics. In L(R), under
9
AD, for all α < ω1 , ω1 → (ω1 )α
ω . A direct generalization always fails:

Remark 9.8 (ZF+DC). Suppose there exists an ω1 -sequence of distinct reals.


Then there is no κ such that κ → (κ)ω
2 .
1

So the best attempt could only be λ+ → (λ+ )ω λ . It is still an open question


even whether κ → (κ)ω 2 is consistent at all. In [41] Lemma 212 proves that a
universal strong version of λ+ → (λ+ )ωλ is consistently false with I0.
But there is a partial result:
Proposition 9.9 (Lemma 180 in [41]). Suppose I0(λ). Then for all α < λ,
L(Vλ+1 )  λ+ → (λ+ )α +
(λ,<λ) , i.e., for all π : {σ ⊆ λ : ot(σ) = α} → λ there
exists H ⊆ λ+ such that |{π(σ) : σ ⊆ H, ot(σ) = α}| < λ.
Finally, we consider the measure on Turing degrees. A standard result of
AD is that the filter U = {A : A contains a cone} on sets of Turing degrees
is an ultrafilter (Martin), i.e., every set of degrees either contains or is disjoint
from a cone. This result is usually called Turing Determinacy, and it is at the
base of many results, like Silver Dichotomy for every set in L(R) under AD. In
the context of I0 Turing degrees can be substituted with Zermelo degrees: given
a ⊆ λ, let M (a) be the smallest model of Zermelo set theory that contains a.
We say that a is equivalent with b if M (a) = M (b), and a is reducible to b if
M (a) ⊆ M (b). Then a Zermelo degree is a class of such equivalence relation.
Unfortunately Zermelo Degree Determinacy is consistently false under I0:
9 Recall that κ → (κ)α
β is that if π : {σ ⊆ κ : ot(σ) = α} → β then there is a set H ⊆ κ of
cardinality κ homogeneous for π, i.e., |{π(σ) : σ ⊆ H, ot(σ) = α}| = 1.

60
Theorem 9.10 (Shi, 2015, [35]). Suppose I0(λ) and that all the λ-supercompact
cardinals under λ are Laver indestructible. Then there is a set of Zermelo
degrees in L(Vλ+1 ) that neither contains nor is disjoint from a cone.
This result is significantly more problematic respect the previous results:
in 9.1 and Lemma 212 in [41] we had that small changes would destroy that
ADL(R) -like properties, but in this case, modulo a reasonable indestructibility
preparation, it seems that it is intrinsic from I0, and for deep reasons. But
there is one ambiguity still to settle: for now, there is no proof that there
are λ singular cardinals of cofinality ω such that in L(Vλ+1 ) Zermelo Degree
Determinacy holds, so maybe its failure is not something peculiar to I0, but a
general property of such λ’s, just like it is not possible to have a club filter an
ultrafilter everywhere or the full partition property.
Question 9.11. Is there a λ such that Zermelo Degree Determinacy holds in
L(Vλ+1 )?

10 Icarus sets
Is it possible to go beyond I0? The question is very interesting: if I0 holds,
and since Reinhardt is inconsistent in ZFC, there should be something in be-
tween, or maybe even a hierarchy between the two, where at a certain point the
inconsistency comes up, so it is worth to investigate exactly where it is. But
the question is not only speculative: results like 8.22 are reflection results, from
I0 to I1, so to find combinatorial properties that are independent from I0 one
should have a starting point in a higher ground.
It seems that there is not much space above I0: already an elementary
embedding from L(Vλ+2 ) to itself is inconsistent. We consider then models that
are strictly between the two, i.e., L(Vλ+1 ) ⊂ M ⊂ L(Vλ+2 ). They should be
models of ZF, so something like L(N ), with Vλ+1 ⊂ N ⊂ Vλ+2 , i.e., models
built from collections of subsets of Vλ+1 . In similarity with ADL(R) , the most
important of such models are of the form L(X, Vλ+1 ), with X ⊂ Vλ+1 :
Definition 10.1. A set X ⊆ Vλ+1 is an Icarus set iff there is an elementary
embedding j : L(X, Vλ+1 ) ≺ L(X, Vλ+1 ) with crt(j) < λ.
The name comes from the fact that we are trying to go as high as possible
before reaching an inconsistency. Trivially, if I0(λ) holds, Vλ+1 (and in fact
every set in L(Vλ+1 ) ∩ Vλ+2 ) is Icarus. On the other hand, a well-ordering of
Vλ+1 cannot be Icarus, because of Kunen’s theorem.
In this kind of models many structural properties of L(Vλ+1 ) still hold:
Definition 10.2. Let X ⊆ Vλ+1 . Then in L(X, Vλ+1 ) DCλ holds and Θ =
L(X,V )
ΘVλ+1 λ+1 is regular.
The structure of such embeddings is also very similar to the I0 ones:
Theorem 10.3 (Woodin, Lemma 5 in [41]). Let X ⊆ Vλ+1 be an Icarus set.
For every j : L(X, Vλ+1 ) ≺ L(X, Vλ+1 ) there exist a L(X, Vλ+1 )-ultrafilter U
in Vλ+1 and jU , kU : L(X, Vλ+1 ) ≺ L(X, Vλ+1 ) such that jU is the elementary
embedding from U , j = jU ◦ kU and j ↾ LΘ (X, Vλ+1 ) = jU ↾ LΘ (X, Vλ+1 ).

61
Proof. The proof is the same as in 5.12. The only delicate point is to prove that
the collapse of the ultrapower is indeed L(X, Vλ+1 ), as by elementarity Z ≡
L(j(X), Vλ+1 ). But j(X) ∈ L(X, Vλ+1 ), and X = {a ∈ Vλ+1 : j(a) ∈ j(X)},
therefore L(j(X), Vλ+1 ) = L(X, Vλ+1 ).
There is one difference: in the I0 case we had that a weakly proper embedding
was nicely amenable, and for every a ∈ L(Vλ+1 ) if we defined a0 = a and
an+1 = j(an ), then han : n ∈ ωi ∈ L(Vλ+1 ). This is because everything is
definable from Vλ+1 and fixed points, and Vλ+1 is morally closed under ω-
sequences. But in the Icarus case it is not clear whether it is necessary that
hXn : n ∈ ωi ∈ L(X, Vλ+1 ).
Definition 10.4 (Woodin, [41]). Let X ⊆ Vλ+1 be an Icarus set, and let j :
L(X, Vλ+1 ) ≺ L(X, Vλ+1 ) with crt(j) < λ. Then:
• j is weakly proper iff j = jU ;
• j is proper iff it is weakly proper and hXn : n ∈ ωi ∈ L(X, Vλ+1 ), with
X0 = X and Xn+1 = j(Xn ).
In the particular case of X = Vλ+1 , then, weakly proper and proper are the
same concept.
One could ask whether it is a good definition, as there can be a different
Z ⊆ Vλ+1 such that L(X, Vλ+1 ) = L(Z, Vλ+1 ). For this, we have to study the
fixed points of j under Θ:
Proposition 10.5. Let X ⊆ Vλ+1 be an Icarus set and let j : L(X, Vλ+1 ) ≺
L(X, Vλ+1 ) with crt(j) < λ. Then hXn : n ∈ ωi ∈ L(X, Vλ+1 ) iff the fixed points
of j are cofinal in Θ.
Proof. Suppose that hXn : n ∈ ωi = Y ∈ L(X, Vλ+1 ). We can suppose that j
is weakly proper. Then the proof is similar to the proof of Corollary 5.15: In
this case every ordinal β < Θ is definable from some elements of I ∪ Vλ+1 ∪
{X}. As Y ∈ L(X, Vλ+1 ), L(Y, Vλ+1 ) = L(X, Vλ+1 ), and j(Lω (Y, Vλ+1 )) =
Lω (Y, Vλ+1 ). Let α0 < Θ, and suppose that α0 is definable from {i0 , . . . , in } ∪
Lω (Y, Vλ+1 ). Let α be the supremum of the ordinals that are definable from
{i0 , . . . , in }∪Lω (Y, Vλ+1 ). Since Θ is regular, α < Θ, and since j(Lω (Y, Vλ+1 )) =
Lω (Y, Vλ+1 ), j(α) = α > α0 .
Now suppose that the fixed points of j are cofinal in Θ. Let j(X) ∈
Lγ (X, Vλ+1 ) and α > γ such that j(α) = α. Then j(Lα (X, Vλ+1 )) = Lα (X, Vλ+1 )
and Lα (X, Vλ+1 ) is closed by ω-sequences, therefore hj(X), j 2 (X), . . . i ∈ Lα (Vλ+1 ).

Proposition 10.6 (Woodin, Lemma 6 of [41]). Let X ⊆ Vλ+1 and j : L(X, Vλ+1 ) ≺
L(X, Vλ+1 ) with crt(j) < λ. Then if hXn : n ∈ ωi ∈ L(X, Vλ+1 ), for all
Y ∈ L(X, Vλ+1 ) ∩ Vλ+2 , hYn : n ∈ ωi ∈ L(X, Vλ+1 ), with Y0 = Y and
Yn+1 = j(Yn ).
Proof. Again, let Y ∈ Lγ (X, Vλ+1 ) and α > γ such that j(α) = α. Then
j(Lα (X, Vλ+1 )) = Lα (X, Vλ+1 ) and Lα (X, Vλ+1 ) is closed by ω-sequences, there-
fore hYn : n ∈ ωi ∈ Lα (Vλ+1 ).
Not surprisingly, given that being proper implies some degree of amenability,
properness is bound to iterability:

62
Theorem 10.7 (Woodin, Lemma 16 and Lemma 21 in [41]). Let X ⊆ Vλ+1
and j : L(X, Vλ+1 ) ≺ L(X, Vλ+1 ) with crt(j) < λ. If j is weakly proper, then it
is finitely iterable. If j is proper, then it is iterable.
The Coding Lemma holds for any embedding, but the equivalent of 7.8 is
proven in Lemma 207 of [41] only for the case where the embedding is proper.
It is not known whether properness is necessary. In fact it is not even known
whether it is a concept that is distinct from weak properness:
Question 10.8. Is there an Icarus set X ⊆ Vλ+1 and an elementary embedding
j : L(X, Vλ+1 ) ≺ L(X, Vλ+1 ) that is weakly proper but not proper?
We have seen that if we limit ourselves to I0 the answer is negative. Towards
a solution for this problem in [7] and [8] it is proven that there are N1 and N2 ,
Vλ+1 ⊆ N1 , N2 ⊆ Vλ+2 , such that there are embeddings weakly proper but
non proper10 from L(N1 ) to itself, and such that all weakly proper embeddings
from L(N2 ) to itself are not proper. But it is also proven that there cannot be
X ⊆ Vλ+1 such that L(N1 ) = L(X, Vλ+1 ) or L(N2 ) = L(X, Vλ+1 ).
The existence of an Icarus set outside L(Vλ+1 ) ∩ Vλ+2 is not automatically
an axiom strictly stronger than I0. We end this section introducing canonical
Icarus sets, whose existence is strictly stronger than I0, and that form a hierarchy
above it.
Definition 10.9. Let A be a transitive set. Define L+A as LST, the language of
set theory, extended with the constants {C} ∪ {ca }a∈A ∪ {di }i∈ω .
Let X ⊆ Vλ+1 . Suppose that in L(X, Vλ+1 ) there is a class of indiscernibles
for formulas with parameters in {X} ∪ Vλ+1 . Then X ♯ is the theory in L+ A of

(L(X, Vλ+1 ), {X} ∪ Vλ+1 , {a}a∈{X}∪Vλ+1 , {di }i∈ω ),

where di is the i-th indiscernible.


We can consider therefore (Vλ+1 )♯ . Contrary to the 0♯ case, we do not know
whether an I0 embedding implies the existence of (Vλ+1 )♯ : it is an inner model
question, see Remark 28 in [41]. On the other hand, if (Vλ+1 )♯ exists, with a
shift of indiscernibles there is surely an embedding k : L(Vλ+1 ) ≺ L(Vλ+1 ), but
it is not an I0 embedding, as crt(k) > Θ.
So we suppose that (Vλ+1 )♯ exists.
L(V )
Lemma 10.10. Suppose that (Vλ+1 )♯ exists. Then cof(ΘVλ+1λ+1 ) = ω.

Proof. By the theory of the sharp, if (Vλ+1 )♯ exists every element of L(Vλ+1 )
is defined from a finite subset of Vλ+1 and finite indiscernibles. Let αn be the
supremum of the ordinals definable from Vλ+1 and the first n indiscernibles.
L(V )
Then the supremum of the αn ’s is ΘVλ+1λ+1 .

Therefore, if (Vλ+1 )♯ exists, L(Vλ+1 ) is very far from V (cfr. with Theorem
7.8).
What if (Vλ+1 )♯ is an Icarus set? Note then that I0 is the same as ∃j :
(Vλ+1 , (Vλ+1 )♯ ) ≺ (Vλ+1 , (Vλ+1 )♯ ), therefore not only (Vλ+1 )♯ is Icarus is strictly
stronger then I0(λ), but any I0-embedding must be definable in L((Vλ+1 )♯ , Vλ+1 ).
10 We say that j : L(N ) ≺ L(N ) is proper if for any X ∈ N , hXn : n ∈ ωi ∈ L(N ).

63
By the same proof of 6.37, therefore, (Vλ+1 )♯ is Icarus strongly implies I0(λ)
(in fact, a j : Lω ((Vλ+1 )♯ , Vλ+1 ) ≺ Lω ((Vλ+1 )♯ , Vλ+1 ) suffices to carry on the
construction of the inverse limits).
If such construction is carefully analyzed, one can realize that the λ̄ such
that I0(λ̄) holds constructed in this way has more properties, as it is limit of
critical points of embeddings:
Theorem 10.11 (Shi, Trang, 2017 [36]). Suppose that (Vλ+1 )♯ is Icarus and let
λ̄ < λ be such that I0(λ̄) holds because of the inverse limit construction. Then λ̄
is limit of λ̄+ -supercompact cardinals. Therefore there is no λ̄+ -Aronszajn tree,
there is no good scale at λ̄ and Stationary Reflection at λ̄+ is true.
This is interesting if compared to Theorem 8.10.
The following is the key to the analysis of Icarus sets:
Theorem 10.12 (Woodin, Lemma 213 in [41]). Suppose that X ⊆ Vλ+1 is an
Icarus set, and let Y ∈ L(X, Vλ+1 ) ∩ Vλ+2 such that ΘL(Y,Vλ+1) < ΘL(X,Vλ+1 ) .
Then Y ♯ exists and Y ♯ ∈ L(X, Vλ+1 ).
So let X ⊆ Vλ+1 be an Icarus set such that ΘL(X,Vλ+1 ) > ΘL(Vλ+1 ) . Then
(Vλ+1 )♯ exists and, since it is definable, it is an Icarus set. Moreover ΘL(X,Vλ+1 ) ≥

ΘL((Vλ+1 ) ,Vλ+1 ) . In this sense (Vλ+1 )♯ is a “canonical” Icarus set. If X ⊆ Vλ+1

is any Icarus set such that ΘL(X, Vλ+1 ) > ΘL((Vλ+1 ) ,Vλ+1 ) , then (Vλ+1 )♯♯ exists,
♯♯
it is an Icarus set and ΘL(X,Vλ+1 ) ≥ ΘL((Vλ+1 ) ,Vλ+1 ) .
In other words, if X ⊆ Vλ+1 is an Icarus set then the largeness of ΘL(X,Vλ+1 )
“measures” how strong it actually is, as an axiom, and the Θ’s of the models
♯ ♯♯
built from the sharps (therefore ΘL((Vλ+1 ) ,Vλ+1 , ΘL((Vλ+1 ) ,Vλ+1 , . . . ) are the
reference for such measure. Also, the construction for strong implication still
holds. Therefore we have a new hierarchy:
Proposition 10.13. Let λ be a cardinal. Then Vλ+1 is Icarus is strongly im-
plied by (Vλ+1 )♯ is Icarus, that is strongly implied by (Vλ+1 )♯♯ is Icarus, etc.
In other words, (Vλ+1 )(n+1)♯ is Icarus strongly implies that (Vλ+1 )n♯ is Icarus.
Moreover, if X ⊆ Vλ+1 is Icarus and there exists n ∈ ω such that ΘL(X,Vλ+1 ) <
n♯ m♯
ΘL((Vλ+1 ) ,Vλ+1 ) , then there exists an m < n such that ΘL(X,Vλ+1 ) = ΘL((Vλ+1 ) ,Vλ+1 )
and for any s ≤ m (Vλ+1 )m♯ is Icarus.
Pushing this result above S
the first ω sharps is more difficult. One can try to
define Yα+1 = Yα♯ and Yγ = α<γ Yα for γ limit, but at a certain point there
would be a γ limit such that Yγ is not a subset of Vλ+1 , so then (Yγ )♯ is still
not a subset of Vλ+1 , and so on. The way to do it is to enlarge our analysis to
L(N ) with Vλ+1 ⊂ N ⊂ Vλ+2 , and if necessary at the successor of a limit stage
adding just less of N ♯ so that everything is codeable with a subset of Vλ+1 . It
is a complex endeavour, that takes all Section 4 of [41].

11 Further Developments
The main line of research right now is finding an answer for Question 7.20. The
research is ongoing and very promising: if the answer is yes, then not only many
already known results will get unlocked (for example the Ultrafilter Axiom at
ω or the full power of Generic Absoluteness), but it would give an elegant tree

64
representation to all subsets of Vλ+1 . The potential would be huge, just like
what happened for ADL(R) .
Another approach would be to investigate the inner model theory of I0. Here
the main result is this:
Theorem 11.1 (Woodin, 2011, [40], [43]). Let δ be an extendible cardinal.
Assume that N is a weak extender model for δ supercompact and γ > δ is a
cardinal in N . Let j : H(γ + )N → H(j(γ)+ )N be an elementary embedding with
δ ≤ crt(j) and j 6= id. Then j ∈ N .
In other words, if there is a (reasonable) canonical inner model for supercom-
pactness and I0 holds, then I0 must hold in the inner model. There is no hope
therefore to reach results of consistency equivalence to I0 via inner models, and
for this reason Woodin conceived Ultimate-L, a canonical inner model for all
large cardinals. If such hypothesis will prove solid, then we would have an ac-
tual canonical model, a preferred V , to work with. This would overcome all the
problems in the Section 9, and moreover it would be interesting to understand
the role of I0 in this setting.
Question 11.2. Is it true that Ultimate-L  I0(λ) iff Ultimate-L  L(Vλ+1 ) 2
AC?
One can ask if there are other axioms above I0, still in ZFC. The most
natural approach would be to add to L(Vλ+1 ) the measures that define j, just
like for L[µ], but this is still open:
Question 11.3. Let j : L(Vλ+1 ) ≺ L(Vλ+1 ) weakly proper, and Uj that defines
it. Is V 6= L(Vλ+1 )[Uj ]?
Finally, there is the Reinhardt cardinal approach, i.e., large cardinals in ZF
without AC. There is a whole hierarchy above, like super-Reinhardt cardinals
and Berkeley cardinals, and it goes frontally against the Ultimate-L approach.
Yet, the situation about Reinhardt cardinals is similar to the I0 one: very
few published results (for example [?]), and a lot of underground activity, via
lecture notes, seminar slides, Wikipedia pages, MathOverflow questions, . . . .
The possible intersections with inner model theory, though, promise to make
this topic a catalyst of set theorists’ attention in the near future, and the hope is
that this will bring more publications, and maybe even a survey, of this exciting
topic.
Acknowledgments. The paper was written under the Italian program “Rita
Levi Montalcini 2013”, but it is a collection of notes, thoughts, sketches and
ideas that started in the Summer Semester of 2008 at Berkeley, propelled by a
course on “Beyond I0” by Woodin. With nine years in the making it is therefore
a hard task to thank all the people that helped me writing this paper, as almost
all the people I interacted with in my academic career has been an inspiration
for this, directly or indirectly. Knowing that this is just a small percentage of
the whole picture, I would like to thank first of all Hugh Woodin, the demiurge
of the I0 world, that welcomed me in Berkeley when I was still a student and
continued to be a creative influence during the years; Alessandro Andretta, for
his steady support in all this years, and because of his (decisive) insistence for
this survey to be written. I would also like to thank Sy Friedman and Liuzhen
Wu, that helped to push my research in an unexpected direction, and Scott

65
Cramer and Xianghui Shi, for the many, many discussions on I0 and similars.
Special thanks also for Luca Motto Ros, that helped me in shaping this paper.

References
[1] P. Corazza, The wholeness axiom and Laver sequences. Annals of Pure and
Applied Logic 105 (2000), 157–260.
[2] P. Corazza, Lifting elementary embeddings j : Vλ → Vλ . Archive for Mathe-
matical Logic 46 (2007), 61–72.
[3] S. Cramer, Inverse limit reflection and the structure of L(Vλ+1 ). Journal of
Mathematical Logic 15 (2015), 1–38.
[4] S. Cramer, Implications of very large cardinals. In Foundations of Mathemat-
ics, Contemporary Mathematics 690 (2017) (also on the author’s website).
[5] J. Cummings, M. Foreman, Diagonal Prikry extensions. Journal of Symbolic
Logic 75 (2010), 1383–1402.
[6] P. Dehornoy, Braid groups and left distributive operations. Transactions of
the American Mathematical Society 345 (1994), 115–151.
[7] V. Dimonte, Totally non-proper ordinals beyond L(Vλ+1 ). Archive for Math-
ematical Logic 50 (2011), 565–584.
[8] V. Dimonte, A partially non-proper ordinal beyond L(Vλ+1 ). Annals of Pure
and Applied Logic 163 (2012), 1309–1321.
[9] V. Dimonte, S. Friedman, Rank-into-rank hypotheses and the failure of GCH.
Archive for Mathematical Logic 53 (2014), 351–366.
[10] V. Dimonte, L. Wu, A general tool for consistency results related to I1.
European Journal of Mathematics 2 (2016), 474–492.
[11] V. Dimonte, L. Wu, The *-Prikry condition. Unpublished manuscript.
[12] N. Dobrinen, S. Friedman, The consistency strength of the tree property at
the double successor of a measurable cardinal. Fundamenta mathematicae
208 (2010), 123–153.
[13] R. Dougherty, T.J. Jech, Finite left-distributive algebras and embedding
algebras. Advances in Mathematics 130 (1997), 201–241.
[14] W. B. Easton, Powers of regular cardinals. Annals of Mathematical Logic
1 (1970), 139–178.
[15] H. Gaifman, Elementary embeddings of models of set theory and certain
subtheories. Axiomatic Set Theory, Proceedings of the Symposium in Pure
Mathematics 13 part 2 (1974), 33–101.
[16] M. Foreman, A. Kanamori (eds), Handbook of Set theory. Springer, Dor-
drecht (2010).

66
[17] S.-D. Friedman, Large cardinals and L-like universes. Set Theory: Recent
Trends and Applications (A. Andretta editor). Quaderni di Matematica 17
(2006), 93–110.
[18] M. Gitik, Blowing up the power of a singular cardinal - wider gaps. Annals
of Pure and Applied Logic 116 (2002), 1–38.
[19] M. Gitik, Prikry-type forcings. Handbook of set theory, Vols 2, 1351–1447.
Springer, Dordrecht 2010.
[20] G. Kafkoulis, Coding lemmata in L(Vλ+1 ). Archive for Mathematical Logic
43 (2004), 193–213.
[21] A. Kanamori, The Higher Infinite. Springer, Berlin (1994).
[22] J. Hamkins, Fragile measurability. Journal of Symbolic Logic 59 (1994),
262–282.
[23] J. D. Hamkins, G. Kirmayer, and N. L. Perlmutter, Generalizations of the
Kunen inconsistency. Annals of Pure and Applied Logic 163 (2012) 1872–
1890.
[24] K. Kunen, Elementary embeddings and infinite combinatorics. Journal of
Symbolic Logic 36 (1971), 407–413.
[25] R. Laver, The left distributive law and the freeness of an algebra of elemen-
tary embeddings. Advances in Mathematics 91 (1992), 209–231.
[26] R. Laver, On the algebra of elementary embeddings of a rank into itself.
Advances in Mathematics 110 (1995), 334–346.
[27] R. Laver, Implications between strong large cardinal axioms. Annals of Pure
and Applied Logic 90 (1997), 79–90.
[28] R. Laver, Reflection of elementary embedding axioms on the L(Vλ+1 ) hier-
archy. Annals of Pure and Applied Logic 107 (2001), 227–238.
[29] D.A. Martin, Infinite games. Proceedings of the ICM, Helsinki, 1978, Acad.
Sci. Fennica, Helsinki, 269–273.
[30] A. R. D. Mathias, On sequences generic in the sense of Prikry. Journal of
the Australian Mathematical Society 15 (1973), 409–414.
[31] C. Merimovich, Extender Based Radin Forcing. Transactions of the Amer-
ican Mathematical Society 355 (2003), 1729–1772.
[32] Y.N. Moschovakis, Determinacy and prewellorderings of the continuum. In
Y. Bar-Hillel, Mathematical Logic and Foundations of Set Theory, Amster-
dam, North-Holland, 1970.
[33] I. Neeman, Aronszajn trees and failure of the Singular Cardinal Hypothesis.
Journal of Mathematical Logic 9 (2009), 139–157.
[34] W. Powell, Variations of Keisler’s theorem for complete embeddings. Fun-
damenta Mathematicae 81 (1974), 121–132.

67
[35] X. Shi, Axiom I0 and higher degree theory. Journal of Symbolic Logic 80
(2015), 970–1021.
[36] X. Shi, N. Trang, I0 and combinatorics at λ+ . Archive for Mathematical
Logic 56 (2017), 131–154.
[37] J. Steel, Scales in L(R). Cabal Seminar 79–81, 107–156, Lecture Notes in
Mathematics, 1019, Springer, Berlin, 1983.
[38] A. Suzuki, No elementary embedding from V into V is definable from pa-
rameters. Journal of Symbolic Logic, 64 (1999), 1591–1594.
[39] W. H. Woodin, An AD-like Axiom. Lecture notes written by George Kafk-
oulis, 1989
[40] W. H. Woodin, Suitable extender models I. Journal of Mathematical Logic,
10 (2010) 101–339.
[41] W. H. Woodin, Suitable extender models II: beyond ω-huge. Journal of
Mathematical Logic, 11 (2011) 115–436.
[42] W. H. Woodin, The Weak Ultrafilter Axiom. Archive for Mathematical
Logic, 55 (2016) 319–351.
[43] W. H. Woodin, J. Davis, D. Rodrı́guez, The HOD Dichotomy. Unpublished
lecture notes.

68

You might also like